Programs & Examples On #Dbo

How to compare oldValues and newValues on React Hooks useEffect?

Here's a custom hook that I use which I believe is more intuitive than using usePrevious.

import { useRef, useEffect } from 'react'

// useTransition :: Array a => (a -> Void, a) -> Void
//                              |_______|  |
//                                  |      |
//                              callback  deps
//
// The useTransition hook is similar to the useEffect hook. It requires
// a callback function and an array of dependencies. Unlike the useEffect
// hook, the callback function is only called when the dependencies change.
// Hence, it's not called when the component mounts because there is no change
// in the dependencies. The callback function is supplied the previous array of
// dependencies which it can use to perform transition-based effects.
const useTransition = (callback, deps) => {
  const func = useRef(null)

  useEffect(() => {
    func.current = callback
  }, [callback])

  const args = useRef(null)

  useEffect(() => {
    if (args.current !== null) func.current(...args.current)
    args.current = deps
  }, deps)
}

You'd use useTransition as follows.

useTransition((prevRate, prevSendAmount, prevReceiveAmount) => {
  if (sendAmount !== prevSendAmount || rate !== prevRate && sendAmount > 0) {
    const newReceiveAmount = sendAmount * rate
    // do something
  } else {
    const newSendAmount = receiveAmount / rate
    // do something
  }
}, [rate, sendAmount, receiveAmount])

Hope that helps.

React Hook Warnings for async function in useEffect: useEffect function must return a cleanup function or nothing

When you use an async function like

async () => {
    try {
        const response = await fetch(`https://www.reddit.com/r/${subreddit}.json`);
        const json = await response.json();
        setPosts(json.data.children.map(it => it.data));
    } catch (e) {
        console.error(e);
    }
}

it returns a promise and useEffect doesn't expect the callback function to return Promise, rather it expects that nothing is returned or a function is returned.

As a workaround for the warning you can use a self invoking async function.

useEffect(() => {
    (async function() {
        try {
            const response = await fetch(
                `https://www.reddit.com/r/${subreddit}.json`
            );
            const json = await response.json();
            setPosts(json.data.children.map(it => it.data));
        } catch (e) {
            console.error(e);
        }
    })();
}, []);

or to make it more cleaner you could define a function and then call it

useEffect(() => {
    async function fetchData() {
        try {
            const response = await fetch(
                `https://www.reddit.com/r/${subreddit}.json`
            );
            const json = await response.json();
            setPosts(json.data.children.map(it => it.data));
        } catch (e) {
            console.error(e);
        }
    };
    fetchData();
}, []);

the second solution will make it easier to read and will help you write code to cancel previous requests if a new one is fired or save the latest request response in state

Working codesandbox

Flutter: RenderBox was not laid out

I had a simmilar problem, but in my case I was put a row in the leading of the Listview, and it was consumming all the space, of course. I just had to take the Row out of the leading, and it was solved. I would recomend to check if the problem is a widget larger than its containner can have.

Flutter - The method was called on null

The reason for this error occurs is that you are using the CryptoListPresenter _presenter without initializing.

I found that CryptoListPresenter _presenter would have to be initialized to fix because _presenter.loadCurrencies() is passing through a null variable at the time of instantiation;

there are two ways to initialize

  1. Can be initialized during an declaration, like this

    CryptoListPresenter _presenter = CryptoListPresenter();
    
  2. In the second, initializing(with assigning some value) it when initState is called, which the framework will call this method once for each state object.

    @override
    void initState() {
      _presenter = CryptoListPresenter(...);
    }
    

What is the Record type in typescript?

A Record lets you create a new type from a Union. The values in the Union are used as attributes of the new type.

For example, say I have a Union like this:

type CatNames = "miffy" | "boris" | "mordred";

Now I want to create an object that contains information about all the cats, I can create a new type using the values in the CatName Union as keys.

type CatList = Record<CatNames, {age: number}>

If I want to satisfy this CatList, I must create an object like this:

const cats:CatList = {
  miffy: { age:99 },
  boris: { age:16 },
  mordred: { age:600 }
}

You get very strong type safety:

  • If I forget a cat, I get an error.
  • If I add a cat that's not allowed, I get an error.
  • If I later change CatNames, I get an error. This is especially useful because CatNames is likely imported from another file, and likely used in many places.

Real-world React example.

I used this recently to create a Status component. The component would receive a status prop, and then render an icon. I've simplified the code quite a lot here for illustrative purposes

I had a union like this:

type Statuses = "failed" | "complete";

I used this to create an object like this:

const icons: Record<
  Statuses,
  { iconType: IconTypes; iconColor: IconColors }
> = {
  failed: {
    iconType: "warning",
    iconColor: "red"
  },
  complete: {
    iconType: "check",
    iconColor: "green"
  };

I could then render by destructuring an element from the object into props, like so:

const Status = ({status}) => <Icon {...icons[status]} />

If the Statuses union is later extended or changed, I know my Status component will fail to compile and I'll get an error that I can fix immediately. This allows me to add additional error states to the app.

Note that the actual app had dozens of error states that were referenced in multiple places, so this type safety was extremely useful.

How to scroll page in flutter

You can try CustomScrollView. Put your CustomScrollView inside Column Widget.

Just for example -

class App extends StatelessWidget {

 App({Key key}): super(key: key);

  @override
  Widget build(BuildContext context) {
    return new Scaffold(
      appBar: AppBar(
          title: const Text('AppBar'),
      ),
      body: new Container(
          constraints: BoxConstraints.expand(),
          decoration: new BoxDecoration(
            image: new DecorationImage(
              alignment: Alignment.topLeft,
              image: new AssetImage('images/main-bg.png'),
              fit: BoxFit.cover,
            )
          ),
          child: new Column(
            children: <Widget>[               
              Expanded(
                child: new CustomScrollView(
                  scrollDirection: Axis.vertical,
                  shrinkWrap: false,
                  slivers: <Widget>[
                    new SliverPadding(
                      padding: const EdgeInsets.symmetric(vertical: 0.0),
                      sliver: new SliverList(
                        delegate: new SliverChildBuilderDelegate(
                          (context, index) => new YourRowWidget(),
                          childCount: 5,
                        ),
                      ),
                    ),
                  ],
                ),
              ),
            ],
          )),
    );
  }
}

In above code I am displaying a list of items ( total 5) in CustomScrollView. YourRowWidget widget gets rendered 5 times as list item. Generally you should render each row based on some data.

You can remove decoration property of Container widget, it is just for providing background image.

Under which circumstances textAlign property works in Flutter?

For maximum flexibility, I usually prefer working with SizedBox like this:

Row(
                                children: <Widget>[
                                  SizedBox(
                                      width: 235,
                                      child: Text('Hey, ')),
                                  SizedBox(
                                      width: 110,
                                      child: Text('how are'),
                                  SizedBox(
                                      width: 10,
                                      child: Text('you?'))
                                ],
                              )

I've experienced problems with text alignment when using alignment in the past, whereas sizedbox always does the work.

react button onClick redirect page

This can be done very simply, you don't need to use a different function or library for it.

onClick={event =>  window.location.href='/your-href'}

How to set the width of a RaisedButton in Flutter?

you can do as they say in the comments or you can save the effort and work with RawMaterialButton . which have everything and you can change the border to be circular and alot of other attributes. ex shape(increase the radius to have more circular shape)

shape: new RoundedRectangleBorder(borderRadius: BorderRadius.circular(25)),//ex add 1000 instead of 25

and you can use whatever shape you want as a button by using GestureDetector which is a widget and accepts another widget under child attribute. like in the other example here

GestureDetector(
onTap: () {//handle the press action here }
child:Container(

              height: 80,
              width: 80,
              child:new Card(

                color: Colors.blue,
                shape: new RoundedRectangleBorder(borderRadius: BorderRadius.circular(25)),
                elevation: 0.0,                
              child: Icon(Icons.add,color: Colors.white,),
              ),
              )
)

How to remove whitespace from a string in typescript?

Trim just removes the trailing and leading whitespace. Use .replace(/ /g, "") if there are just spaces to be replaced.

this.maintabinfo = this.inner_view_data.replace(/ /g, "").toLowerCase();

You should not use <Link> outside a <Router>

If you don't want to change much, use below code inside onClick()method.

this.props.history.push('/');

How to work with progress indicator in flutter?

For me, one neat way to do this is to show a SnackBar at the bottom while the Signing-In process is taken place, this is a an example of what I mean:

enter image description here

Here is how to setup the SnackBar.

Define a global key for your Scaffold

final GlobalKey<ScaffoldState> _scaffoldKey = new GlobalKey<ScaffoldState>();

Add it to your Scaffold key attribute

return new Scaffold(
      key: _scaffoldKey,
.......

My SignIn button onPressed callback:

onPressed: () {
                  _scaffoldKey.currentState.showSnackBar(
                      new SnackBar(duration: new Duration(seconds: 4), content:
                      new Row(
                        children: <Widget>[
                          new CircularProgressIndicator(),
                          new Text("  Signing-In...")
                        ],
                      ),
                      ));
                  _handleSignIn()
                      .whenComplete(() =>
                      Navigator.of(context).pushNamed("/Home")
                  );
                }

It really depends on how you want to build your layout, and I am not sure what you have in mind.

Edit

You probably want it this way, I have used a Stack to achieve this result and just show or hide my indicator based on onPressed

enter image description here

class TestSignInView extends StatefulWidget {
  @override
  _TestSignInViewState createState() => new _TestSignInViewState();
}


class _TestSignInViewState extends State<TestSignInView> {
  bool _load = false;
  @override
  Widget build(BuildContext context) {
    Widget loadingIndicator =_load? new Container(
      color: Colors.grey[300],
      width: 70.0,
      height: 70.0,
      child: new Padding(padding: const EdgeInsets.all(5.0),child: new Center(child: new CircularProgressIndicator())),
    ):new Container();
    return new Scaffold(
      backgroundColor: Colors.white,
      body:  new Stack(children: <Widget>[new Padding(
        padding: const EdgeInsets.symmetric(vertical: 50.0, horizontal: 20.0),
        child: new ListView(

          children: <Widget>[
            new Column(
              mainAxisAlignment: MainAxisAlignment.center,
              crossAxisAlignment: CrossAxisAlignment.center
              ,children: <Widget>[
            new TextField(),
            new TextField(),

            new FlatButton(color:Colors.blue,child: new Text('Sign In'),
                onPressed: () {
              setState((){
                _load=true;
              });

                  //Navigator.of(context).push(new MaterialPageRoute(builder: (_)=>new HomeTest()));
                }
            ),

            ],),],
        ),),
        new Align(child: loadingIndicator,alignment: FractionalOffset.center,),

      ],));
  }

}

groovy.lang.MissingPropertyException: No such property: jenkins for class: groovy.lang.Binding

in my case I have used - (Hyphen) in my script name in case of Jenkinsfile Library. Got resolved after replacing Hyphen(-) with Underscore(_)

Typescript Date Type?

The answer is super simple, the type is Date:

const d: Date = new Date(); // but the type can also be inferred from "new Date()" already

It is the same as with every other object instance :)

Setting up Gradle for api 26 (Android)

allprojects {
    repositories {
        jcenter()
        maven {
            url "https://maven.google.com"
        }
    }
}

android {
    compileSdkVersion 26
    buildToolsVersion "26.0.1"
    defaultConfig {
        applicationId "com.keshav.retroft2arrayinsidearrayexamplekeshav"
        minSdkVersion 15
        targetSdkVersion 26
        versionCode 1
        versionName "1.0"
        testInstrumentationRunner "android.support.test.runner.AndroidJUnitRunner"
    }
    buildTypes {
        release {
            minifyEnabled false
            proguardFiles getDefaultProguardFile('proguard-android.txt'), 'proguard-rules.pro'
        }
    }
}
 compile 'com.android.support:appcompat-v7:26.0.1'
    compile 'com.android.support:recyclerview-v7:26.0.1'
    compile 'com.android.support:cardview-v7:26.0.1'

How to print a Groovy variable in Jenkins?

You shouldn't use ${varName} when you're outside of strings, you should just use varName. Inside strings you use it like this; echo "this is a string ${someVariable}";. Infact you can place an general java expression inside of ${...}; echo "this is a string ${func(arg1, arg2)}.

Error: the entity type requires a primary key

Make sure you have the following condition:

  1. Use [key] if your primary key name is not Id or ID.
  2. Use the public keyword.
  3. Primary key should have getter and setter.

Example:

public class MyEntity {
   [key]
   public Guid Id {get; set;}
}

How to use paths in tsconfig.json?

/ starts from the root only, to get the relative path we should use ./ or ../

Typescript ReferenceError: exports is not defined

Simply add libraryTarget: 'umd', like so

const webpackConfig = {
  output: {
    libraryTarget: 'umd' // Fix: "Uncaught ReferenceError: exports is not defined".
  }
};

module.exports = webpackConfig; // Export all custom Webpack configs.

How do I pass a list as a parameter in a stored procedure?

The preferred method for passing an array of values to a stored procedure in SQL server is to use table valued parameters.

First you define the type like this:

CREATE TYPE UserList AS TABLE ( UserID INT );

Then you use that type in the stored procedure:

create procedure [dbo].[get_user_names]
@user_id_list UserList READONLY,
@username varchar (30) output
as
select last_name+', '+first_name 
from user_mstr
where user_id in (SELECT UserID FROM @user_id_list)

So before you call that stored procedure, you fill a table variable:

DECLARE @UL UserList;
INSERT @UL VALUES (5),(44),(72),(81),(126)

And finally call the SP:

EXEC dbo.get_user_names @UL, @username OUTPUT;

Could not find a declaration file for module 'module-name'. '/path/to/module-name.js' implicitly has an 'any' type

You have to do is edit your TypeScript Config file (tsconfig.json) and add a new key-value pair as:

"noImplicitAny": false

For Example: "noImplicitAny": false, "allowJs": true, "skipLibCheck": true, "esModuleInterop": true, continue....

Why does C++ code for testing the Collatz conjecture run faster than hand-written assembly?

You did not post the code generated by the compiler, so there' some guesswork here, but even without having seen it, one can say that this:

test rax, 1
jpe even

... has a 50% chance of mispredicting the branch, and that will come expensive.

The compiler almost certainly does both computations (which costs neglegibly more since the div/mod is quite long latency, so the multiply-add is "free") and follows up with a CMOV. Which, of course, has a zero percent chance of being mispredicted.

How to use the COLLATE in a JOIN in SQL Server?

Correct syntax looks like this. See MSDN.

SELECT *
  FROM [FAEB].[dbo].[ExportaComisiones] AS f
  JOIN [zCredifiel].[dbo].[optPerson] AS p

  ON p.vTreasuryId COLLATE Latin1_General_CI_AS = f.RFC COLLATE Latin1_General_CI_AS 

"Port 4200 is already in use" when running the ng serve command

For Windows:

Open Command Prompt and

type: netstat -a -o -n

Find the PID of the process that you want to kill.

Type: taskkill /F /PID 16876

This one 16876 - is the PID for the process that I want to kill - in that case, the process is 4200 - check the attached file.you can give any port number.

enter image description here

Now, Type : ng serve to start your angular app at the same port 4200

Jenkins CI Pipeline Scripts not permitted to use method groovy.lang.GroovyObject

You have to disable the sandbox for Groovy in your job configuration.

Currently this is not possible for multibranch projects where the groovy script comes from the scm. For more information see https://issues.jenkins-ci.org/browse/JENKINS-28178

Stored procedure with default parameters

I'd do this one of two ways. Since you're setting your start and end dates in your t-sql code, i wouldn't ask for parameters in the stored proc

Option 1

Create Procedure [Test] AS
    DECLARE @StartDate varchar(10)
    DECLARE @EndDate varchar(10)
    Set @StartDate = '201620' --Define start YearWeek
    Set @EndDate  = (SELECT CAST(DATEPART(YEAR,getdate()) AS varchar(4)) + CAST(DATEPART(WEEK,getdate())-1 AS varchar(2)))

SELECT 
*
FROM
    (SELECT DISTINCT [YEAR],[WeekOfYear] FROM [dbo].[DimDate] WHERE [Year]+[WeekOfYear] BETWEEN @StartDate AND @EndDate ) dimd
    LEFT JOIN [Schema].[Table1] qad ON (qad.[Year]+qad.[Week of the Year]) = (dimd.[Year]+dimd.WeekOfYear)

Option 2

Create Procedure [Test] @StartDate varchar(10),@EndDate varchar(10) AS

SELECT 
*
FROM
    (SELECT DISTINCT [YEAR],[WeekOfYear] FROM [dbo].[DimDate] WHERE [Year]+[WeekOfYear] BETWEEN @StartDate AND @EndDate ) dimd
    LEFT JOIN [Schema].[Table1] qad ON (qad.[Year]+qad.[Week of the Year]) = (dimd.[Year]+dimd.WeekOfYear)

Then run exec test '2016-01-01','2016-01-25'

Violation of PRIMARY KEY constraint. Cannot insert duplicate key in object

To prevent inserting a record that exist already. I'd check if the ID value exists in the database. For the example of a Table created with an IDENTITY PRIMARY KEY:

CREATE TABLE [dbo].[Persons] (    
    ID INT IDENTITY(1,1) PRIMARY KEY,
    LastName VARCHAR(40) NOT NULL,
    FirstName VARCHAR(40)
);

When JANE DOE and JOE BROWN already exist in the database.

SET IDENTITY_INSERT [dbo].[Persons] OFF;
INSERT INTO [dbo].[Persons] (FirstName,LastName)
VALUES ('JANE','DOE'); 
INSERT INTO Persons (FirstName,LastName) 
VALUES ('JOE','BROWN');

DATABASE OUTPUT of TABLE [dbo].[Persons] will be:

ID    LastName   FirstName
1     DOE        Jane
2     BROWN      JOE

I'd check if i should update an existing record or insert a new one. As the following JAVA example:

int NewID = 1;
boolean IdAlreadyExist = false;
// Using SQL database connection
// STEP 1: Set property
System.setProperty("java.net.preferIPv4Stack", "true");
// STEP 2: Register JDBC driver
Class.forName("com.microsoft.sqlserver.jdbc.SQLServerDriver");
// STEP 3: Open a connection
try (Connection conn1 = DriverManager.getConnection(DB_URL, USER,pwd) {
    conn1.setAutoCommit(true);
    String Select = "select * from Persons where  ID = " + ID;
    Statement st1 = conn1.createStatement();
    ResultSet rs1 = st1.executeQuery(Select);
    // iterate through the java resultset
    while (rs1.next()) {
        int ID = rs1.getInt("ID");
        if (NewID==ID) {
            IdAlreadyExist = true;
        }

    }
    conn1.close();
} catch (SQLException e1) {
    System.out.println(e1);
}
if (IdAlreadyExist==false) {
    //Insert new record code here
} else {
    //Update existing record code here
}

Could not find server 'server name' in sys.servers. SQL Server 2014

At first check out that your linked server is in the list by this query

select name from sys.servers

If it not exists then try to add to the linked server

EXEC sp_addlinkedserver @server = 'SERVER_NAME' --or may be server ip address

After that login to that linked server by

EXEC sp_addlinkedsrvlogin 'SERVER_NAME'
                         ,'false'
                         ,NULL
                         ,'USER_NAME'
                         ,'PASSWORD'

Then you can do whatever you want ,treat it like your local server

exec [SERVER_NAME].[DATABASE_NAME].dbo.SP_NAME @sample_parameter

Finally you can drop that server from linked server list by

sp_dropserver 'SERVER_NAME', 'droplogins'

If it will help you then please upvote.

Typescript import/as vs import/require?

These are mostly equivalent, but import * has some restrictions that import ... = require doesn't.

import * as creates an identifier that is a module object, emphasis on object. According to the ES6 spec, this object is never callable or newable - it only has properties. If you're trying to import a function or class, you should use

import express = require('express');

or (depending on your module loader)

import express from 'express';

Attempting to use import * as express and then invoking express() is always illegal according to the ES6 spec. In some runtime+transpilation environments this might happen to work anyway, but it might break at any point in the future without warning, which will make you sad.

Copy output of a JavaScript variable to the clipboard

function copyToClipboard(text) {
    var dummy = document.createElement("textarea");
    // to avoid breaking orgain page when copying more words
    // cant copy when adding below this code
    // dummy.style.display = 'none'
    document.body.appendChild(dummy);
    //Be careful if you use texarea. setAttribute('value', value), which works with "input" does not work with "textarea". – Eduard
    dummy.value = text;
    dummy.select();
    document.execCommand("copy");
    document.body.removeChild(dummy);
}
copyToClipboard('hello world')
copyToClipboard('hello\nworld')

Read and write a text file in typescript

First you will need to install node definitions for Typescript. You can find the definitions file here:

https://github.com/DefinitelyTyped/DefinitelyTyped/blob/master/node/node.d.ts

Once you've got file, just add the reference to your .ts file like this:

/// <reference path="path/to/node.d.ts" />

Then you can code your typescript class that read/writes, using the Node File System module. Your typescript class myClass.ts can look like this:

/// <reference path="path/to/node.d.ts" />

class MyClass {

    // Here we import the File System module of node
    private fs = require('fs');

    constructor() { }

    createFile() {

        this.fs.writeFile('file.txt', 'I am cool!',  function(err) {
            if (err) {
                return console.error(err);
            }
            console.log("File created!");
        });
    }

    showFile() {

        this.fs.readFile('file.txt', function (err, data) {
            if (err) {
                return console.error(err);
            }
            console.log("Asynchronous read: " + data.toString());
        });
    }
}

// Usage
// var obj = new MyClass();
// obj.createFile();
// obj.showFile();

Once you transpile your .ts file to a javascript (check out here if you don't know how to do it), you can run your javascript file with node and let the magic work:

> node myClass.js

SQL Server: Error converting data type nvarchar to numeric

You might need to revise the data in the column, but anyway you can do one of the following:-

1- check if it is numeric then convert it else put another value like 0

Select COLUMNA AS COLUMNA_s, CASE WHEN Isnumeric(COLUMNA) = 1
THEN CONVERT(DECIMAL(18,2),COLUMNA) 
ELSE 0 END AS COLUMNA

2- select only numeric values from the column

SELECT COLUMNA AS COLUMNA_s ,CONVERT(DECIMAL(18,2),COLUMNA) AS COLUMNA
where Isnumeric(COLUMNA) = 1

What is the syntax for Typescript arrow functions with generics?

while the popular answer with extends {} works and is better than extends any, it forces the T to be an object

const foo = <T extends {}>(x: T) => x;

to avoid this and preserve the type-safety, you can use extends unknown instead

const foo = <T extends unknown>(x: T) => x;

How to solve ERR_CONNECTION_REFUSED when trying to connect to localhost running IISExpress - Error 502 (Cannot debug from Visual Studio)?

I tried a lot of methods on Chrome but the only thing that worked for me was "Clear browsing data"

Had to do the "advanced" version because standard didn't work. I suspect it was "Content Settings" that was doing it.

Send POST parameters with MultipartFormData using Alamofire, in iOS Swift

Well, since Multipart Form Data is intended to be used for binary ( and not for text) data transmission, I believe it's bad practice to send data in encoded to String over it.

Another disadvantage is impossibility to send more complex parameters like JSON.

That said, a better option would be to send all data in binary form, that is as Data.

Say I need to send this data

let name = "Arthur"
let userIDs = [1,2,3]
let usedAge = 20

...alongside with user's picture:

let image = UIImage(named: "img")!

For that I would convert that text data to JSON and then to binary alongside with image:

//Convert image to binary
let data = UIImagePNGRepresentation(image)!

//Convert text data to binary
let dict: Dictionary<String, Any> = ["name": name, "userIDs": userIDs, "usedAge": usedAge]
userData = try? JSONSerialization.data(withJSONObject: dict)

And then, finally send it via Multipart Form Data request:

Alamofire.upload(multipartFormData: { (multiFoormData) in
        multiFoormData.append(userData, withName: "user")
        multiFoormData.append(data, withName: "picture", mimeType: "image/png")
    }, to: url) { (encodingResult) in
        ...
    }

Casting int to bool in C/C++

The following cites the C11 standard (final draft).

6.3.1.2: When any scalar value is converted to _Bool, the result is 0 if the value compares equal to 0; otherwise, the result is 1.

bool (mapped by stdbool.h to the internal name _Bool for C) itself is an unsigned integer type:

... The type _Bool and the unsigned integer types that correspond to the standard signed integer types are the standard unsigned integer types.

According to 6.2.5p2:

An object declared as type _Bool is large enough to store the values 0 and 1.

AFAIK these definitions are semantically identical to C++ - with the minor difference of the built-in(!) names. bool for C++ and _Bool for C.

Note that C does not use the term rvalues as C++ does. However, in C pointers are scalars, so assigning a pointer to a _Bool behaves as in C++.

Sending cookies with postman

I used postman chrome extension until it became deprecated. Chrome extension also less usable and powerful then native postman application. So, it became not very convenient to use chrome extension. I have found next approach:

  1. copy any request in chrome/any other browser as CURL request (image 1)
  2. import to postman copied request (image 2)
  3. save imported request in postman's list

copy curl request image 1

enter image description here image 2

Spark read file from S3 using sc.textFile ("s3n://...)

Ran into the same problem in Spark 2.0.2. Resolved it by feeding it the jars. Here's what I ran:

$ spark-shell --jars aws-java-sdk-1.7.4.jar,hadoop-aws-2.7.3.jar,jackson-annotations-2.7.0.jar,jackson-core-2.7.0.jar,jackson-databind-2.7.0.jar,joda-time-2.9.6.jar

scala> val hadoopConf = sc.hadoopConfiguration
scala> hadoopConf.set("fs.s3.impl","org.apache.hadoop.fs.s3native.NativeS3FileSystem")
scala> hadoopConf.set("fs.s3.awsAccessKeyId",awsAccessKeyId)
scala> hadoopConf.set("fs.s3.awsSecretAccessKey", awsSecretAccessKey)
scala> val sqlContext = new org.apache.spark.sql.SQLContext(sc)
scala> sqlContext.read.parquet("s3://your-s3-bucket/")

obviously, you need to have the jars in the path where you're running spark-shell from

Pandas unstack problems: ValueError: Index contains duplicate entries, cannot reshape

Here's an example DataFrame which show this, it has duplicate values with the same index. The question is, do you want to aggregate these or keep them as multiple rows?

In [11]: df
Out[11]:
   0  1  2      3
0  1  2  a  16.86
1  1  2  a  17.18
2  1  4  a  17.03
3  2  5  b  17.28

In [12]: df.pivot_table(values=3, index=[0, 1], columns=2, aggfunc='mean')  # desired?
Out[12]:
2        a      b
0 1
1 2  17.02    NaN
  4  17.03    NaN
2 5    NaN  17.28

In [13]: df1 = df.set_index([0, 1, 2])

In [14]: df1
Out[14]:
           3
0 1 2
1 2 a  16.86
    a  17.18
  4 a  17.03
2 5 b  17.28

In [15]: df1.unstack(2)
ValueError: Index contains duplicate entries, cannot reshape

One solution is to reset_index (and get back to df) and use pivot_table.

In [16]: df1.reset_index().pivot_table(values=3, index=[0, 1], columns=2, aggfunc='mean')
Out[16]:
2        a      b
0 1
1 2  17.02    NaN
  4  17.03    NaN
2 5    NaN  17.28

Another option (if you don't want to aggregate) is to append a dummy level, unstack it, then drop the dummy level...

Base64: java.lang.IllegalArgumentException: Illegal character

The Base64.Encoder.encodeToString method automatically uses the ISO-8859-1 character set.

For an encryption utility I am writing, I took the input string of cipher text and Base64 encoded it for transmission, then reversed the process. Relevant parts shown below. NOTE: My file.encoding property is set to ISO-8859-1 upon invocation of the JVM so that may also have a bearing.

static String getBase64EncodedCipherText(String cipherText) {
    byte[] cText = cipherText.getBytes();
    // return an ISO-8859-1 encoded String
    return Base64.getEncoder().encodeToString(cText);
}

static String getBase64DecodedCipherText(String encodedCipherText) throws IOException {
    return new String((Base64.getDecoder().decode(encodedCipherText)));
}

public static void main(String[] args) {
    try {
        String cText = getRawCipherText(null, "Hello World of Encryption...");

        System.out.println("Text to encrypt/encode: Hello World of Encryption...");
        // This output is a simple sanity check to display that the text
        // has indeed been converted to a cipher text which 
        // is unreadable by all but the most intelligent of programmers.
        // It is absolutely inhuman of me to do such a thing, but I am a
        // rebel and cannot be trusted in any way.  Please look away.
        System.out.println("RAW CIPHER TEXT: " + cText);
        cText = getBase64EncodedCipherText(cText);
        System.out.println("BASE64 ENCODED: " + cText);
        // There he goes again!!
        System.out.println("BASE64 DECODED:  " + getBase64DecodedCipherText(cText));
        System.out.println("DECODED CIPHER TEXT: " + decodeRawCipherText(null, getBase64DecodedCipherText(cText)));
    } catch (Exception e) {
        e.printStackTrace();
    }

}

The output looks like:

Text to encrypt/encode: Hello World of Encryption...
RAW CIPHER TEXT: q$;?C?l??<8??U???X[7l
BASE64 ENCODED: HnEPJDuhQ+qDbInUCzw4gx0VDqtVwef+WFs3bA==
BASE64 DECODED:  q$;?C?l??<8??U???X[7l``
DECODED CIPHER TEXT: Hello World of Encryption...

EXEC sp_executesql with multiple parameters

This also works....sometimes you may want to construct the definition of the parameters outside of the actual EXEC call.

DECLARE @Parmdef nvarchar (500)
DECLARE @SQL nvarchar (max)
DECLARE @xTxt1  nvarchar (100) = 'test1'
DECLARE @xTxt2  nvarchar (500) = 'test2' 
SET @parmdef = '@text1 nvarchar (100), @text2 nvarchar (500)'
SET @SQL = 'PRINT @text1 + '' '' + @text2'
EXEC sp_executeSQL @SQL, @Parmdef, @xTxt1, @xTxt2

Spring Boot - Error creating bean with name 'dataSource' defined in class path resource

Looks like the initial problem is with the auto-config.

If you don't need the datasource, simply remove it from the auto-config process:

@EnableAutoConfiguration(exclude={DataSourceAutoConfiguration.class})

How to load local file in sc.textFile, instead of HDFS

If your trying to read file form HDFS. trying setting path in SparkConf

 val conf = new SparkConf().setMaster("local[*]").setAppName("HDFSFileReader")
 conf.set("fs.defaultFS", "hdfs://hostname:9000")

React Error: Target Container is not a DOM Element

Just to give an alternative solution, because it isn't mentioned.

It's perfectly fine to use the HTML attribute defer here. So when loading the DOM, a regular <script> will load when the DOM hits the script. But if we use defer, then the DOM and the script will load in parallel. The cool thing is the script gets evaluated in the end - when the DOM has loaded (source).

<script src="{% static "build/react.js" %}" defer></script>

NameError: uninitialized constant (rails)

Similar with @Michael-Neal.

I had named the controller as singular. app/controllers/product_controller.rb

When I renamed it as plural, error solved. app/controllers/products_controller.rb

There is already an object named in the database

"There is already an object named 'AboutUs' in the database."

This exception tells you that somebody has added an object named 'AboutUs' to the database already.

AutomaticMigrationsEnabled = true; can lead to it since data base versions are not controlled by you in this case. In order to avoid unpredictable migrations and make sure that every developer on the team works with the same data base structure I suggest you set AutomaticMigrationsEnabled = false;.

Automatic migrations and Coded migrations can live alongside if you are very careful and the only one developer on a project.

There is a quote from Automatic Code First Migrations post on Data Developer Center:

Automatic Migrations allows you to use Code First Migrations without having a code file in your project for each change you make. Not all changes can be applied automatically - for example column renames require the use of a code-based migration.

Recommendation for Team Environments

You can intersperse automatic and code-based migrations but this is not recommended in team development scenarios. If you are part of a team of developers that use source control you should either use purely automatic migrations or purely code-based migrations. Given the limitations of automatic migrations we recommend using code-based migrations in team environments.

The OLE DB provider "Microsoft.ACE.OLEDB.12.0" for linked server "(null)"

Instead of changing the user, I've found this advise:

https://social.technet.microsoft.com/Forums/lync/en-US/bb2dc720-f8f9-4b93-b5d1-cfb4f8a8b1cb/the-ole-db-provider-microsoftaceoledb120-for-linked-server-null-reported-an-error-access?forum=sqldataaccess

This might help someone else out - after trying every solution to trying and fix this error on SQL 64..

Cannot initialize the data source object of OLE DB provider "Microsoft.ACE.OLEDB.12.0" for linked server "(null)".

..I found an article here...

http://sqlserverpedia.com/blog/sql-server-bloggers/too-many-bits/

..which suggested I give Everyone full permission on this folder..

C:\Users\SQL Service account name\AppData\Local\Temp

And hey presto! My query suddenly burst into life. I punched the air in delight.

Edwaldo

How to define the basic HTTP authentication using cURL correctly?

curl -u username:password http://
curl -u username http://

From the documentation page:

-u, --user <user:password>

Specify the user name and password to use for server authentication. Overrides -n, --netrc and --netrc-optional.

If you simply specify the user name, curl will prompt for a password.

The user name and passwords are split up on the first colon, which makes it impossible to use a colon in the user name with this option. The password can, still.

When using Kerberos V5 with a Windows based server you should include the Windows domain name in the user name, in order for the server to succesfully obtain a Kerberos Ticket. If you don't then the initial authentication handshake may fail.

When using NTLM, the user name can be specified simply as the user name, without the domain, if there is a single domain and forest in your setup for example.

To specify the domain name use either Down-Level Logon Name or UPN (User Principal Name) formats. For example, EXAMPLE\user and [email protected] respectively.

If you use a Windows SSPI-enabled curl binary and perform Kerberos V5, Negotiate, NTLM or Digest authentication then you can tell curl to select the user name and password from your environment by specifying a single colon with this option: "-u :".

If this option is used several times, the last one will be used.

http://curl.haxx.se/docs/manpage.html#-u

Note that you do not need --basic flag as it is the default.

iOS8 Beta Ad-Hoc App Download (itms-services)

Specify a 'display-image' and 'full-size-image' as described here: http://www.informit.com/articles/article.aspx?p=1829415&seqNum=16

iOS8 requires these images

SQL Server after update trigger

First off, your trigger as you already see is going to update every record in the table. There is no filtering done to accomplish jus the rows changed.

Secondly, you're assuming that only one row changes in the batch which is incorrect as multiple rows could change.

The way to do this properly is to use the virtual inserted and deleted tables: http://msdn.microsoft.com/en-us/library/ms191300.aspx

How to use the unsigned Integer in Java 8 and Java 9?

Well, even in Java 8, long and int are still signed, only some methods treat them as if they were unsigned. If you want to write unsigned long literal like that, you can do

static long values = Long.parseUnsignedLong("18446744073709551615");

public static void main(String[] args) {
    System.out.println(values); // -1
    System.out.println(Long.toUnsignedString(values)); // 18446744073709551615
}

Unity 2d jumping script

Usually for jumping people use Rigidbody2D.AddForce with Forcemode.Impulse. It may seem like your object is pushed once in Y axis and it will fall down automatically due to gravity.

Example:

rigidbody2D.AddForce(new Vector2(0, 10), ForceMode2D.Impulse);

Why am I getting a "401 Unauthorized" error in Maven?

I had put a not encrypted password in the settings.xml .

I tested the call with curl

curl -u username:password http://url/artifactory/libs-snapshot-local/com/myproject/api/1.0-SNAPSHOT/api-1.0-20160128.114425-1.jar --request PUT --data target/api-1.0-SNAPSHOT.jar 

and I got the error:

{
  "errors" : [ {
    "status" : 401,
    "message" : "Artifactory configured to accept only encrypted passwords but received a clear text password."
  } ]
}

I retrieved my encrypted password clicking on my artifactory profile and unlocking it.

Custom Listview Adapter with filter Android

I hope it will be helpful for others.

// put below code (method) in Adapter class
public void filter(String charText) {
    charText = charText.toLowerCase(Locale.getDefault());
    myList.clear();
    if (charText.length() == 0) {
        myList.addAll(arraylist);
    }
    else
    {
        for (MyBean wp : arraylist) {
            if (wp.getName().toLowerCase(Locale.getDefault()).contains(charText)) {
                myList.add(wp);
            }
        }
    }
    notifyDataSetChanged();
}

declare below code in adapter class

private ArrayList<MyBean> myList;  // for loading main list
private ArrayList<MyBean> arraylist=null;  // for loading  filter data

below code in adapter Constructor

this.arraylist = new ArrayList<MyBean>();
    this.arraylist.addAll(myList);

and below code in your activity class

final EditText searchET = (EditText)findViewById(R.id.search_et);
    // Capture Text in EditText
    searchET.addTextChangedListener(new TextWatcher() {

        @Override
        public void afterTextChanged(Editable arg0) {
            // TODO Auto-generated method stub
            String text = searchET.getText().toString().toLowerCase(Locale.getDefault());
            adapter.filter(text);
        }

        @Override
        public void beforeTextChanged(CharSequence arg0, int arg1,
                                      int arg2, int arg3) {
            // TODO Auto-generated method stub
        }

        @Override
        public void onTextChanged(CharSequence arg0, int arg1, int arg2,
                                  int arg3) {
            // TODO Auto-generated method stub
        }
    });

Spring Data JPA find by embedded object property

This method name should do the trick:

Page<QueuedBook> findByBookIdRegion(Region region, Pageable pageable);

More info on that in the section about query derivation of the reference docs.

SQL Server: how to select records with specific date from datetime column

SELECT *
FROM LogRequests
WHERE cast(dateX as date) between '2014-05-09' and '2014-05-10';

This will select all the data between the 2 dates

CMake is not able to find BOOST libraries

I just want to point out that the FindBoost macro might be looking for an earlier version, for instance, 1.58.0 when you might have 1.60.0 installed. I suggest popping open the FindBoost macro from whatever it is you are attempting to build, and checking if that's the case. You can simply edit it to include your particular version. (This was my problem.)

BadImageFormatException. This will occur when running in 64 bit mode with the 32 bit Oracle client components installed

I had the same issue, then I fix it by change configuration manager x86 -> x64 and build

image

SQL Server - An expression of non-boolean type specified in a context where a condition is expected, near 'RETURN'

Your problem might be here:

OR
                        (
                            SELECT m.ResourceNo FROM JobMember m
                            JOIN JobTask t ON t.JobTaskNo = m.JobTaskNo
                            WHERE t.TaskManagerNo = @UserResourceNo
                            OR
                            t.AlternateTaskManagerNo = @UserResourceNo
                        )

try changing to

OR r.ResourceNo IN
                        (
                            SELECT m.ResourceNo FROM JobMember m
                            JOIN JobTask t ON t.JobTaskNo = m.JobTaskNo
                            WHERE t.TaskManagerNo = @UserResourceNo
                            OR
                            t.AlternateTaskManagerNo = @UserResourceNo
                        )

Sql error on update : The UPDATE statement conflicted with the FOREIGN KEY constraint

It sometimes happens when you try to Insert/Update an entity while the foreign key that you are trying to Insert/Update actually does not exist. So, be sure that the foreign key exists and try again.

A JNI error has occurred, please check your installation and try again in Eclipse x86 Windows 8.1

In Netbeans 8.0.2:

  1. Right click on your package.
  2. Select "Properties".
  3. Go to "Run" option.
  4. Select main class by browsing your class name.
  5. Click the "Ok" button.

ReferenceError: $ is not defined

Use Google CDN for fast loading:

<script type="text/javascript" src="//ajax.googleapis.com/ajax/libs/jquery/2.0.0/jquery.min.js"></script>

SQL Server stored procedure Nullable parameter

You can/should set your parameter to value to DBNull.Value;

if (variable == "")
{
     cmd.Parameters.Add("@Param", SqlDbType.VarChar, 500).Value = DBNull.Value;
}
else
{
     cmd.Parameters.Add("@Param", SqlDbType.VarChar, 500).Value = variable;
}

Or you can leave your server side set to null and not pass the param at all.

The ALTER TABLE statement conflicted with the FOREIGN KEY constraint

Try DELETE the current datas from tblDomare.PersNR . Because the values in tblDomare.PersNR didn't match with any of the values in tblBana.BanNR.

Facebook API "This app is in development mode"

I have the same problem while integrating the Facebook SDK for login.

I'm suggesting below approach for development mode > you can test all things if you are login with same account, which is used for 'developers.facebook.com' and if you want to use another accounts then you need to add Roles for that particular app, for that you can add developer or testers by using fid or facebook username.

Eg: - Select the particular app > Roles and then add developer or testers.

enter image description here

How to directly execute SQL query in C#?

Something like this should suffice, to do what your batch file was doing (dumping the result set as semi-colon delimited text to the console):

// sqlcmd.exe
// -S .\PDATA_SQLEXPRESS
// -U sa
// -P 2BeChanged!
// -d PDATA_SQLEXPRESS
// -s ; -W -w 100
// -Q "SELECT tPatCulIntPatIDPk, tPatSFirstname, tPatSName, tPatDBirthday  FROM  [dbo].[TPatientRaw] WHERE tPatSName = '%name%' "

DataTable dt            = new DataTable() ;
int       rows_returned ;

const string credentials = @"Server=(localdb)\.\PDATA_SQLEXPRESS;Database=PDATA_SQLEXPRESS;User ID=sa;Password=2BeChanged!;" ;
const string sqlQuery = @"
  select tPatCulIntPatIDPk ,
         tPatSFirstname    ,
         tPatSName         ,
         tPatDBirthday
  from dbo.TPatientRaw
  where tPatSName = @patientSurname
  " ;

using ( SqlConnection connection = new SqlConnection(credentials) )
using ( SqlCommand    cmd        = connection.CreateCommand() )
using ( SqlDataAdapter sda       = new SqlDataAdapter( cmd ) )
{
  cmd.CommandText = sqlQuery ;
  cmd.CommandType = CommandType.Text ;
  connection.Open() ;
  rows_returned = sda.Fill(dt) ;
  connection.Close() ;
}

if ( dt.Rows.Count == 0 )
{
  // query returned no rows
}
else
{

  //write semicolon-delimited header
  string[] columnNames = dt.Columns
                           .Cast<DataColumn>()
                           .Select( c => c.ColumnName )
                           .ToArray()
                           ;
  string   header      = string.Join("," , columnNames) ;
  Console.WriteLine(header) ;

  // write each row
  foreach ( DataRow dr in dt.Rows )
  {

    // get each rows columns as a string (casting null into the nil (empty) string
    string[] values = new string[dt.Columns.Count];
    for ( int i = 0 ; i < dt.Columns.Count ; ++i )
    {
      values[i] = ((string) dr[i]) ?? "" ; // we'll treat nulls as the nil string for the nonce
    }

    // construct the string to be dumped, quoting each value and doubling any embedded quotes.
    string data = string.Join( ";" , values.Select( s => "\""+s.Replace("\"","\"\"")+"\"") ) ;
    Console.WriteLine(values);

  }

}

Test credit card numbers for use with PayPal sandbox

It turns out, after messing around with all of the settings in the test business account, that one (or more) of the fraud related settings in the payment receiving preferences / security settings screens were causing the test payments to fail (without any useful error).

How do I get AWS_ACCESS_KEY_ID for Amazon?

  1. Open the AWS Console
  2. Click on your username near the top right and select My Security Credentials
  3. Click on Users in the sidebar
  4. Click on your username
  5. Click on the Security Credentials tab
  6. Click Create Access Key
  7. Click Show User Security Credentials

Error: "The sandbox is not in sync with the Podfile.lock..." after installing RestKit with cocoapods

This can also happen if you use Windows for the Git checkout, and the Podfile.lock is created with windows (CRLF) line endings.

How to execute function in SQL Server 2008

It looks like there's something else called Afisho_rankimin in your DB so the function is not being created. Try calling your function something else. E.g.

CREATE FUNCTION dbo.Afisho_rankimin1(@emri_rest int)
RETURNS int
AS
   BEGIN
       Declare @rankimi int
       Select @rankimi=dbo.RESTORANTET.Rankimi
       From RESTORANTET
       Where  dbo.RESTORANTET.ID_Rest=@emri_rest
       RETURN @rankimi
  END
  GO

Note that you need to call this only once, not every time you call the function. After that try calling

SELECT dbo.Afisho_rankimin1(5) AS Rankimi 

how to call scalar function in sql server 2008

For some reason I was not able to use my scalar function until I referenced it using brackets, like so:

select [dbo].[fun_functional_score]('01091400003')

SQL Server Insert if not exists

Depending on your version (2012?) of SQL Server aside from the IF EXISTS you can also use MERGE like so:

ALTER PROCEDURE [dbo].[EmailsRecebidosInsert]
    ( @_DE nvarchar(50)
    , @_ASSUNTO nvarchar(50)
    , @_DATA nvarchar(30))
AS BEGIN
    MERGE [dbo].[EmailsRecebidos] [Target]
    USING (VALUES (@_DE, @_ASSUNTO, @_DATA)) [Source]([De], [Assunto], [Data])
         ON [Target].[De] = [Source].[De] AND [Target].[Assunto] = [Source].[Assunto] AND [Target].[Data] = [Source].[Data]
     WHEN NOT MATCHED THEN
        INSERT ([De], [Assunto], [Data])
        VALUES ([Source].[De], [Source].[Assunto], [Source].[Data]);
END

using stored procedure in entity framework

You can call a stored procedure using SqlQuery (See here)

// Prepare the query
var query = context.Functions.SqlQuery(
    "EXEC [dbo].[GetFunctionByID] @p1", 
    new SqlParameter("p1", 200));

// add NoTracking() if required

// Fetch the results
var result = query.ToList();

Entity Framework code-first: migration fails with update-database, forces unneccessary(?) add-migration

I also tried deleting the database again, called update-database and then add-migration. I ended up with an additional migration that seems not to change anything (see below)

Based on above details, I think you have done last thing first. If you run Update database before Add-migration, it won't update the database with your migration schemas. First you need to add the migration and then run update command.

Try them in this order using package manager console.

PM> Enable-migrations //You don't need this as you have already done it
PM> Add-migration Give_it_a_name
PM> Update-database

Conversion failed when converting from a character string to uniqueidentifier - Two GUIDs

DECLARE @t TABLE (ID UNIQUEIDENTIFIER DEFAULT NEWID(),myid UNIQUEIDENTIFIER
                , friendid UNIQUEIDENTIFIER, time1 Datetime, time2 Datetime)
insert into @t (myid,friendid,time1,time2) 
values
 ( CONVERT(uniqueidentifier,'0C6A36BA-10E4-438F-BA86-0D5B68A2BB15'),
   CONVERT(uniqueidentifier,'DF215E10-8BD4-4401-B2DC-99BB03135F2E'),
   '2014-01-05 02:04:41.953','2014-01-05 12:04:41.953')

SELECT * FROM @t

Result Set With out any errors

+------------------------------------------------------------------------------------------------------------------------------------------------------------------------+
¦                  ID                  ¦                 myid                 ¦               friendid               ¦          time1          ¦          time2          ¦
¦--------------------------------------+--------------------------------------+--------------------------------------+-------------------------+-------------------------¦
¦ CF628202-33F3-49CF-8828-CB2D93C69675 ¦ 0C6A36BA-10E4-438F-BA86-0D5B68A2BB15 ¦ DF215E10-8BD4-4401-B2DC-99BB03135F2E ¦ 2014-01-05 02:04:41.953 ¦ 2014-01-05 12:04:41.953 ¦
+------------------------------------------------------------------------------------------------------------------------------------------------------------------------+

How to call Stored Procedure in Entity Framework 6 (Code-First)?

When EDMX create this time if you select stored procedured in table select option then just call store procedured using procedured name...

var num1 = 1; 
var num2 = 2; 

var result = context.proc_name(num1,num2).tolist();// list or single you get here.. using same thing you can call insert,update or delete procedured.

ImportError: No module named dateutil.parser

For Python 3:

pip3 install python-dateutil

Update records using LINQ

You have two options as far as I know:

  1. Perform your query, iterate over it to modify the entities, then call SaveChanges().
  2. Execute a SQL command like you mentioned at the top of your question. To see how to do this, take a look at this page.

If you use option 2, you're losing some of the abstraction that the Entity Framework gives you, but if you need to perform a very large update, this might be the best choice for performance reasons.

SQL Server loop - how do I loop through a set of records

By using T-SQL and cursors like this :

DECLARE @MyCursor CURSOR;
DECLARE @MyField YourFieldDataType;
BEGIN
    SET @MyCursor = CURSOR FOR
    select top 1000 YourField from dbo.table
        where StatusID = 7      

    OPEN @MyCursor 
    FETCH NEXT FROM @MyCursor 
    INTO @MyField

    WHILE @@FETCH_STATUS = 0
    BEGIN
      /*
         YOUR ALGORITHM GOES HERE   
      */
      FETCH NEXT FROM @MyCursor 
      INTO @MyField 
    END; 

    CLOSE @MyCursor ;
    DEALLOCATE @MyCursor;
END;

Multiple separate IF conditions in SQL Server

To avoid syntax errors, be sure to always put BEGIN and END after an IF clause, eg:

IF (@A!= @SA)
   BEGIN
   --do stuff
   END
IF (@C!= @SC)
   BEGIN
   --do stuff
   END

... and so on. This should work as expected. Imagine BEGIN and END keyword as the opening and closing bracket, respectively.

insert/delete/update trigger in SQL server

Not possible, per MSDN:

You can have the same code execute for multiple trigger types, but the syntax does not allow for multiple code blocks in one trigger:

Trigger on an INSERT, UPDATE, or DELETE statement to a table or view (DML Trigger)

CREATE TRIGGER [ schema_name . ]trigger_name 
ON { table | view } 
[ WITH <dml_trigger_option> [ ,...n ] ]
{ FOR | AFTER | INSTEAD OF } 
{ [ INSERT ] [ , ] [ UPDATE ] [ , ] [ DELETE ] } 
[ NOT FOR REPLICATION ] 
AS { sql_statement  [ ; ] [ ,...n ] | EXTERNAL NAME <method specifier [ ; ] > }

EntityType has no key defined error

Additionally Remember, Don't forget to add public keyword like this

[Key] 
int RoleId { get; set; } //wrong method

you must use Public keyword like this

[Key] 
public int RoleId { get; set; } //correct method

SQL Insert Query Using C#

The most common mistake (especially when using express) to the "my insert didn't happen" is : looking in the wrong file.

If you are using file-based express (rather than strongly attached), then the file in your project folder (say, c:\dev\myproject\mydb.mbd) is not the file that is used in your program. When you build, that file is copied - for example to c:\dev\myproject\bin\debug\mydb.mbd; your program executes in the context of c:\dev\myproject\bin\debug\, and so it is here that you need to look to see if the edit actually happened. To check for sure: query for the data inside the application (after inserting it).

Log record changes in SQL server in an audit table

I know this is old, but maybe this will help someone else.

Do not log "new" values. Your existing table, GUESTS, has the new values. You'll have double entry of data, plus your DB size will grow way too fast that way.

I cleaned this up and minimized it for this example, but here is the tables you'd need for logging off changes:

CREATE TABLE GUESTS (
      GuestID INT IDENTITY(1,1) PRIMARY KEY, 
      GuestName VARCHAR(50), 
      ModifiedBy INT, 
      ModifiedOn DATETIME
)

CREATE TABLE GUESTS_LOG (
      GuestLogID INT IDENTITY(1,1) PRIMARY KEY, 
      GuestID INT, 
      GuestName VARCHAR(50), 
      ModifiedBy INT, 
      ModifiedOn DATETIME
)

When a value changes in the GUESTS table (ex: Guest name), simply log off that entire row of data, as-is, to your Log/Audit table using the Trigger. Your GUESTS table has current data, the Log/Audit table has the old data.

Then use a select statement to get data from both tables:

SELECT 0 AS 'GuestLogID', GuestID, GuestName, ModifiedBy, ModifiedOn FROM [GUESTS] WHERE GuestID = 1
UNION
SELECT GuestLogID, GuestID, GuestName, ModifiedBy, ModifiedOn FROM [GUESTS_LOG] WHERE GuestID = 1
ORDER BY ModifiedOn ASC

Your data will come out with what the table looked like, from Oldest to Newest, with the first row being what was created & the last row being the current data. You can see exactly what changed, who changed it, and when they changed it.

Optionally, I used to have a function that looped through the RecordSet (in Classic ASP), and only displayed what values had changed on the web page. It made for a GREAT audit trail so that users could see what had changed over time.

Stored procedure or function expects parameter which is not supplied

in my case, I was passing all the parameters but one of the parameter my code was passing a null value for string.

Eg: cmd.Parameters.AddWithValue("@userName", userName);

in the above case, if the data type of userName is string, I was passing userName as null.

How to use both onclick and target="_blank"

Instead use window.open():

The syntax is:

window.open(strUrl, strWindowName[, strWindowFeatures]);

Your code should have:

window.open('Prosjektplan.pdf');

Your code should be:

<p class="downloadBoks"
   onclick="window.open('Prosjektplan.pdf')">Prosjektbeskrivelse</p>

LEFT JOIN in LINQ to entities?

You can use this not only in entities but also store procedure or other data source:

var customer = (from cus in _billingCommonservice.BillingUnit.CustomerRepository.GetAll()  
                          join man in _billingCommonservice.BillingUnit.FunctionRepository.ManagersCustomerValue()  
                          on cus.CustomerID equals man.CustomerID  
                          // start left join  
                          into a  
                          from b in a.DefaultIfEmpty(new DJBL_uspGetAllManagerCustomer_Result() )  
                          select new { cus.MobileNo1,b.ActiveStatus });  

The SELECT permission was denied on the object 'Users', database 'XXX', schema 'dbo'

May be your Plesk panel or other panel subscription has been expired....please check subscription End.

HTTP Status 500 - org.apache.jasper.JasperException: java.lang.NullPointerException

NullPointerException with JSP can also happen if:

A getter returns a non-public inner class.

This code will fail if you remove Getters's access modifier or make it private or protected.

JAVA:

package com.myPackage;
public class MyClass{ 
    //: Must be public or you will get:
    //: org.apache.jasper.JasperException: 
    //: java.lang.NullPointerException
    public class Getters{
        public String 
        myProperty(){ return(my_property); }
    };;

    //: JSP EL can only access functions:
    private Getters _get;
    public  Getters  get(){ return _get; }

    private String 
    my_property;

    public MyClass(String my_property){
        super();
        this.my_property    = my_property;
        _get = new Getters();
    };;
};;

JSP

<%@ taglib uri   ="http://java.sun.com/jsp/jstl/core" prefix="c" %>
<%@ page import="com.myPackage.MyClass" %>
<%
    MyClass inst = new MyClass("[PROP_VALUE]");
    pageContext.setAttribute("my_inst", inst ); 
%><html lang="en"><body>
    ${ my_inst.get().myProperty() }
</body></html>

Default values and initialization in Java

Yes, an instance variable will be initialized to a default value. For a local variable, you need to initialize before use:

public class Main {

    int instaceVariable; // An instance variable will be initialized to the default value

    public static void main(String[] args) {
        int localVariable = 0; // A local variable needs to be initialized before use
    }
}

Given URL is not allowed by the Application configuration Facebook application error

Under Basic settings:

  1. Add the platform - Mine was web.
  2. Supply the site URL - mind the http or https.
  3. You can also supply the mobile site URL if you have any - remember to mind the http or https here as well.
  4. Save the changes.

Then hit the advanced tab and scroll down to locate Valid OAuth redirect URIs its right below Client Token.

  1. Supply the redirection URL - The URL to redirect to after the login.
  2. Save the changes.

Then get back to your website or web page and refresh.

How do I create a new user in a SQL Azure database?

Edit - Contained User (v12 and later)

As of Sql Azure 12, databases will be created as Contained Databases which will allow users to be created directly in your database, without the need for a server login via master.

CREATE USER [MyUser] WITH PASSWORD = 'Secret';
ALTER ROLE [db_datareader] ADD MEMBER [MyUser];

Note when connecting to the database when using a contained user that you must always specify the database in the connection string.

Connecting via a contained User

Traditional Server Login - Database User (Pre v 12)

Just to add to @Igorek's answer, you can do the following in Sql Server Management Studio:

Create the new Login on the server
In master (via the Available databases drop down in SSMS - this is because USE master doesn't work in Azure):

enter image description here

create the login:

CREATE LOGIN username WITH password='password';

Create the new User in the database
Switch to the actual database (again via the available databases drop down, or a new connection)

CREATE USER username FROM LOGIN username;

(I've assumed that you want the user and logins to tie up as username, but change if this isn't the case.)

Now add the user to the relevant security roles

EXEC sp_addrolemember N'db_owner', N'username'
GO

(Obviously an app user should have less privileges than dbo.)

How to get the list of all database users

SELECT name FROM sys.database_principals WHERE
type_desc = 'SQL_USER' AND default_schema_name = 'dbo'

This selects all the users in the SQL server that the administrator created!

where to place CASE WHEN column IS NULL in this query

Try this:

CASE WHEN table3.col3 IS NULL THEN table2.col3 ELSE table3.col3 END as col4

The as col4 should go at the end of the CASE the statement. Also note that you're missing the END too.

Another probably more simple option would be:

IIf([table3.col3] Is Null,[table2.col3],[table3.col3])

Just to clarify, MS Access does not support COALESCE. If it would that would be the best way to go.

Edit after radical question change:

To turn the query into SQL Server then you can use COALESCE (so it was technically answered before too):

SELECT dbo.AdminID.CountryID, dbo.AdminID.CountryName, dbo.AdminID.RegionID, 
dbo.AdminID.[Region name], dbo.AdminID.DistrictID, dbo.AdminID.DistrictName,
dbo.AdminID.ADMIN3_ID, dbo.AdminID.ADMIN3,
COALESCE(dbo.EU_Admin3.EUID, dbo.EU_Admin2.EUID)
FROM dbo.AdminID

BTW, your CASE statement was missing a , before the field. That's why it didn't work.

Conditional WHERE clause in SQL Server

To answer the underlying question of how to use a CASE expression in the WHERE clause:

First remember that the value of a CASE expression has to have a normal data type value, not a boolean value. It has to be a varchar, or an int, or something. It's the same reason you can't say SELECT Name, 76 = Age FROM [...] and expect to get 'Frank', FALSE in the result set.

Additionally, all expressions in a WHERE clause need to have a boolean value. They can't have a value of a varchar or an int. You can't say WHERE Name; or WHERE 'Frank';. You have to use a comparison operator to make it a boolean expression, so WHERE Name = 'Frank';

That means that the CASE expression must be on one side of a boolean expression. You have to compare the CASE expression to something. It can't stand by itself!

Here:

WHERE 
    DateDropped = 0
    AND CASE
            WHEN @JobsOnHold  = 1 AND DateAppr >= 0 THEN 'True'
            WHEN DateAppr != 0 THEN 'True'
            ELSE 'False'
        END = 'True'

Notice how in the end the CASE expression on the left will turn the boolean expression into either 'True' = 'True' or 'False' = 'True'.

Note that there's nothing special about 'False' and 'True'. You can use 0 and 1 if you'd rather, too.

You can typically rewrite the CASE expression into boolean expressions we're more familiar with, and that's generally better for performance. However, sometimes is easier or more maintainable to use an existing expression than it is to convert the logic.

Ways to iterate over a list in Java

Right, many alternatives are listed. The easiest and cleanest would be just using the enhanced for statement as below. The Expression is of some type that is iterable.

for ( FormalParameter : Expression ) Statement

For example, to iterate through, List<String> ids, we can simply so,

for (String str : ids) {
    // Do something
}

how to resolve DTS_E_OLEDBERROR. in ssis

Solution for this issue is:

  1. Create another connection manager for your excel or flat files else you just have to pass variable values in connection string:

  2. Right Click on Connection Manager>>properties>>Expression >>Select "ConnectionString" from drop down and pass the input variable like path , filename ..

Regarding Java switch statements - using return and omitting breaks in each case

Though the question is old enough it still can be referenced nowdays.

Semantically that is exactly what Java 12 introduced (https://openjdk.java.net/jeps/325), thus, exactly in that simple example provided I can't see any problem or cons.

Non-invocable member cannot be used like a method?

I had the same issue and realized that removing the parentheses worked. Sometimes having someone else read your code can be useful if you have been the only one working on it for some time.

E.g.

  cmd.CommandType = CommandType.Text(); 

Replace: cmd.CommandType = CommandType.Text;

Insert Multiple Rows Into Temp Table With SQL Server 2012

Yes, SQL Server 2012 supports multiple inserts - that feature was introduced in SQL Server 2008.

That makes me wonder if you have Management Studio 2012, but you're really connected to a SQL Server 2005 instance ...

What version of the SQL Server engine do you get from SELECT @@VERSION ??

How to make a input field readonly with JavaScript?

document.getElementById("").readOnly = true

SQL Query - SUM(CASE WHEN x THEN 1 ELSE 0) for multiple columns

I would change the query in the following ways:

  1. Do the aggregation in subqueries. This can take advantage of more information about the table for optimizing the group by.
  2. Combine the second and third subqueries. They are aggregating on the same column. This requires using a left outer join to ensure that all data is available.
  3. By using count(<fieldname>) you can eliminate the comparisons to is null. This is important for the second and third calculated values.
  4. To combine the second and third queries, it needs to count an id from the mde table. These use mde.mdeid.

The following version follows your example by using union all:

SELECT CAST(Detail.ReceiptDate AS DATE) AS "Date",
       SUM(TOTALMAILED) as TotalMailed,
       SUM(TOTALUNDELINOTICESRECEIVED) as TOTALUNDELINOTICESRECEIVED,
       SUM(TRACEUNDELNOTICESRECEIVED) as TRACEUNDELNOTICESRECEIVED
FROM ((select SentDate AS "ReceiptDate", COUNT(*) as TotalMailed,
              NULL as TOTALUNDELINOTICESRECEIVED, NULL as TRACEUNDELNOTICESRECEIVED
       from MailDataExtract
       where SentDate is not null
       group by SentDate
      ) union all
      (select MDE.ReturnMailDate AS ReceiptDate, 0,
              COUNT(distinct mde.mdeid) as TOTALUNDELINOTICESRECEIVED,
              SUM(case when sd.ReturnMailTypeId = 1 then 1 else 0 end) as TRACEUNDELNOTICESRECEIVED
       from MailDataExtract MDE left outer join
            DTSharedData.dbo.ScanData SD
            ON SD.ScanDataID = MDE.ReturnScanDataID
       group by MDE.ReturnMailDate;
      )
     ) detail
GROUP BY CAST(Detail.ReceiptDate AS DATE)
ORDER BY 1;

The following does something similar using full outer join:

SELECT coalesce(sd.ReceiptDate, mde.ReceiptDate) AS "Date",
       sd.TotalMailed, mde.TOTALUNDELINOTICESRECEIVED,
       mde.TRACEUNDELNOTICESRECEIVED
FROM (select cast(SentDate as date) AS "ReceiptDate", COUNT(*) as TotalMailed
      from MailDataExtract
      where SentDate is not null
      group by cast(SentDate as date)
     ) sd full outer join
    (select cast(MDE.ReturnMailDate as date) AS ReceiptDate,
            COUNT(distinct mde.mdeID) as TOTALUNDELINOTICESRECEIVED,
            SUM(case when sd.ReturnMailTypeId = 1 then 1 else 0 end) as TRACEUNDELNOTICESRECEIVED
     from MailDataExtract MDE left outer join
          DTSharedData.dbo.ScanData SD
          ON SD.ScanDataID = MDE.ReturnScanDataID
     group by cast(MDE.ReturnMailDate as date)
    ) mde
    on sd.ReceiptDate = mde.ReceiptDate
ORDER BY 1;

Conversion failed when converting the varchar value to data type int in sql

Your problem seams to be located here:

SELECT @maxCode = CAST(MAX(CAST(SUBSTRING(Voucher_No,LEN(@startFrom)+1,LEN(Voucher_No)- LEN(@Prefix)) AS INT)) AS varchar(100)) FROM dbo.Journal_Entry;
SET @sCode=CAST(@maxCode AS INT)

As the error says, you're casting a string that contains a letter 'J' to an INT which for obvious reasons is not possible.

Either fix SUBSTRING or don't store the letter 'J' in the database and only prepend it when reading.

Showing/Hiding Table Rows with Javascript - can do with ID - how to do with Class?

document.getElementsByClassName returns a NodeList, not a single element, I'd recommend either using jQuery, since you'd only have to use something like $('.new').toggle()

or if you want plain JS try :

function toggle_by_class(cls, on) {
    var lst = document.getElementsByClassName(cls);
    for(var i = 0; i < lst.length; ++i) {
        lst[i].style.display = on ? '' : 'none';
    }
}

How can I find out what FOREIGN KEY constraint references a table in SQL Server?

In SQL Server Management Studio you can just right click the table in the object explorer and select "View Dependencies". This would give a you a good starting point. It shows tables, views, and procedures that reference the table.

Entity Framework (EF) Code First Cascade Delete for One-to-Zero-or-One relationship

This code worked for me

protected override void OnModelCreating(DbModelBuilder modelBuilder)
    {
        modelBuilder.Entity<UserDetail>()
            .HasRequired(d => d.User)
            .WithOptional(u => u.UserDetail)
            .WillCascadeOnDelete(true);
    }

The migration code was:

public override void Up()
    {
        AddForeignKey("UserDetail", "UserId", "User", "UserId", cascadeDelete: true);
    }

And it worked fine. When I first used

modelBuilder.Entity<User>()
    .HasOptional(a => a.UserDetail)
    .WithOptionalDependent()
    .WillCascadeOnDelete(true);

The migration code was:

AddForeignKey("User", "UserDetail_UserId", "UserDetail", "UserId", cascadeDelete: true); 

but it does not match any of the two overloads available (in EntityFramework 6)

UPDATE and REPLACE part of a string

You have one table where you have date Code which is seven character something like

"32-1000"

Now you want to replace all

"32-"

With

"14-"

The SQL query you have to run is

Update Products Set Code = replace(Code, '32-', '14-') Where ...(Put your where statement in here)

Parse JSON response using jQuery

Give this a try:

success: function(json) {
   console.log(JSON.stringify(json.topics));
   $.each(json.topics, function(idx, topic){
     $("#nav").html('<a href="' + topic.link_src + '">' + topic.link_text + "</a>");
   });
},

What is the printf format specifier for bool?

If you like C++ better than C, you can try this:

#include <ios>
#include <iostream>

bool b = IsSomethingTrue();
std::cout << std::boolalpha << b;

Procedure or function !!! has too many arguments specified

Use the following command before defining them:

cmd.Parameters.Clear()

How to generate and manually insert a uniqueidentifier in sql server?

Kindly check Column ApplicationId datatype in Table aspnet_Users , ApplicationId column datatype should be uniqueidentifier .

*Your parameter order is passed wrongly , Parameter @id should be passed as first argument, but in your script it is placed in second argument..*

So error is raised..

Please refere sample script:

DECLARE @id uniqueidentifier
SET @id = NEWID()
Create Table #temp1(AppId uniqueidentifier)

insert into #temp1 values(@id)

Select * from #temp1

Drop Table #temp1

NameError: global name 'xrange' is not defined in Python 3

I solved the issue by adding this import
More info

from past.builtins import xrange

Introducing FOREIGN KEY constraint may cause cycles or multiple cascade paths - why?

None of the aforementioned solutions worked for me. What I had to do was use a nullable int (int?) on the foreign key that was not required (or not a not null column key) and then delete some of my migrations.

Start by deleting the migrations, then try the nullable int.

Problem was both a modification and model design. No code change was necessary.

SQL Server 2008 Row Insert and Update timestamps

try

CREATE TABLE [dbo].[Names]
(
    [Name] [nvarchar](64) NOT NULL,
    [CreateTS] [smalldatetime] NOT NULL CONSTRAINT CreateTS_DF DEFAULT CURRENT_TIMESTAMP,
    [UpdateTS] [smalldatetime] NOT NULL

)

PS I think a smalldatetime is good enough. You may decide differently.

Can you not do this at the "moment of impact" ?

In Sql Server, this is common:

Update dbo.MyTable 
Set 

ColA = @SomeValue , 
UpdateDS = CURRENT_TIMESTAMP
Where...........

Sql Server has a "timestamp" datatype.

But it may not be what you think.

Here is a reference:

http://msdn.microsoft.com/en-us/library/ms182776(v=sql.90).aspx

Here is a little RowVersion (synonym for timestamp) example:

CREATE TABLE [dbo].[Names]
(
    [Name] [nvarchar](64) NOT NULL,
    RowVers rowversion ,
    [CreateTS] [datetime] NOT NULL CONSTRAINT CreateTS_DF DEFAULT CURRENT_TIMESTAMP,
    [UpdateTS] [datetime] NOT NULL

)


INSERT INTO dbo.Names (Name,UpdateTS)
select 'John' , CURRENT_TIMESTAMP
UNION ALL select 'Mary' , CURRENT_TIMESTAMP
UNION ALL select 'Paul' , CURRENT_TIMESTAMP

select *  ,  ConvertedRowVers = CONVERT(bigint,RowVers) from [dbo].[Names]

Update dbo.Names Set Name = Name

select *  ,  ConvertedRowVers = CONVERT(bigint,RowVers) from [dbo].[Names]

Maybe a complete working example:

DROP TABLE [dbo].[Names]
GO


CREATE TABLE [dbo].[Names]
(
    [Name] [nvarchar](64) NOT NULL,
    RowVers rowversion ,
    [CreateTS] [datetime] NOT NULL CONSTRAINT CreateTS_DF DEFAULT CURRENT_TIMESTAMP,
    [UpdateTS] [datetime] NOT NULL

)

GO

CREATE TRIGGER dbo.trgKeepUpdateDateInSync_ByeByeBye ON dbo.Names
AFTER INSERT, UPDATE
AS

BEGIN

Update dbo.Names Set UpdateTS = CURRENT_TIMESTAMP from dbo.Names myAlias , inserted triggerInsertedTable where 
triggerInsertedTable.Name = myAlias.Name

END


GO






INSERT INTO dbo.Names (Name,UpdateTS)
select 'John' , CURRENT_TIMESTAMP
UNION ALL select 'Mary' , CURRENT_TIMESTAMP
UNION ALL select 'Paul' , CURRENT_TIMESTAMP

select *  ,  ConvertedRowVers = CONVERT(bigint,RowVers) from [dbo].[Names]

Update dbo.Names Set Name = Name , UpdateTS = '03/03/2003' /* notice that even though I set it to 2003, the trigger takes over */

select *  ,  ConvertedRowVers = CONVERT(bigint,RowVers) from [dbo].[Names]

Matching on the "Name" value is probably not wise.

Try this more mainstream example with a SurrogateKey

DROP TABLE [dbo].[Names]
GO


CREATE TABLE [dbo].[Names]
(
    SurrogateKey int not null Primary Key Identity (1001,1),
    [Name] [nvarchar](64) NOT NULL,
    RowVers rowversion ,
    [CreateTS] [datetime] NOT NULL CONSTRAINT CreateTS_DF DEFAULT CURRENT_TIMESTAMP,
    [UpdateTS] [datetime] NOT NULL

)

GO

CREATE TRIGGER dbo.trgKeepUpdateDateInSync_ByeByeBye ON dbo.Names
AFTER UPDATE
AS

BEGIN

   UPDATE dbo.Names
    SET UpdateTS = CURRENT_TIMESTAMP
    From  dbo.Names myAlias
    WHERE exists ( select null from inserted triggerInsertedTable where myAlias.SurrogateKey = triggerInsertedTable.SurrogateKey)

END


GO






INSERT INTO dbo.Names (Name,UpdateTS)
select 'John' , CURRENT_TIMESTAMP
UNION ALL select 'Mary' , CURRENT_TIMESTAMP
UNION ALL select 'Paul' , CURRENT_TIMESTAMP

select *  ,  ConvertedRowVers = CONVERT(bigint,RowVers) from [dbo].[Names]

Update dbo.Names Set Name = Name , UpdateTS = '03/03/2003' /* notice that even though I set it to 2003, the trigger takes over */

select *  ,  ConvertedRowVers = CONVERT(bigint,RowVers) from [dbo].[Names]

CocoaPods Errors on Project Build

update: a podfile.lock is necessary and should not be ignored by version control, it keeps track of the versions of libraries installed at a certain pod install. (It's similar to gemfile.lock and composer.lock for rails and php dependency management, respectively). To learn more please read the docs. Credit goes to cbowns.


In my case, what I did was that I was doing some house cleaning for my project (ie branching out the integration tests as a git submodule.. removing duplicate files etc).. and pushed the final result to a git remote repo.. all the clients who cloned my repo suffered from the above error. Inspired by Hlung's comment above, I realized that there were some dangling pod scripts that were attempting to run against some non-existent files. So I went to my target build phase, and deleted all the remaining phases that had anything to do with cocoa pods (and Hlung's comment he suggests deleting Copy Pods Manifest.lock and copy pod resources.. mine were named different maybe b/c I'm using Xcode 5.. the point being is to delete those dangling build phases)..

Fill DataTable from SQL Server database

Try with following:

public DataTable fillDataTable(string table)
    {
        string query = "SELECT * FROM dstut.dbo." +table;

        SqlConnection sqlConn = new SqlConnection(conSTR);
        sqlConn.Open();
        SqlCommand cmd = new SqlCommand(query, sqlConn);
        SqlDataAdapter da=new SqlDataAdapter(cmd);
        DataTable dt = new DataTable();
        da.Fill(dt);
        sqlConn.Close();
        return dt;
    }

Hope it is helpful.

creating triggers for After Insert, After Update and After Delete in SQL

(Update: overlooked a fault in the matter, I have corrected)

(Update2: I wrote from memory the code screwed up, repaired it)

(Update3: check on SQLFiddle)

create table Derived_Values
  (
    BusinessUnit nvarchar(100) not null
    ,Questions nvarchar(100) not null
    ,Answer nvarchar(100)
    )

go

ALTER TABLE Derived_Values ADD CONSTRAINT PK_Derived_Values
PRIMARY KEY CLUSTERED (BusinessUnit, Questions);

create table Derived_Values_Test
  (
    BusinessUnit nvarchar(150)
    ,Questions nvarchar(100)
    ,Answer nvarchar(100)
    )

go

CREATE TRIGGER trgAfterUpdate ON  [Derived_Values]
FOR UPDATE
AS  
begin
    declare @BusinessUnit nvarchar(50)
    set @BusinessUnit = 'Updated Record -- After Update Trigger.'

    insert into 
        [Derived_Values_Test]
        --(BusinessUnit,Questions, Answer) 
    SELECT 
        @BusinessUnit + i.BusinessUnit, i.Questions, i.Answer
    FROM 
        inserted i
        inner join deleted d on i.BusinessUnit = d.BusinessUnit
end

go

CREATE TRIGGER trgAfterDelete ON  [Derived_Values]
FOR UPDATE
AS  
begin
    declare @BusinessUnit nvarchar(50)
    set @BusinessUnit = 'Deleted Record -- After Delete Trigger.'

    insert into 
        [Derived_Values_Test]
        --(BusinessUnit,Questions, Answer) 
    SELECT 
        @BusinessUnit + d.BusinessUnit, d.Questions, d.Answer
    FROM 
        deleted d
end

go

insert Derived_Values (BusinessUnit,Questions, Answer) values ('BU1', 'Q11', 'A11')
insert Derived_Values (BusinessUnit,Questions, Answer) values ('BU1', 'Q12', 'A12')
insert Derived_Values (BusinessUnit,Questions, Answer) values ('BU2', 'Q21', 'A21')
insert Derived_Values (BusinessUnit,Questions, Answer) values ('BU2', 'Q22', 'A22')

UPDATE Derived_Values SET Answer='Updated Answers A11' from Derived_Values WHERE (BusinessUnit = 'BU1') AND (Questions = 'Q11');
UPDATE Derived_Values SET Answer='Updated Answers A12' from Derived_Values WHERE (BusinessUnit = 'BU1') AND (Questions = 'Q12');
UPDATE Derived_Values SET Answer='Updated Answers A21' from Derived_Values WHERE (BusinessUnit = 'BU2') AND (Questions = 'Q21');
UPDATE Derived_Values SET Answer='Updated Answers A22' from Derived_Values WHERE (BusinessUnit = 'BU2') AND (Questions = 'Q22');

delete Derived_Values;

and then:

SELECT * FROM Derived_Values;
go

select * from Derived_Values_Test;


Record Count: 0;

BUSINESSUNIT    QUESTIONS   ANSWER
Updated Record -- After Update Trigger.BU1  Q11 Updated Answers A11
Deleted Record -- After Delete Trigger.BU1  Q11 A11
Updated Record -- After Update Trigger.BU1  Q12 Updated Answers A12
Deleted Record -- After Delete Trigger.BU1  Q12 A12
Updated Record -- After Update Trigger.BU2  Q21 Updated Answers A21
Deleted Record -- After Delete Trigger.BU2  Q21 A21
Updated Record -- After Update Trigger.BU2  Q22 Updated Answers A22
Deleted Record -- After Delete Trigger.BU2  Q22 A22

(Update4: If you want to sync: SQLFiddle)

create table Derived_Values
  (
    BusinessUnit nvarchar(100) not null
    ,Questions nvarchar(100) not null
    ,Answer nvarchar(100)
    )

go

ALTER TABLE Derived_Values ADD CONSTRAINT PK_Derived_Values
PRIMARY KEY CLUSTERED (BusinessUnit, Questions);

create table Derived_Values_Test
  (
    BusinessUnit nvarchar(150) not null
    ,Questions nvarchar(100) not null
    ,Answer nvarchar(100)
    )

go

ALTER TABLE Derived_Values_Test ADD CONSTRAINT PK_Derived_Values_Test
PRIMARY KEY CLUSTERED (BusinessUnit, Questions);

CREATE TRIGGER trgAfterInsert ON  [Derived_Values]
FOR INSERT
AS  
begin
    insert
        [Derived_Values_Test]
        (BusinessUnit,Questions,Answer)
    SELECT 
        i.BusinessUnit, i.Questions, i.Answer
    FROM 
        inserted i
end

go


CREATE TRIGGER trgAfterUpdate ON  [Derived_Values]
FOR UPDATE
AS  
begin
    declare @BusinessUnit nvarchar(50)
    set @BusinessUnit = 'Updated Record -- After Update Trigger.'

    update
        [Derived_Values_Test]
    set
        --BusinessUnit = i.BusinessUnit
        --,Questions = i.Questions
        Answer = i.Answer
    from
        [Derived_Values]
        inner join inserted i 
    on
        [Derived_Values].BusinessUnit = i.BusinessUnit
        and
        [Derived_Values].Questions = i.Questions
end

go

CREATE TRIGGER trgAfterDelete ON  [Derived_Values]
FOR DELETE
AS  
begin
    delete 
        [Derived_Values_Test]
    from
        [Derived_Values_Test]
        inner join deleted d 
    on
        [Derived_Values_Test].BusinessUnit = d.BusinessUnit
        and
        [Derived_Values_Test].Questions = d.Questions
end

go

insert Derived_Values (BusinessUnit,Questions, Answer) values ('BU1', 'Q11', 'A11')
insert Derived_Values (BusinessUnit,Questions, Answer) values ('BU1', 'Q12', 'A12')
insert Derived_Values (BusinessUnit,Questions, Answer) values ('BU2', 'Q21', 'A21')
insert Derived_Values (BusinessUnit,Questions, Answer) values ('BU2', 'Q22', 'A22')

UPDATE Derived_Values SET Answer='Updated Answers A11' from Derived_Values WHERE (BusinessUnit = 'BU1') AND (Questions = 'Q11');
UPDATE Derived_Values SET Answer='Updated Answers A12' from Derived_Values WHERE (BusinessUnit = 'BU1') AND (Questions = 'Q12');
UPDATE Derived_Values SET Answer='Updated Answers A21' from Derived_Values WHERE (BusinessUnit = 'BU2') AND (Questions = 'Q21');
UPDATE Derived_Values SET Answer='Updated Answers A22' from Derived_Values WHERE (BusinessUnit = 'BU2') AND (Questions = 'Q22');

--delete Derived_Values;

And then:

SELECT * FROM Derived_Values;
go

select * from Derived_Values_Test;


BUSINESSUNIT    QUESTIONS   ANSWER
BU1 Q11 Updated Answers A11
BU1 Q12 Updated Answers A12
BU2 Q21 Updated Answers A21
BU2 Q22 Updated Answers A22

BUSINESSUNIT    QUESTIONS   ANSWER
BU1 Q11 Updated Answers A11
BU1 Q12 Updated Answers A12
BU2 Q21 Updated Answers A21
BU2 Q22 Updated Answers A22

Replacing NULL with 0 in a SQL server query

Use COALESCE, which returns the first not-null value e.g.

SELECT COALESCE(sum(case when c.runstatus = 'Succeeded' then 1 end), 0) as Succeeded

Will set Succeeded as 0 if it is returned as NULL.

Best practices when running Node.js with port 80 (Ubuntu / Linode)

Drop root privileges after you bind to port 80 (or 443).

This allows port 80/443 to remain protected, while still preventing you from serving requests as root:

function drop_root() {
    process.setgid('nobody');
    process.setuid('nobody');
}

A full working example using the above function:

var process = require('process');
var http = require('http');
var server = http.createServer(function(req, res) {
    res.write("Success!");
    res.end();
});

server.listen(80, null, null, function() {
    console.log('User ID:',process.getuid()+', Group ID:',process.getgid());
    drop_root();
    console.log('User ID:',process.getuid()+', Group ID:',process.getgid());
});

See more details at this full reference.

node.js: cannot find module 'request'

I had same problem, for me npm install request --save solved the problem. Hope it helps.

Can you run GUI applications in a Docker container?

Here's a lightweight solution that avoids having to install any X server, vnc server or sshd daemon on the container. What it gains in simplicity it loses in security and isolation.

It assumes that you connect to the host machine using ssh with X11 forwarding.

In the sshd configuration of the host, add the line

X11UseLocalhost no

So that the forwarded X server port on the host is opened on all interfaces (not just lo) and in particular on the Docker virtual interface, docker0.

The container, when run, needs access to the .Xauthority file so that it can connect to the server. In order to do that, we define a read-only volume pointing to the home directory on the host (maybe not a wise idea!) and also set the XAUTHORITY variable accordingly.

docker run -v $HOME:/hosthome:ro -e XAUTHORITY=/hosthome/.Xauthority

That is not enough, we also have to pass the DISPLAY variable from the host, but substituting the hostname by the ip:

-e DISPLAY=$(echo $DISPLAY | sed "s/^.*:/$(hostname -i):/")

We can define an alias:

 alias dockerX11run='docker run -v $HOME:/hosthome:ro -e XAUTHORITY=/hosthome/.Xauthority -e DISPLAY=$(echo $DISPLAY | sed "s/^.*:/$(hostname -i):/")'

And test it like this:

dockerX11run centos xeyes

SQL SERVER: Check if variable is null and then assign statement for Where Clause

is null can be used to check whether null data is coming from a query as in following example:

 declare @Mem varchar(20),@flag int
select @mem=MemberClub from [dbo].[UserMaster] where UserID=@uid
if(@Mem is null)
begin
    set @flag= 0;
end
else
begin
    set @flag=1;
end
return @flag;

How to generate and auto increment Id with Entity Framework

You have a bad table design. You can't autoincrement a string, that doesn't make any sense. You have basically two options:

1.) change type of ID to int instead of string
2.) not recommended!!! - handle autoincrement by yourself. You first need to get the latest value from the database, parse it to the integer, increment it and attach it to the entity as a string again. VERY BAD idea

First option requires to change every table that has a reference to this table, BUT it's worth it.

Subquery returned more than 1 value.This is not permitted when the subquery follows =,!=,<,<=,>,>= or when the subquery is used as an expression

Use In instead of =

 select * from dbo.books
 where isbn in (select isbn from dbo.lending 
                where act between @fdate and @tdate
                and stat ='close'
               )

or you can use Exists

SELECT t1.*,t2.*
FROM  books   t1 
WHERE  EXISTS ( SELECT * FROM dbo.lending t2 WHERE t1.isbn = t2.isbn and
                t2.act between @fdate and @tdate and t2.stat ='close' )

How to Solve Max Connection Pool Error

May be this is alltime multiple connection open issue, you are somewhere in your code opening connections and not closing them properly. use

 using (SqlConnection con = new SqlConnection(connectionString))
        {
            con.Open();
         }

Refer this article: http://msdn.microsoft.com/en-us/library/ms254507(v=vs.80).aspx, The Using block in Visual Basic or C# automatically disposes of the connection when the code exits the block, even in the case of an unhandled exception.

I want to use CASE statement to update some records in sql server 2005

If you don't want to repeat the list twice (as per @J W's answer), then put the updates in a table variable and use a JOIN in the UPDATE:

declare @ToDo table (FromName varchar(10), ToName varchar(10))
insert into @ToDo(FromName,ToName) values
 ('AAA','BBB'),
 ('CCC','DDD'),
 ('EEE','FFF')

update ts set LastName = ToName
from dbo.TestStudents ts
       inner join
     @ToDo t
       on
         ts.LastName = t.FromName

Sending an HTTP POST request on iOS

-(void)sendingAnHTTPPOSTRequestOniOSWithUserEmailId: (NSString *)emailId withPassword: (NSString *)password{
//Init the NSURLSession with a configuration
NSURLSessionConfiguration *defaultConfigObject = [NSURLSessionConfiguration defaultSessionConfiguration];
NSURLSession *defaultSession = [NSURLSession sessionWithConfiguration: defaultConfigObject delegate: nil delegateQueue: [NSOperationQueue mainQueue]];

//Create an URLRequest
NSURL *url = [NSURL URLWithString:@"http://www.example.com/apis/login_api"];
NSMutableURLRequest *urlRequest = [NSMutableURLRequest requestWithURL:url];

//Create POST Params and add it to HTTPBody
NSString *params = [NSString stringWithFormat:@"email=%@&password=%@",emailId,password];
[urlRequest setHTTPMethod:@"POST"];
[urlRequest setHTTPBody:[params dataUsingEncoding:NSUTF8StringEncoding]];

//Create task
NSURLSessionDataTask *dataTask = [defaultSession dataTaskWithRequest:urlRequest completionHandler:^(NSData *data, NSURLResponse *response, NSError *error) {
    //Handle your response here
    NSDictionary *responseDict = [NSJSONSerialization JSONObjectWithData:data options:NSJSONReadingAllowFragments error:nil];
     NSLog(@"%@",responseDict);
}];
   [dataTask resume];
}

TypeError: $(...).on is not a function

The usual cause of this is that you're also using Prototype, MooTools, or some other library that makes use of the $ symbol, and you're including that library after jQuery, and so that library is "winning" (taking $ for itself). So the return value of $ isn't a jQuery instance, and so it doesn't have jQuery methods on it (like on).

You can use jQuery with those other libraries, but if you do, you have to use the jQuery symbol rather than its alias $, e.g.:

jQuery('body').on(...);

And it's usually best if you add this immediately after your script tag including jQuery, before the one including the other library:

<script>jQuery.noConflict();</script>

...although it's not required if you load the other library after jQuery (it is if you load the other library first).

Using multiple full-function DOM manipulation libraries on the same page isn't ideal, though, just in terms of page weight. So if you can stick with just Prototype/MooTools/whatever or just jQuery, that's usually better.

HTML iframe - disable scroll

This works for me:

<style>
*{overflow:hidden!important;}
html{overflow:scroll!important;}
</style>

Note: if you need scrollbar in any other element, also add css {overflow:scroll!important;} to that element

Change Schema Name Of Table In SQL

Create Schema :

IF (NOT EXISTS (SELECT * FROM sys.schemas WHERE name = 'exe')) 
BEGIN
    EXEC ('CREATE SCHEMA [exe] AUTHORIZATION [dbo]')
END

ALTER Schema :

ALTER SCHEMA exe 
    TRANSFER dbo.Employees

Multiple condition in single IF statement

Yes that is valid syntax but it may well not do what you want.

Execution will continue after your RAISERROR except if you add a RETURN. So you will need to add a block with BEGIN ... END to hold the two statements.

Also I'm not sure why you plumped for severity 15. That usually indicates a syntax error.

Finally I'd simplify the conditions using IN

CREATE PROCEDURE [dbo].[AddApplicationUser] (@TenantId BIGINT,
                                            @UserType TINYINT,
                                            @UserName NVARCHAR(100),
                                            @Password NVARCHAR(100))
AS
  BEGIN
      IF ( @TenantId IS NULL
           AND @UserType IN ( 0, 1 ) )
        BEGIN
            RAISERROR('The value for @TenantID should not be null',15,1);

            RETURN;
        END
  END 

Create a view with ORDER BY clause

In order to add an ORDER BY to a View Perform the following

CREATE VIEW [dbo].[SQLSTANDARDS_PSHH]
AS


SELECT TOP 99999999999999
Column1,
Column2
FROM
dbo.Table
Order by
Column1

How to get list of all installed packages along with version in composer?

Ivan's answer above is good:

composer global show -i

Added info: if you get a message somewhat like:

Composer could not find a composer.json file in ~/.composer

...you might have no packages installed yet. If so, you can ignore the next part of the message containing:

... please create a composer.json file ...

...as once you install a package the message will go away.

How can I drop a table if there is a foreign key constraint in SQL Server?

Type this .... SET foreign_key_checks = 0;
delete your table then type SET foreign_key_checks = 1;

MySQL – Temporarily disable Foreign Key Checks or Constraints

String or binary data would be truncated. The statement has been terminated

When you define varchar etc without a length, the default is 1.

When n is not specified in a data definition or variable declaration statement, the default length is 1. When n is not specified with the CAST function, the default length is 30.

So, if you expect 400 bytes in the @trackingItems1 column from stock, use nvarchar(400).

Otherwise, you are trying to fit >1 character into nvarchar(1) = fail

As a comment, this is bad use of table value function too because it is "multi statement". It can be written like this and it will run better

ALTER FUNCTION [dbo].[testing1](@price int)
RETURNS
AS
   SELECT ta.item, ta.warehouse, ta.price 
   FROM   stock ta
   WHERE  ta.price >= @price;

Of course, you could just use a normal SELECT statement..

How to use a TRIM function in SQL Server

TRIM all SPACE's TAB's and ENTER's:

DECLARE @Str VARCHAR(MAX) = '      
          [         Foo    ]       
          '

DECLARE @NewStr VARCHAR(MAX) = ''
DECLARE @WhiteChars VARCHAR(4) =
      CHAR(13) + CHAR(10) -- ENTER
    + CHAR(9) -- TAB
    + ' ' -- SPACE

;WITH Split(Chr, Pos) AS (
    SELECT
          SUBSTRING(@Str, 1, 1) AS Chr
        , 1 AS Pos
    UNION ALL
    SELECT
          SUBSTRING(@Str, Pos, 1) AS Chr
        , Pos + 1 AS Pos
    FROM Split
    WHERE Pos <= LEN(@Str)
)
SELECT @NewStr = @NewStr + Chr
FROM Split
WHERE
    Pos >= (
        SELECT MIN(Pos)
        FROM Split
        WHERE CHARINDEX(Chr, @WhiteChars) = 0
    )
    AND Pos <= (
        SELECT MAX(Pos)
        FROM Split
        WHERE CHARINDEX(Chr, @WhiteChars) = 0
    )

SELECT '"' + @NewStr + '"'

As Function

CREATE FUNCTION StrTrim(@Str VARCHAR(MAX)) RETURNS VARCHAR(MAX) BEGIN
    DECLARE @NewStr VARCHAR(MAX) = NULL

    IF (@Str IS NOT NULL) BEGIN
        SET @NewStr = ''

        DECLARE @WhiteChars VARCHAR(4) =
              CHAR(13) + CHAR(10) -- ENTER
            + CHAR(9) -- TAB
            + ' ' -- SPACE

        IF (@Str LIKE ('%[' + @WhiteChars + ']%')) BEGIN

            ;WITH Split(Chr, Pos) AS (
                SELECT
                      SUBSTRING(@Str, 1, 1) AS Chr
                    , 1 AS Pos
                UNION ALL
                SELECT
                      SUBSTRING(@Str, Pos, 1) AS Chr
                    , Pos + 1 AS Pos
                FROM Split
                WHERE Pos <= LEN(@Str)
            )
            SELECT @NewStr = @NewStr + Chr
            FROM Split
            WHERE
                Pos >= (
                    SELECT MIN(Pos)
                    FROM Split
                    WHERE CHARINDEX(Chr, @WhiteChars) = 0
                )
                AND Pos <= (
                    SELECT MAX(Pos)
                    FROM Split
                    WHERE CHARINDEX(Chr, @WhiteChars) = 0
                )
        END
    END

    RETURN @NewStr
END

Example

-- Test
DECLARE @Str VARCHAR(MAX) = '      
          [         Foo    ]       
              '

SELECT 'Str', '"' + dbo.StrTrim(@Str) + '"'
UNION SELECT 'EMPTY', '"' + dbo.StrTrim('') + '"'
UNION SELECT 'EMTPY', '"' + dbo.StrTrim('      ') + '"'
UNION SELECT 'NULL', '"' + dbo.StrTrim(NULL) + '"'

Result

+-------+----------------+
| Test  | Result         |
+-------+----------------+
| EMPTY | ""             |
| EMTPY | ""             |
| NULL  | NULL           |
| Str   | "[   Foo    ]" |
+-------+----------------+

SQL Server stored procedure creating temp table and inserting value

A SELECT INTO statement creates the table for you. There is no need for the CREATE TABLE statement before hand.

What is happening is that you create #ivmy_cash_temp1 in your CREATE statement, then the DB tries to create it for you when you do a SELECT INTO. This causes an error as it is trying to create a table that you have already created.

Either eliminate the CREATE TABLE statement or alter your query that fills it to use INSERT INTO SELECT format.

If you need a unique ID added to your new row then it's best to use SELECT INTO... since IDENTITY() only works with this syntax.

Remove certain characters from a string

One issue with REPLACE will be where city names contain the district name. You can use something like.

SELECT SUBSTRING(O.Ort, LEN(C.CityName) + 2, 8000)
FROM   dbo.tblOrtsteileGeo O
       JOIN dbo.Cities C
         ON C.foo = O.foo
WHERE  O.GKZ = '06440004' 

INSERT INTO from two different server database

It sounds like you might need to create and query linked database servers in SQL Server

At the moment you've created a query that's going between different databases using a 3 part name mydatabase.dbo.mytable but you need to go up a level and use a 4 part name myserver.mydatabase.dbo.mytable, see this post on four part naming for more info

edit
The four part naming for your existing query would be as shown below (which I suspect you may have already tried?), but this assumes you can "get to" the remote database with the four part name, you might need to edit your host file / register the server or otherwise identify where to find database.windows.net.

INSERT INTO [DATABASE.WINDOWS.NET].[basecampdev].[dbo].[invoice]
       ([InvoiceNumber]
       ,[TotalAmount]
       ,[IsActive]
       ,[CreatedBy]
       ,[UpdatedBy]
       ,[CreatedDate]
       ,[UpdatedDate]
       ,[Remarks])
SELECT [InvoiceNumber]
       ,[TotalAmount]
       ,[IsActive]
       ,[CreatedBy]
       ,[UpdatedBy]
       ,[CreatedDate]
       ,[UpdatedDate]
       ,[Remarks] FROM [BC1-PC].[testdabse].[dbo].[invoice]

If you can't access the remote server then see if you can create a linked database server:

EXEC sp_addlinkedserver [database.windows.net];
GO
USE tempdb;
GO
CREATE SYNONYM MyInvoice FOR 
    [database.windows.net].basecampdev.dbo.invoice;
GO

Then you can just query against MyEmployee without needing the full four part name

Use stored procedure to insert some data into a table

If you are trying to return back the ID within the scope, using the SCOPE_IDENTITY() would be a better approach. I would not advice to use @@IDENTITY, as this can return any ID.

CREATE PROC [dbo].[sp_Test] (
  @myID int output,
  @myFirstName nvarchar(50),
  @myLastName nvarchar(50),
  @myAddress nvarchar(50),
  @myPort int
) AS
BEGIN
    INSERT INTO Dvds (myFirstName, myLastName, myAddress, myPort)
    VALUES (@myFirstName, @myLastName, @myAddress, @myPort);

    SET @myID = SCOPE_IDENTITY();
END
GO

Stored Procedure parameter default value - is this a constant or a variable

It has to be a constant - the value has to be computable at the time that the procedure is created, and that one computation has to provide the value that will always be used.

Look at the definition of sys.all_parameters:

default_value sql_variant If has_default_value is 1, the value of this column is the value of the default for the parameter; otherwise, NULL.

That is, whatever the default for a parameter is, it has to fit in that column.


As Alex K pointed out in the comments, you can just do:

CREATE PROCEDURE [dbo].[problemParam] 
    @StartDate INT = NULL,
    @EndDate INT = NULL
AS  
BEGIN
   SET @StartDate = COALESCE(@StartDate,CONVERT(INT,(CONVERT(CHAR(8),GETDATE()-130,112))))

provided that NULL isn't intended to be a valid value for @StartDate.


As to the blog post you linked to in the comments - that's talking about a very specific context - that, the result of evaluating GETDATE() within the context of a single query is often considered to be constant. I don't know of many people (unlike the blog author) who would consider a separate expression inside a UDF to be part of the same query as the query that calls the UDF.

@Cacheable key on multiple method arguments

Update: Current Spring cache implementation uses all method parameters as the cache key if not specified otherwise. If you want to use selected keys, refer to Arjan's answer which uses SpEL list {#isbn, #includeUsed} which is the simplest way to create unique keys.

From Spring Documentation

The default key generation strategy changed with the release of Spring 4.0. Earlier versions of Spring used a key generation strategy that, for multiple key parameters, only considered the hashCode() of parameters and not equals(); this could cause unexpected key collisions (see SPR-10237 for background). The new 'SimpleKeyGenerator' uses a compound key for such scenarios.

Before Spring 4.0

I suggest you to concat the values of the parameters in Spel expression with something like key="#checkWarehouse.toString() + #isbn.toString()"), I believe this should work as org.springframework.cache.interceptor.ExpressionEvaluator returns Object, which is later used as the key so you don't have to provide an int in your SPEL expression.

As for the hash code with a high collision probability - you can't use it as the key.

Someone in this thread has suggested to use T(java.util.Objects).hash(#p0,#p1, #p2) but it WILL NOT WORK and this approach is easy to break, for example I've used the data from SPR-9377 :

    System.out.println( Objects.hash("someisbn", new Integer(109), new Integer(434)));
    System.out.println( Objects.hash("someisbn", new Integer(110), new Integer(403)));

Both lines print -636517714 on my environment.

P.S. Actually in the reference documentation we have

@Cacheable(value="books", key="T(someType).hash(#isbn)") 
public Book findBook(ISBN isbn, boolean checkWarehouse, boolean includeUsed)

I think that this example is WRONG and misleading and should be removed from the documentation, as the keys should be unique.

P.P.S. also see https://jira.springsource.org/browse/SPR-9036 for some interesting ideas regarding the default key generation.

I'd like to add for the sake of correctness and as an entertaining mathematical/computer science fact that unlike built-in hash, using a secure cryptographic hash function like MD5 or SHA256, due to the properties of such function IS absolutely possible for this task, but to compute it every time may be too expensive, checkout for example Dan Boneh cryptography course to learn more.

Git - push current branch shortcut

You should take a look to a similar question in Default behavior of "git push" without a branch specified

Basically it explains how to set the default behavior to push your current branch just executing git push. Probably what you need is:

git config --global push.default current

Other options:

  • nothing : Do not push anything
  • matching : Push all matching branches
  • upstream/tracking : Push the current branch to whatever it is tracking
  • current : Push the current branch

Drop primary key using script in SQL Server database

You can look up the constraint name in the sys.key_constraints table:

SELECT name
FROM   sys.key_constraints
WHERE  [type] = 'PK'
       AND [parent_object_id] = Object_id('dbo.Student');

If you don't care about the name, but simply want to drop it, you can use a combination of this and dynamic sql:

DECLARE @table NVARCHAR(512), @sql NVARCHAR(MAX);

SELECT @table = N'dbo.Student';

SELECT @sql = 'ALTER TABLE ' + @table 
    + ' DROP CONSTRAINT ' + name + ';'
    FROM sys.key_constraints
    WHERE [type] = 'PK'
    AND [parent_object_id] = OBJECT_ID(@table);

EXEC sp_executeSQL @sql;

This code is from Aaron Bertrand (source).

SQL Server principal "dbo" does not exist,

This may also happen when the database is a restore from a different SQL server or instance. In that case, the security principal 'dbo' in the database is not the same as the security principal on the SQL server on which the db was restored. Don't ask me how I know this...

How to pass multiple values to single parameter in stored procedure

I think, below procedure help you to what you are looking for.

 CREATE PROCEDURE [dbo].[FindEmployeeRecord]
        @EmployeeID nvarchar(Max)
    AS
    BEGIN
    DECLARE @sqLQuery VARCHAR(MAX)
    Declare @AnswersTempTable Table
    (  
        EmpId int,         
        EmployeeName nvarchar (250),       
        EmployeeAddress nvarchar (250),  
        PostalCode nvarchar (50),
        TelephoneNo nvarchar (50),
        Email nvarchar (250),
        status nvarchar (50),  
        Sex nvarchar (50) 
    )

    Set @sqlQuery =
    'select e.EmpId,e.EmployeeName,e.Email,e.Sex,ed.EmployeeAddress,ed.PostalCode,ed.TelephoneNo,ed.status
    from Employee e
    join EmployeeDetail ed on e.Empid = ed.iEmpID
    where Convert(nvarchar(Max),e.EmpId) in ('+@EmployeeId+')
    order by EmpId'
    Insert into @AnswersTempTable
    exec (@sqlQuery)
    select * from @AnswersTempTable
    END

Inserting values into a SQL Server database using ado.net via C#

you should remove last comma and as nrodic said your command is not correct.

you should change it like this :

SqlCommand cmd = new SqlCommand("INSERT INTO dbo.regist (" + " FirstName, Lastname, Username, Password, Age, Gender,Contact " + ") VALUES (" + " textBox1.Text, textBox2.Text, textBox3.Text, textBox4.Text, comboBox1.Text,comboBox2.Text,textBox7.Text" + ")", cn);

Linq where clause compare only date value without time value

There is also EntityFunctions.TruncateTime or DbFunctions.TruncateTime in EF 6.0

Entity Framework Migrations renaming tables and columns

In EF Core, I use the following statements to rename tables and columns:

As for renaming tables:

    protected override void Up(MigrationBuilder migrationBuilder)
    {
        migrationBuilder.RenameTable(name: "OldTableName", schema: "dbo", newName: "NewTableName", newSchema: "dbo");
    }

    protected override void Down(MigrationBuilder migrationBuilder)
    {
        migrationBuilder.RenameTable(name: "NewTableName", schema: "dbo", newName: "OldTableName", newSchema: "dbo");
    }

As for renaming columns:

    protected override void Up(MigrationBuilder migrationBuilder)
    {
        migrationBuilder.RenameColumn(name: "OldColumnName", table: "TableName", newName: "NewColumnName", schema: "dbo");
    }

    protected override void Down(MigrationBuilder migrationBuilder)
    {
        migrationBuilder.RenameColumn(name: "NewColumnName", table: "TableName", newName: "OldColumnName", schema: "dbo");
    }

Bulk load data conversion error (type mismatch or invalid character for the specified codepage) for row 1, column 4 (Year)

In my case, I was dealing with a file that was generated by hadoop on a linux box. When I tried to import to sql I had this issue. The fix wound up being to use the hex value for 'line feed' 0x0a. It also worked for bulk insert

bulk insert table from 'file' 
WITH (FIELDTERMINATOR = ',', ROWTERMINATOR = '0x0a')

The property 'value' does not exist on value of type 'HTMLElement'

Based on Tomasz Nurkiewiczs answer, the "problem" is that typescript is typesafe. :) So the document.getElementById() returns the type HTMLElement which does not contain a value property. The subtype HTMLInputElement does however contain the value property.

So a solution is to cast the result of getElementById() to HTMLInputElement like this:

var inputValue = (<HTMLInputElement>document.getElementById(elementId)).value;

<> is the casting operator in typescript. See the question TypeScript: casting HTMLElement.

The resulting javascript from the line above looks like this:

inputValue = (document.getElementById(elementId)).value;

i.e. containing no type information.

Stored procedure return into DataSet in C# .Net

Try this

    DataSet ds = new DataSet("TimeRanges");
    using(SqlConnection conn = new SqlConnection("ConnectionString"))
    {               
            SqlCommand sqlComm = new SqlCommand("Procedure1", conn);               
            sqlComm.Parameters.AddWithValue("@Start", StartTime);
            sqlComm.Parameters.AddWithValue("@Finish", FinishTime);
            sqlComm.Parameters.AddWithValue("@TimeRange", TimeRange);

            sqlComm.CommandType = CommandType.StoredProcedure;

            SqlDataAdapter da = new SqlDataAdapter();
            da.SelectCommand = sqlComm;

            da.Fill(ds);
     }

How to convert Nvarchar column to INT

I know its Too late But I hope it will work new comers Try This Its Working ... :D

select 
  case 
      when isnumeric(my_NvarcharColumn) = 1 then 
              cast(my_NvarcharColumn AS int)
      else
              NULL
 end

AS 'my_NvarcharColumnmitter'
from A

Can I pass variable to select statement as column name in SQL Server

You can't use variable names to bind columns or other system objects, you need dynamic sql

DECLARE @value varchar(10)  
SET @value = 'intStep'  
DECLARE @sqlText nvarchar(1000); 

SET @sqlText = N'SELECT ' + @value + ' FROM dbo.tblBatchDetail'
Exec (@sqlText)

How to check if a String contains any letter from a to z?

What about:

//true if it doesn't contain letters
bool result = hello.Any(x => !char.IsLetter(x));

No numeric types to aggregate - change in groupby() behaviour?

I got this error generating a data frame consisting of timestamps and data:

df = pd.DataFrame({'data':value}, index=pd.DatetimeIndex(timestamp))

Adding the suggested solution works for me:

df = pd.DataFrame({'data':value}, index=pd.DatetimeIndex(timestamp), dtype=float))

Thanks Chang She!

Example:

                     data
2005-01-01 00:10:00  7.53
2005-01-01 00:20:00  7.54
2005-01-01 00:30:00  7.62
2005-01-01 00:40:00  7.68
2005-01-01 00:50:00  7.81
2005-01-01 01:00:00  7.95
2005-01-01 01:10:00  7.96
2005-01-01 01:20:00  7.95
2005-01-01 01:30:00  7.98
2005-01-01 01:40:00  8.06
2005-01-01 01:50:00  8.04
2005-01-01 02:00:00  8.06
2005-01-01 02:10:00  8.12
2005-01-01 02:20:00  8.12
2005-01-01 02:30:00  8.25
2005-01-01 02:40:00  8.27
2005-01-01 02:50:00  8.17
2005-01-01 03:00:00  8.21
2005-01-01 03:10:00  8.29
2005-01-01 03:20:00  8.31
2005-01-01 03:30:00  8.25
2005-01-01 03:40:00  8.19
2005-01-01 03:50:00  8.17
2005-01-01 04:00:00  8.18
                     data
2005-01-01 00:00:00  7.636000
2005-01-01 01:00:00  7.990000
2005-01-01 02:00:00  8.165000
2005-01-01 03:00:00  8.236667
2005-01-01 04:00:00  8.180000

Are arrays passed by value or passed by reference in Java?

Everything in Java is passed by value .

In the case of the array the reference is copied into a new reference, but remember that everything in Java is passed by value .

Take a look at this interesting article for further information ...

Alter a SQL server function to accept new optional parameter

From CREATE FUNCTION:

When a parameter of the function has a default value, the keyword DEFAULT must be specified when the function is called to retrieve the default value. This behavior is different from using parameters with default values in stored procedures in which omitting the parameter also implies the default value.

So you need to do:

SELECT dbo.fCalculateEstimateDate(647,DEFAULT)

How do CORS and Access-Control-Allow-Headers work?

Yes, you need to have the header Access-Control-Allow-Origin: http://domain.com:3000 or Access-Control-Allow-Origin: * on both the OPTIONS response and the POST response. You should include the header Access-Control-Allow-Credentials: true on the POST response as well.

Your OPTIONS response should also include the header Access-Control-Allow-Headers: origin, content-type, accept to match the requested header.

Failed to start component [StandardEngine[Catalina].StandardHost[localhost].StandardContext[/JDBC_DBO]]

web module -> Properties -> Deployment Assembly -> (add folder "src/main/webapp", Maven Dependencies and other needed module)

How do check if a parameter is empty or null in Sql Server stored procedure in IF statement?

that is the right behavior.

if you set @item1 to a value the below expression will be true

IF (@item1 IS NOT NULL) OR (LEN(@item1) > 0)

Anyway in SQL Server there is not a such function but you can create your own:

CREATE FUNCTION dbo.IsNullOrEmpty(@x varchar(max)) returns bit as
BEGIN
IF @SomeVarcharParm IS NOT NULL AND LEN(@SomeVarcharParm) > 0
    RETURN 0
ELSE
    RETURN 1
END

How to do a SOAP wsdl web services call from the command line

For Windows users looking for a PowerShell alternative, here it is (using POST). I've split it up onto multiple lines for readability.

$url = 'https://sandbox.mediamind.com/Eyeblaster.MediaMind.API/V2/AuthenticationService.svc'
$headers = @{
    'Content-Type' = 'text/xml';
    'SOAPAction' = 'http://api.eyeblaster.com/IAuthenticationService/ClientLogin'
}
$envelope = @'
    <Envelope xmlns="http://schemas.xmlsoap.org/soap/envelope/">
        <Body>
            <yourEnvelopeContentsHere/>
        </Body>
    </Envelope>
'@     # <--- This line must not be indented

Invoke-WebRequest -Uri $url -Headers $headers -Method POST -Body $envelope

SQL update trigger only when column is modified

You want to do the following:

ALTER TRIGGER [dbo].[tr_SCHEDULE_Modified]
   ON [dbo].[SCHEDULE]
   AFTER UPDATE
AS 
BEGIN
SET NOCOUNT ON;

    IF (UPDATE(QtyToRepair))
    BEGIN
        UPDATE SCHEDULE SET modified = GETDATE()
            , ModifiedUser = SUSER_NAME()
            , ModifiedHost = HOST_NAME()
        FROM SCHEDULE S
        INNER JOIN Inserted I ON S.OrderNo = I.OrderNo AND S.PartNumber = I.PartNumber
        WHERE S.QtyToRepair <> I.QtyToRepair
    END
END

Please note that this trigger will fire each time you update the column no matter if the value is the same or not.

getting "No column was specified for column 2 of 'd'" in sql server cte?

evidently, as stated in the parser response, a column name is needed for both cases. In either versions the columns of "d" are not named.

in case 1: your column 2 of d is sum(totalitems) which is not named. duration will retain the name "duration"

in case 2: both month(clothdeliverydate) and SUM(CONVERT(INT, deliveredqty)) have to be named

Set IDENTITY_INSERT ON is not working

You might be just missing the column list, as the message says

SET IDENTITY_INSERT [MyDB].[dbo].[Equipment] ON

INSERT INTO [MyDB].[dbo].[Equipment]
            (COL1,
             COL2)
SELECT COL1,
       COL2
FROM   [MyDBQA].[dbo].[Equipment]

SET IDENTITY_INSERT [MyDB].[dbo].[Equipment] OFF 

SQL where datetime column equals today's date?

Can you try this?

SELECT [Title], [Firstname], [Surname], [Company_name], [Interest] 
FROM [dbo].[EXTRANET_users] 
WHERE CAST(Submission_date AS DATE) = CAST(GETDATE() AS DATE)

T-SQL doesn't really have the "implied" casting like C# does - you need to explicitly use CAST (or CONVERT).

Also, use GETDATE() or CURRENT_TIMESTAMP to get the "now" date and time.

Update: since you're working against SQL Server 2000 - none of those approaches so far work. Try this instead:

SELECT [Title], [Firstname], [Surname], [Company_name], [Interest] 
FROM [dbo].[EXTRANET_users] 
WHERE DATEADD(dd, 0, DATEDIFF(dd, 0, submission_date)) = DATEADD(dd, 0, DATEDIFF(dd, 0, GETDATE()))

Optional query string parameters in ASP.NET Web API

Use initial default values for all parameters like below

public string GetFindBooks(string author="", string title="", string isbn="", string  somethingelse="", DateTime? date= null) 
{
    // ...
}

Inner Joining three tables

Just do the same thing agin but then for TableC

SELECT *
FROM dbo.tableA A 
INNER JOIN dbo.TableB B ON A.common = B.common
INNER JOIN dbo.TableC C ON A.common = C.common

Check for file exists or not in sql server?

Not tested but you can try something like this :

Declare @count as int
Set @count=1
Declare @inputFile varchar(max)
Declare @Sample Table
(id int,filepath varchar(max) ,Isexists char(3))

while @count<(select max(id) from yourTable)
BEGIN
Set @inputFile =(Select filepath from yourTable where id=@count)
DECLARE @isExists INT
exec master.dbo.xp_fileexist @inputFile , 
@isExists OUTPUT
insert into @Sample
Select @count,@inputFile ,case @isExists 
when 1 then 'Yes' 
else 'No' 
end as isExists
set @count=@count+1
END

How to pass arguments from command line to gradle

There's a great example here:

https://kb.novaordis.com/index.php/Gradle_Pass_Configuration_on_Command_Line

Which details that you can pass parameters and then provide a default in an ext variable like so:

gradle -Dmy_app.color=blue

and then reference in Gradle as:

ext {
   color = System.getProperty("my_app.color", "red");
}

And then anywhere in your build script you can reference it as course anywhere you can reference it as project.ext.color

More tips here: https://kb.novaordis.com/index.php/Gradle_Variables_and_Properties

Retrieving Data from SQL Using pyodbc

import pyodbc
conn = pyodbc.connect('Driver={SQL Server};'
                  'Server=db-server;'
                  'Database=db;'
                  'Trusted_Connection=yes;')
sql = "SELECT * FROM [mytable] "
cursor.execute(sql)
for r in cursor:
    print(r)

Self Join to get employee manager name

TableName :Manager

EmpId   EmpName     ManagerId
1       Monib       4

2       zahir       1

3       Sarfudding  NULL

4       Aslam       3


select e.EmpId as EmployeeId,e.EmpName as EmployeeName,e.ManagerId as ManagerId,e1.EmpName as Managername from Manager e
  join manager e1 on e.ManagerId=e1.empId

Save bitmap to file function

Two example works for me, for your reference.

Bitmap bitmap = Utils.decodeBase64(base64);
try {
    File file = new File(filePath);
    FileOutputStream fOut = new FileOutputStream(file);
    bitmap.compress(Bitmap.CompressFormat.PNG, 85, fOut);
    fOut.flush();
    fOut.close();
}
catch (Exception e) {
    e.printStackTrace();
    LOG.i(null, "Save file error!");
    return false;
}

and this one

Bitmap savePic = Utils.decodeBase64(base64);
File file = new File(filePath);
File path = new File(file.getParent());

if (savePic != null) {
    try {
        // build directory
        if (file.getParent() != null && !path.isDirectory()) {
            path.mkdirs();
        }
        // output image to file
        FileOutputStream fos = new FileOutputStream(filePath);
        savePic.compress(Bitmap.CompressFormat.PNG, 90, fos);
        fos.close();
        ret = true;
    } catch (Exception e) {
        e.printStackTrace();
    }
} else {
    LOG.i(TAG, "savePicture image parsing error");
}

How to describe table in SQL Server 2008?

According to this documentation:

DESC MY_TABLE

is equivalent to

SELECT column_name "Name", nullable "Null?", concat(concat(concat(data_type,'('),data_length),')') "Type" FROM user_tab_columns WHERE table_name='TABLE_NAME_TO_DESCRIBE';

I've roughly translated that to the SQL Server equivalent for you - just make sure you're running it on the EX database.

SELECT column_name AS [name],
       IS_NULLABLE AS [null?],
       DATA_TYPE + COALESCE('(' + CASE WHEN CHARACTER_MAXIMUM_LENGTH = -1
                                  THEN 'Max'
                                  ELSE CAST(CHARACTER_MAXIMUM_LENGTH AS VARCHAR(5))
                                  END + ')', '') AS [type]
FROM   INFORMATION_SCHEMA.Columns
WHERE  table_name = 'EMP_MAST'

Export table data from one SQL Server to another

For copying data from source to destination:

use <DestinationDatabase>
select * into <DestinationTable> from <SourceDataBase>.dbo.<SourceTable>

Uncaught syntaxerror: unexpected identifier?

There are errors here :

var formTag = document.getElementsByTagName("form"), // form tag is an array
selectListItem = $('select'),
makeSelect = document.createElement('select'),
makeSelect.setAttribute("id", "groups");

The code must change to:

var formTag = document.getElementsByTagName("form");
var selectListItem = $('select');
var makeSelect = document.createElement('select');
makeSelect.setAttribute("id", "groups");

By the way, there is another error at line 129 :

var createLi.appendChild(createSubList);

Replace it with:

createLi.appendChild(createSubList);

Using stored procedure output parameters in C#

Stored Procedure.........

CREATE PROCEDURE usp_InsertContract
    @ContractNumber varchar(7)
AS
BEGIN

    INSERT into [dbo].[Contracts] (ContractNumber)
        VALUES (@ContractNumber)

    SELECT SCOPE_IDENTITY() AS [SCOPE_IDENTITY]
END

C#

pvCommand.CommandType = CommandType.StoredProcedure;

pvCommand.Parameters.Clear();
pvCommand.Parameters.Add(new SqlParameter("@ContractNumber", contractNumber));
object uniqueId;
int id;
    try
    {
    uniqueId = pvCommand.ExecuteScalar();
     id = Convert.ToInt32(uniqueId);
    }
    catch (Exception e)
    {
        Debug.Print("  Message: {0}", e.Message);
    }
}

EDIT: "I still get back a DBNull value....Object cannot be cast from DBNull to other types. I'll take this up again tomorrow. I'm off to my other job,"

I believe the Id column in your SQL Table isn't a identity column.

enter image description here

BULK INSERT with identity (auto-increment) column

This is a very old post to answer, but none of the answers given solves the problem without changing the posed conditions, which I can't do.

I solved it by using the OPENROWSET variant of BULK INSERT. This uses the same format file and works in the same way, but it allows the data file be read with a SELECT statement.

Create your table:

CREATE TABLE target_table(
id bigint IDENTITY(1,1),
col1 varchar(256) NULL,
col2 varchar(256) NULL,
col3 varchar(256) NULL)

Open a command window an run:

bcp dbname.dbo.target_table format nul -c -x -f C:\format_file.xml -t; -T

This creates the format file based on how the table looks.

Now edit the format file and remove the entire rows where FIELD ID="1" and COLUMN SOURCE="1", since this does not exist in our data file.
Also adjust terminators as may be needed for your data file:

<?xml version="1.0"?>
<BCPFORMAT xmlns="http://schemas.microsoft.com/sqlserver/2004/bulkload/format" xmlns:xsi="http://www.w3.org/2001/XMLSchema-instance">
 <RECORD>
  <FIELD ID="2" xsi:type="CharTerm" TERMINATOR=";" MAX_LENGTH="256" COLLATION="Finnish_Swedish_CI_AS"/>
  <FIELD ID="3" xsi:type="CharTerm" TERMINATOR=";" MAX_LENGTH="256" COLLATION="Finnish_Swedish_CI_AS"/>
  <FIELD ID="4" xsi:type="CharTerm" TERMINATOR="\r\n" MAX_LENGTH="256" COLLATION="Finnish_Swedish_CI_AS"/>
 </RECORD>
 <ROW>
  <COLUMN SOURCE="2" NAME="col1" xsi:type="SQLVARYCHAR"/>
  <COLUMN SOURCE="3" NAME="col2" xsi:type="SQLVARYCHAR"/>
  <COLUMN SOURCE="4" NAME="col3" xsi:type="SQLVARYCHAR"/>
 </ROW>
</BCPFORMAT>

Now we can bulk load the data file into our table with a select, thus having full controll over the columns, in this case by not inserting data into the identity column:

INSERT INTO target_table (col1,col2, col3)
SELECT * FROM  openrowset(
bulk 'C:\data_file.txt',
formatfile='C:\format_file.xml') as t;

Read from database and fill DataTable

Private Function LoaderData(ByVal strSql As String) As DataTable
    Dim cnn As SqlConnection
    Dim dad As SqlDataAdapter

    Dim dtb As New DataTable
    cnn = New SqlConnection(My.Settings.mySqlConnectionString)
    Try
        cnn.Open()
        dad = New SqlDataAdapter(strSql, cnn)
        dad.Fill(dtb)
        cnn.Close()
        dad.Dispose()
    Catch ex As Exception
        cnn.Close()
        MsgBox(ex.Message)
    End Try
    Return dtb
End Function

SQL Server Error : String or binary data would be truncated

This error is usually encountered when inserting a record in a table where one of the columns is a VARCHAR or CHAR data type and the length of the value being inserted is longer than the length of the column.

I am not satisfied how Microsoft decided to inform with this "dry" response message, without any point of where to look for the answer.

How To Create Table with Identity Column

Unique key allows max 2 NULL values. Explaination:

create table teppp
(
id int identity(1,1) primary key,
name varchar(10 )unique,
addresss varchar(10)
)

insert into teppp ( name,addresss) values ('','address1')
insert into teppp ( name,addresss) values ('NULL','address2')
insert into teppp ( addresss) values ('address3')

select * from teppp
null string , address1
NULL,address2
NULL,address3

If you try inserting same values as below:

insert into teppp ( name,addresss) values ('','address4')
insert into teppp ( name,addresss) values ('NULL','address5')
insert into teppp ( addresss) values ('address6')

Every time you will get error like:

Violation of UNIQUE KEY constraint 'UQ__teppp__72E12F1B2E1BDC42'. Cannot insert duplicate key in object 'dbo.teppp'.
The statement has been terminated.

How do I query for all dates greater than a certain date in SQL Server?

To sum it all up, the correct answer is :

select * from db where Date >= '20100401'  (Format of date yyyymmdd)

This will avoid any problem with other language systems and will use the index.

Passing data to a bootstrap modal

Try with this

$(function(){
 //when click a button
  $("#miButton").click(function(){
    $(".dynamic-field").remove();
    //pass the data in the modal body adding html elements
    $('#myModal .modal-body').html('<input type="hidden" name="name" value="your value" class="dynamic-field">') ;
    //open the modal
    $('#myModal').modal('show') 
  })
})

CASE (Contains) rather than equal statement

Pseudo code, something like:

CASE
  When CHARINDEX('lactulose', dbo.Table.Column) > 0 Then 'BP Medication'
ELSE ''
END AS 'Medication Type'

This does not care where the keyword is found in the list and avoids depending on formatting of spaces and commas.

How to convert image into byte array and byte array to base64 String in android?

I wrote the following code to convert an image from sdcard to a Base64 encoded string to send as a JSON object.And it works great:

String filepath = "/sdcard/temp.png";
File imagefile = new File(filepath);
FileInputStream fis = null;
try {
    fis = new FileInputStream(imagefile);
    } catch (FileNotFoundException e) {
    e.printStackTrace();
}

Bitmap bm = BitmapFactory.decodeStream(fis);
ByteArrayOutputStream baos = new ByteArrayOutputStream();  
bm.compress(Bitmap.CompressFormat.JPEG, 100 , baos);    
byte[] b = baos.toByteArray(); 
encImage = Base64.encodeToString(b, Base64.DEFAULT);

SQL Server : check if variable is Empty or NULL for WHERE clause

Just use

If @searchType is null means 'return the whole table' then use

WHERE p.[Type] = @SearchType OR @SearchType is NULL

If @searchType is an empty string means 'return the whole table' then use

WHERE p.[Type] = @SearchType OR @SearchType = ''

If @searchType is null or an empty string means 'return the whole table' then use

WHERE p.[Type] = @SearchType OR Coalesce(@SearchType,'') = ''

SQL Inner-join with 3 tables?

You just need a second inner join that links the ID Number that you have now to the ID Number of the third table. Afterwards, replace the ID Number by the Hall Name and voilá :)

Java :Add scroll into text area

My naive assumption was that the size of scroll pane will be determined automatically...

The only solution that actually worked for me was explicitly seeting bounds of JScrollPane:

import javax.swing.*;

public class MyFrame extends JFrame {

    public MyFrame()
    {
        setBounds(100, 100, 491, 310);
        getContentPane().setLayout(null);

        JTextArea textField = new JTextArea();
        textField.setEditable(false);

        String str = "";
        for (int i = 0; i < 50; ++i)
            str += "Some text\n";
        textField.setText(str);

        JScrollPane scroll = new JScrollPane(textField);
        scroll.setBounds(10, 11, 455, 249);                     // <-- THIS

        getContentPane().add(scroll);
        setLocationRelativeTo ( null );
    }
}

Maybe it will help some future visitors :)

Correct use of transactions in SQL Server

Add a try/catch block, if the transaction succeeds it will commit the changes, if the transaction fails the transaction is rolled back:

BEGIN TRANSACTION [Tran1]

  BEGIN TRY

      INSERT INTO [Test].[dbo].[T1] ([Title], [AVG])
      VALUES ('Tidd130', 130), ('Tidd230', 230)

      UPDATE [Test].[dbo].[T1]
      SET [Title] = N'az2' ,[AVG] = 1
      WHERE [dbo].[T1].[Title] = N'az'

      COMMIT TRANSACTION [Tran1]

  END TRY

  BEGIN CATCH

      ROLLBACK TRANSACTION [Tran1]

  END CATCH  

Improve SQL Server query performance on large tables

How is this possible? Without an index on the er101_upd_date_iso column how can a clustered index scan be used?

An index is a B-Tree where each leaf node is pointing to a 'bunch of rows'(called a 'Page' in SQL internal terminology), That is when the index is a non-clustered index.

Clustered index is a special case, in which the leaf nodes has the 'bunch of rows' (rather than pointing to them). that is why...

1) There can be only one clustered index on the table.

this also means the whole table is stored as the clustered index, that is why you started seeing index scan rather than a table scan.

2) An operation that utilizes clustered index is generally faster than a non-clustered index

Read more at http://msdn.microsoft.com/en-us/library/ms177443.aspx

For the problem you have, you should really consider adding this column to a index, as you said adding a new index (or a column to an existing index) increases INSERT/UPDATE costs. But it might be possible to remove some underutilized index (or a column from an existing index) to replace with 'er101_upd_date_iso'.

If index changes are not possible, i recommend adding a statistics on the column, it can fasten things up when the columns have some correlation with indexed columns

http://msdn.microsoft.com/en-us/library/ms188038.aspx

BTW, You will get much more help if you can post the table schema of ER101_ACCT_ORDER_DTL. and the existing indices too..., probably the query could be re-written to use some of them.

Why is SQL server throwing this error: Cannot insert the value NULL into column 'id'?

I had a similar problem and upon looking into it, it was simply a field in the actual table missing id (id was empty/null) - meaning when you try to make the id field the primary key it will result in error because the table contains a row with null value for the primary key.

This could be the fix if you see a temp table associated with the error. I was using SQL Server Management Studio.

SQL Server ON DELETE Trigger

INSERTED and DELETED are virtual tables. They need to be used in a FROM clause.

CREATE TRIGGER sampleTrigger
    ON database1.dbo.table1
    FOR DELETE
AS
    IF EXISTS (SELECT foo
               FROM database2.dbo.table2
               WHERE id IN (SELECT deleted.id FROM deleted)
               AND bar = 4)

Update Query with INNER JOIN between tables in 2 different databases on 1 server

Following is the MySQL syntax:

UPDATE table1 
INNER JOIN table2 ON table1.field1 = table2.field2
SET table1.field3 = table2.field4 
WHERE ...... ;

http://geekswithblogs.net/faizanahmad/archive/2009/01/05/join-in-sql-update--statement.aspx

Check if a parameter is null or empty in a stored procedure

I use coalesce:

IF ( COALESCE( @PreviousStartDate, '' ) = '' ) ...

Find duplicate records in a table using SQL Server

Try this instead

SELECT MAX(shoppername), COUNT(*) AS cnt
FROM dbo.sales
GROUP BY CHECKSUM(*)
HAVING COUNT(*) > 1

Read about the CHECKSUM function first, as there can be duplicates.

How to prevent colliders from passing through each other?

1.) Never use MESH COLLIDER. Use combination of box and capsule collider.

2.) Check constraints in RigidBody. If you tick Freeze Position X than it will pass through the object on the X axis. (Same for y axis).

The maximum recursion 100 has been exhausted before statement completion

Specify the maxrecursion option at the end of the query:

...
from EmployeeTree
option (maxrecursion 0)

That allows you to specify how often the CTE can recurse before generating an error. Maxrecursion 0 allows infinite recursion.

How to replace a character from a String in SQL?

Are you sure that the data stored in the database is actually a question mark? I would tend to suspect from the sample data that the problem is one of character set conversion where ? is being used as the replacement character when the character can't be represented in the client character set. Possibly, the database is actually storing Microsoft "smart quote" characters rather than simple apostrophes.

What does the DUMP function show is actually stored in the database?

SELECT column_name,
       dump(column_name,1016)
  FROM your_table
 WHERE <<predicate that returns just the sample data you posted>>

What application are you using to view the data? What is the client's NLS_LANG set to?

What is the database and national character set? Is the data stored in a VARCHAR2 column? Or NVARCHAR2?

SELECT parameter, value
  FROM v$nls_parameters
 WHERE parameter LIKE '%CHARACTERSET';

If all the problem characters are stored in the database as 0x19 (decimal 25), your REPLACE would need to be something like

UPDATE table_name
   SET column1 = REPLACE(column1, chr(25), q'[']'),
       column2 = REPLACE(column2, chr(25), q'[']'),
       ...
       columnN = REPLACE(columnN, chr(25), q'[']')
 WHERE INSTR(column1,chr(25)) > 0
    OR INSTR(column2,chr(25)) > 0 
    ...
    OR INSTR(columnN,chr(25)) > 0

Android check internet connection

No need to be complex. The simplest and framework manner is to use ACCESS_NETWORK_STATE permission and just make a connected method

public boolean isOnline() {
    ConnectivityManager cm = (ConnectivityManager) getSystemService(Context.CONNECTIVITY_SERVICE);
    return cm.getActiveNetworkInfo() != null && cm.getActiveNetworkInfo().isConnectedOrConnecting();
}

You can also use requestRouteToHost if you have a particualr host and connection type (wifi/mobile) in mind.

You will also need:

<uses-permission android:name="android.permission.ACCESS_NETWORK_STATE" />

in your android manifest.

for more detail go here

Formatting Numbers by padding with leading zeros in SQL Server

Just use the FORMAT function (works on SQL Server 2012 or newer):

SELECT FORMAT(EmployeeID, '000000')
FROM dbo.RequestItems
WHERE ID=0 

Reference: http://msdn.microsoft.com/en-us/library/hh213505.aspx

Use jQuery to change a second select list based on the first select list option

_x000D_
_x000D_
$("#select1").change(function() {_x000D_
  if ($(this).data('options') === undefined) {_x000D_
    /*Taking an array of all options-2 and kind of embedding it on the select1*/_x000D_
    $(this).data('options', $('#select2 option').clone());_x000D_
  }_x000D_
  var id = $(this).val();_x000D_
  var options = $(this).data('options').filter('[value=' + id + ']');_x000D_
  $('#select2').html(options);_x000D_
});
_x000D_
<script src="https://ajax.googleapis.com/ajax/libs/jquery/2.0.0/jquery.min.js"></script>_x000D_
<select name="select1" id="select1">_x000D_
  <option value="1">Fruit</option>_x000D_
  <option value="2">Animal</option>_x000D_
  <option value="3">Bird</option>_x000D_
  <option value="4">Car</option>_x000D_
</select>_x000D_
_x000D_
_x000D_
<select name="select2" id="select2">_x000D_
  <option value="1">Banana</option>_x000D_
  <option value="1">Apple</option>_x000D_
  <option value="1">Orange</option>_x000D_
  <option value="2">Wolf</option>_x000D_
  <option value="2">Fox</option>_x000D_
  <option value="2">Bear</option>_x000D_
  <option value="3">Eagle</option>_x000D_
  <option value="3">Hawk</option>_x000D_
  <option value="4">BWM<option>_x000D_
</select>
_x000D_
_x000D_
_x000D_

Using jQuery data() to store data

I guess hiding elements doesn't work cross-browser(2012), I have'nt tested it myself.

Docker container will automatically stop after "docker run -d"

You can accomplish what you want with either:

docker run -t -d <image-name>

or

docker run -i -d <image-name>

or

docker run -it -d <image-name>

The command parameter as suggested by other answers (i.e. tail -f /dev/null) is completely optional, and is NOT required to get your container to stay running in the background.

Also note the Docker documentation suggests that combining -i and -t options will cause it to behave like a shell.

See:

https://docs.docker.com/engine/reference/run/#foreground

SQLite - getting number of rows in a database

Not sure if I understand your question, but max(id) won't give you the number of lines at all. For example if you have only one line with id = 13 (let's say you deleted the previous lines), you'll have max(id) = 13 but the number of rows is 1. The correct (and fastest) solution is to use count(). BTW if you wonder why there's a star, it's because you can count lines based on a criteria.

Sublime Text 3 how to change the font size of the file sidebar?

On Ubuntu, for versions of Sublime older than 3.2, what worked for me was changing the dpi scale in Preferences > Settings — User by adding this line:

"dpi_scale": 1.10 

For Sublime 3.2, you can use the following line instead:

"ui_scale": 1.10

Adjust the scale value as needed. After this change, you have to restart Sublime Text for it to take effect.

What is the difference between dynamic programming and greedy approach?

I would like to cite a paragraph which describes the major difference between greedy algorithms and dynamic programming algorithms stated in the book Introduction to Algorithms (3rd edition) by Cormen, Chapter 15.3, page 381:

One major difference between greedy algorithms and dynamic programming is that instead of first finding optimal solutions to subproblems and then making an informed choice, greedy algorithms first make a greedy choice, the choice that looks best at the time, and then solve a resulting subproblem, without bothering to solve all possible related smaller subproblems.

How can I print a circular structure in a JSON-like format?

To update the answer of overriding the way JSON works (probably not recommended, but super simple), don't use circular-json (it's deprecated). Instead, use the successor, flatted:

https://www.npmjs.com/package/flatted

Borrowed from the old answer above from @user1541685 , but replaced with the new one:

npm i --save flatted

then in your js file

const CircularJSON = require('flatted');
const json = CircularJSON.stringify(obj);

How do I obtain the frequencies of each value in an FFT?

The first bin in the FFT is DC (0 Hz), the second bin is Fs / N, where Fs is the sample rate and N is the size of the FFT. The next bin is 2 * Fs / N. To express this in general terms, the nth bin is n * Fs / N.

So if your sample rate, Fs is say 44.1 kHz and your FFT size, N is 1024, then the FFT output bins are at:

  0:   0 * 44100 / 1024 =     0.0 Hz
  1:   1 * 44100 / 1024 =    43.1 Hz
  2:   2 * 44100 / 1024 =    86.1 Hz
  3:   3 * 44100 / 1024 =   129.2 Hz
  4: ...
  5: ...
     ...
511: 511 * 44100 / 1024 = 22006.9 Hz

Note that for a real input signal (imaginary parts all zero) the second half of the FFT (bins from N / 2 + 1 to N - 1) contain no useful additional information (they have complex conjugate symmetry with the first N / 2 - 1 bins). The last useful bin (for practical aplications) is at N / 2 - 1, which corresponds to 22006.9 Hz in the above example. The bin at N / 2 represents energy at the Nyquist frequency, i.e. Fs / 2 ( = 22050 Hz in this example), but this is in general not of any practical use, since anti-aliasing filters will typically attenuate any signals at and above Fs / 2.

Grant Select on a view not base table when base table is in a different database

I tried this in one of my databases.

To get it to work, the user had to be added to the database housing the actual data. No rights were needed, just access.

Have you considered keeping the view in the database it references? Re usability and all if its benefits could follow.

Is there an easy way to add a border to the top and bottom of an Android View?

Try wrapping the image with a linearlayout, and set it's background to the border color you want around the text. Then set the padding on the textview to be the thickness you want for your border.

Create SQL identity as primary key?

Simple change to syntax is all that is needed:

 create table ImagenesUsuario (
   idImagen int not null identity(1,1) primary key
 )

By explicitly using the "constraint" keyword, you can give the primary key constraint a particular name rather than depending on SQL Server to auto-assign a name:

 create table ImagenesUsuario (
   idImagen int not null identity(1,1) constraint pk_ImagenesUsario primary key
 )

Add the "CLUSTERED" keyword if that makes the most sense based on your use of the table (i.e., the balance of searches for a particular idImagen and amount of writing outweighs the benefits of clustering the table by some other index).

How to get a tab character?

Sure there's an entity for tabs:

&#9;

(The tab is ASCII character 9, or Unicode U+0009.)

However, just like literal tabs (ones you type in to your text editor), all tab characters are treated as whitespace by HTML parsers and collapsed into a single space except those within a <pre> block, where literal tabs will be rendered as 8 spaces in a monospace font.

How to display text in pygame?

Here is my answer:


    def draw_text(text, font_name, size, color, x, y, align="nw"):
        font = pg.font.Font(font_name, size)
        text_surface = font.render(text, True, color)
        text_rect = text_surface.get_rect()
        if align == "nw":
            text_rect.topleft = (x, y)
        if align == "ne":
            text_rect.topright = (x, y)
        if align == "sw":
            text_rect.bottomleft = (x, y)
        if align == "se":
            text_rect.bottomright = (x, y)
        if align == "n":
            text_rect.midtop = (x, y)
        if align == "s":
            text_rect.midbottom = (x, y)
        if align == "e":
            text_rect.midright = (x, y)
        if align == "w":
            text_rect.midleft = (x, y)
        if align == "center":
            text_rect.center = (x, y)
        screen.blit(text_surface, text_rect)

Of course, you'll need to import pygame, a font and a screen, but this is just a def to add on to the rest of the code, and then call "draw_text".

How can I detect when an Android application is running in the emulator?

Checking the answers, none of them worked when using LeapDroid, Droid4x or Andy emulators,

What does work for all cases is the following:

 private static String getSystemProperty(String name) throws Exception {
    Class systemPropertyClazz = Class.forName("android.os.SystemProperties");
    return (String) systemPropertyClazz.getMethod("get", new Class[]{String.class}).invoke(systemPropertyClazz, new Object[]{name});
}

public boolean isEmulator() {
    boolean goldfish = getSystemProperty("ro.hardware").contains("goldfish");
    boolean emu = getSystemProperty("ro.kernel.qemu").length() > 0;
    boolean sdk = getSystemProperty("ro.product.model").equals("sdk");
    return goldfish || emu || sdk;
}

Resizing SVG in html?

Use the following code:

<g transform="scale(0.1)">
...
</g>

How can I avoid Java code in JSP files, using JSP 2?

The use of scriptlets (those <% %> things) in JSP is indeed highly discouraged since the birth of taglibs (like JSTL) and EL (Expression Language, those ${} things) way back in 2001.

The major disadvantages of scriptlets are:

  1. Reusability: you can't reuse scriptlets.
  2. Replaceability: you can't make scriptlets abstract.
  3. OO-ability: you can't make use of inheritance/composition.
  4. Debuggability: if scriptlet throws an exception halfway, all you get is a blank page.
  5. Testability: scriptlets are not unit-testable.
  6. Maintainability: per saldo more time is needed to maintain mingled/cluttered/duplicated code logic.

Sun Oracle itself also recommends in the JSP coding conventions to avoid use of scriptlets whenever the same functionality is possible by (tag) classes. Here are several cites of relevance:

From JSP 1.2 Specification, it is highly recommended that the JSP Standard Tag Library (JSTL) be used in your web application to help reduce the need for JSP scriptlets in your pages. Pages that use JSTL are, in general, easier to read and maintain.

...

Where possible, avoid JSP scriptlets whenever tag libraries provide equivalent functionality. This makes pages easier to read and maintain, helps to separate business logic from presentation logic, and will make your pages easier to evolve into JSP 2.0-style pages (JSP 2.0 Specification supports but de-emphasizes the use of scriptlets).

...

In the spirit of adopting the model-view-controller (MVC) design pattern to reduce coupling between the presentation tier from the business logic, JSP scriptlets should not be used for writing business logic. Rather, JSP scriptlets are used if necessary to transform data (also called "value objects") returned from processing the client's requests into a proper client-ready format. Even then, this would be better done with a front controller servlet or a custom tag.


How to replace scriptlets entirely depends on the sole purpose of the code/logic. More than often this code is to be placed in a fullworthy Java class:

  • If you want to invoke the same Java code on every request, less-or-more regardless of the requested page, e.g. checking if a user is logged in, then implement a filter and write code accordingly in doFilter() method. E.g.:

    public void doFilter(ServletRequest request, ServletResponse response, FilterChain chain) throws ServletException, IOException {
        if (((HttpServletRequest) request).getSession().getAttribute("user") == null) {
            ((HttpServletResponse) response).sendRedirect("login"); // Not logged in, redirect to login page.
        } else {
            chain.doFilter(request, response); // Logged in, just continue request.
        }
    }
    

    When mapped on an appropriate <url-pattern> covering the JSP pages of interest, then you don't need to copypaste the same piece of code overall JSP pages.


  • If you want to invoke some Java code to preprocess a request, e.g. preloading some list from a database to display in some table, if necessary based on some query parameters, then implement a servlet and write code accordingly in doGet() method. E.g.:

    protected void doGet(HttpServletRequest request, HttpServletResponse response) throws ServletException, IOException {
        try {
            List<Product> products = productService.list(); // Obtain all products.
            request.setAttribute("products", products); // Store products in request scope.
            request.getRequestDispatcher("/WEB-INF/products.jsp").forward(request, response); // Forward to JSP page to display them in a HTML table.
        } catch (SQLException e) {
            throw new ServletException("Retrieving products failed!", e);
        }
    }
    

    This way dealing with exceptions is easier. The DB is not accessed in the midst of JSP rendering, but far before the JSP is been displayed. You still have the possibility to change the response whenever the DB access throws an exception. In the above example, the default error 500 page will be displayed which you can anyway customize by an <error-page> in web.xml.


  • If you want to invoke some Java code to postprocess a request, e.g. processing a form submit, then implement a servlet and write code accordingly in doPost() method. E.g.:

    protected void doPost(HttpServletRequest request, HttpServletResponse response) throws ServletException, IOException {
        String username = request.getParameter("username");
        String password = request.getParameter("password");
        User user = userService.find(username, password);
    
        if (user != null) {
            request.getSession().setAttribute("user", user); // Login user.
            response.sendRedirect("home"); // Redirect to home page.
        } else {
            request.setAttribute("message", "Unknown username/password. Please retry."); // Store error message in request scope.
            request.getRequestDispatcher("/WEB-INF/login.jsp").forward(request, response); // Forward to JSP page to redisplay login form with error.
        }
    }
    

    This way dealing with different result page destinations is easier: redisplaying the form with validation errors in case of an error (in this particular example you can redisplay it using ${message} in EL), or just taking to the desired target page in case of success.


  • If you want to invoke some Java code to control the execution plan and/or the destination of the request and the response, then implement a servlet according to the MVC's Front Controller Pattern. E.g.:

    protected void service(HttpServletRequest request, HttpServletResponse response) throws ServletException, IOException {
        try {
            Action action = ActionFactory.getAction(request);
            String view = action.execute(request, response);
    
            if (view.equals(request.getPathInfo().substring(1)) {
                request.getRequestDispatcher("/WEB-INF/" + view + ".jsp").forward(request, response);
            } else {
                response.sendRedirect(view);
            }
        } catch (Exception e) {
            throw new ServletException("Executing action failed.", e);
        }
    }
    

    Or just adopt an MVC framework like JSF, Spring MVC, Wicket, etc so that you end up with just a JSP/Facelets page and a JavaBean class without the need for a custom servlet.


  • If you want to invoke some Java code to control the flow inside a JSP page, then you need to grab an (existing) flow control taglib like JSTL core. E.g. displaying List<Product> in a table:

    <%@ taglib uri="http://java.sun.com/jsp/jstl/core" prefix="c" %>
    ...
    <table>
        <c:forEach items="${products}" var="product">
            <tr>
                <td>${product.name}</td>
                <td>${product.description}</td>
                <td>${product.price}</td>
            </tr>
        </c:forEach>
    </table>
    

    With XML-style tags which fit nicely among all that HTML, the code is better readable (and thus better maintainable) than a bunch of scriptlets with various opening and closing braces ("Where the heck does this closing brace belong to?"). An easy aid is to configure your web application to throw an exception whenever scriptlets are still been used by adding the following piece to web.xml:

    <jsp-config>
        <jsp-property-group>
            <url-pattern>*.jsp</url-pattern>
            <scripting-invalid>true</scripting-invalid>
        </jsp-property-group>
    </jsp-config>
    

    In Facelets, the successor of JSP, which is part of the Java EE provided MVC framework JSF, it is already not possible to use scriptlets. This way you're automatically forced to do things "the right way".


  • If you want to invoke some Java code to access and display "backend" data inside a JSP page, then you need to use EL (Expression Language), those ${} things. E.g. redisplaying submitted input values:

    <input type="text" name="foo" value="${param.foo}" />
    

    The ${param.foo} displays the outcome of request.getParameter("foo").


  • If you want to invoke some utility Java code directly in the JSP page (typically public static methods), then you need to define them as EL functions. There's a standard functions taglib in JSTL, but you can also easily create functions yourself. Here's an example how JSTL fn:escapeXml is useful to prevent XSS attacks.

    <%@ taglib uri="http://java.sun.com/jsp/jstl/functions" prefix="fn" %>
    ...
    <input type="text" name="foo" value="${fn:escapeXml(param.foo)}" />
    

    Note that the XSS sensitivity is in no way specifically related to Java/JSP/JSTL/EL/whatever, this problem needs to be taken into account in every web application you develop. The problem of scriptlets is that it provides no way of builtin preventions, at least not using the standard Java API. JSP's successor Facelets has already implicit HTML escaping, so you don't need to worry about XSS holes in Facelets.

See also:

Changing factor levels with dplyr mutate

You can use the recode function from dplyr.

df <- iris %>%
     mutate(Species = recode(Species, setosa = "SETOSA",
         versicolor = "VERSICOLOR",
         virginica = "VIRGINICA"
     )
)

How does createOrReplaceTempView work in Spark?

SparkSQl support writing programs using Dataset and Dataframe API, along with it need to support sql.

In order to support Sql on DataFrames, first it requires a table definition with column names are required, along with if it creates tables the hive metastore will get lot unnecessary tables, because Spark-Sql natively resides on hive. So it will create a temporary view, which temporarily available in hive for time being and used as any other hive table, once the Spark Context stop it will be removed.

In order to create the view, developer need an utility called createOrReplaceTempView

Python and pip, list all versions of a package that's available?

Update:
As of Sep 2017 this method no longer works: --no-install was removed in pip 7

Use pip install -v, you can see all versions that available

root@node7:~# pip install web.py -v
Downloading/unpacking web.py
  Using version 0.37 (newest of versions: 0.37, 0.36, 0.35, 0.34, 0.33, 0.33, 0.32, 0.31, 0.22, 0.2)
  Downloading web.py-0.37.tar.gz (90Kb): 90Kb downloaded
  Running setup.py egg_info for package web.py
    running egg_info
    creating pip-egg-info/web.py.egg-info

To not install any package, use one of following solution:

root@node7:~# pip install --no-deps --no-install flask -v                                                                                                      
Downloading/unpacking flask
  Using version 0.10.1 (newest of versions: 0.10.1, 0.10, 0.9, 0.8.1, 0.8, 0.7.2, 0.7.1, 0.7, 0.6.1, 0.6, 0.5.2, 0.5.1, 0.5, 0.4, 0.3.1, 0.3, 0.2, 0.1)
  Downloading Flask-0.10.1.tar.gz (544Kb): 544Kb downloaded

or

root@node7:~# cd $(mktemp -d)
root@node7:/tmp/tmp.c6H99cWD0g# pip install flask -d . -v
Downloading/unpacking flask
  Using version 0.10.1 (newest of versions: 0.10.1, 0.10, 0.9, 0.8.1, 0.8, 0.7.2, 0.7.1, 0.7, 0.6.1, 0.6, 0.5.2, 0.5.1, 0.5, 0.4, 0.3.1, 0.3, 0.2, 0.1)
  Downloading Flask-0.10.1.tar.gz (544Kb): 4.1Kb downloaded

Tested with pip 1.0

root@node7:~# pip --version
pip 1.0 from /usr/lib/python2.7/dist-packages (python 2.7)

How can I remove the top and right axis in matplotlib?

[edit] matplotlib in now (2013-10) on version 1.3.0 which includes this

That ability was actually just added, and you need the Subversion version for it. You can see the example code here.

I am just updating to say that there's a better example online now. Still need the Subversion version though, there hasn't been a release with this yet.

[edit] Matplotlib 0.99.0 RC1 was just released, and includes this capability.

checked = "checked" vs checked = true

The element has both an attribute and a property named checked. The property determines the current state.

The attribute is a string, and the property is a boolean. When the element is created from the HTML code, the attribute is set from the markup, and the property is set depending on the value of the attribute.

If there is no value for the attribute in the markup, the attribute becomes null, but the property is always either true or false, so it becomes false.

When you set the property, you should use a boolean value:

document.getElementById('myRadio').checked = true;

If you set the attribute, you use a string:

document.getElementById('myRadio').setAttribute('checked', 'checked');

Note that setting the attribute also changes the property, but setting the property doesn't change the attribute.

Note also that whatever value you set the attribute to, the property becomes true. Even if you use an empty string or null, setting the attribute means that it's checked. Use removeAttribute to uncheck the element using the attribute:

document.getElementById('myRadio').removeAttribute('checked');

How do I find a stored procedure containing <text>?

For any SQL server newer than SQL server 2000:

SELECT object_name = OBJECT_NAME(sm.object_id), o.type_desc, sm.definition  
FROM sys.sql_modules AS sm  
JOIN sys.objects AS o ON sm.object_id = o.object_id  
WHERE sm.definition like '%searchString%'  
ORDER BY  o.type, o.name, o.object_id

If someone is stuck with SQL server 2000, the table sql_modules doesn't exist, so you would use syscomments, you will get multiple records for stored procdedures larger than 4000 characters, but they will have the same c.number field so you can group the parts together to get the full stored procedure text:

    Select o.id, c.number, o.name, c.text  
    from syscomments c 
    inner join sysobjects o on o.id = c.id 
    where c.encrypted = 0 and o.type = 'P'  
      and c.id in  
     (Select id from syscomments where text like '%searchtext%')
    order by objecttype, o.name, o.id, c.number, c.colid        

Memcached vs. Redis?

Test. Run some simple benchmarks. For a long while I considered myself an old school rhino since I used mostly memcached and considered Redis the new kid.

With my current company Redis was used as the main cache. When I dug into some performance stats and simply started testing, Redis was, in terms of performance, comparable or minimally slower than MySQL.

Memcached, though simplistic, blew Redis out of water totally. It scaled much better:

  • for bigger values (required change in slab size, but worked)
  • for multiple concurrent requests

Also, memcached eviction policy is in my view, much better implemented, resulting in overall more stable average response time while handling more data than the cache can handle.

Some benchmarking revealed that Redis, in our case, performs very poorly. This I believe has to do with many variables:

  • type of hardware you run Redis on
  • types of data you store
  • amount of gets and sets
  • how concurrent your app is
  • do you need data structure storage

Personally, I don't share the view Redis authors have on concurrency and multithreading.

What's the best practice to round a float to 2 decimals?

Here is a shorter implementation comparing to @Jav_Rock's

   /**
     * Round to certain number of decimals
     * 
     * @param d
     * @param decimalPlace the numbers of decimals
     * @return
     */

    public static float round(float d, int decimalPlace) {
         return BigDecimal.valueOf(d).setScale(decimalPlace,BigDecimal.ROUND_HALF_UP).floatValue();
    }



    System.out.println(round(2.345f,2));//two decimal digits, //2.35

Finding diff between current and last version

Firstly, use "git log" to list the logs for the repository.

Now, select the two commit IDs, pertaining to the two commits. You want to see the differences (example - Top most commit and some older commit (as per your expectation of current-version and some old version)).

Next, use:

git diff <commit_id1> <commit_id2>

or

git difftool <commit_id1> <commit_id2>

How to change the font and font size of an HTML input tag?

in your css :

 #txtComputer {
      font-size: 24px;
 }

You can style an input entirely (background, color, etc.) and even use the hover event.

How to download/checkout a project from Google Code in Windows?

If you have a github account and don't want to download software, you can export to github, then download a zip from github.

Should I use 'border: none' or 'border: 0'?

This is the result in Firefox 78.0.2 (64-Bit):

img {
    border: none;
        border-top-color: currentcolor;
        border-top-style: none;
        border-top-width: medium;
        border-right-color: currentcolor;
        border-right-style: none;
        border-right-width: medium;
        border-bottom-color: currentcolor;
        border-bottom-style: none;
        border-bottom-width: medium;
        border-left-color: currentcolor;
        border-left-style: none;
        border-left-width: medium;
}

img {
    border: 0;
        border-top-color: currentcolor;
        border-top-style: none;
        border-top-width: 0px;
        border-right-color: currentcolor;
        border-right-style: none;
        border-right-width: 0px;
        border-bottom-color: currentcolor;
        border-bottom-style: none;
        border-bottom-width: 0px;
        border-left-color: currentcolor;
        border-left-style: none;
        border-left-width: 0px;
        border-image-outset: 0;
        border-image-repeat: stretch;
        border-image-slice: 100%;
        border-image-source: none;
        border-image-width: 1;
}

Date: 20200720

How to make a great R reproducible example

I have a very easy and efficient way to make a R example that has not been mentioned above. You can define your structure firstly. For example,

mydata <- data.frame(a=character(0), b=numeric(0),  c=numeric(0), d=numeric(0))

>fix(mydata)

When you execute 'fix' command, you will get this pop-up box

Then you can input your data manually. This is efficient for smaller examples rather than big ones.

Replacing a fragment with another fragment inside activity group

Use the below code in android.support.v4

FragmentTransaction ft1 = getFragmentManager().beginTransaction();
WebViewFragment w1 = new WebViewFragment();
w1.init(linkData.getLink());
ft1.addToBackStack(linkData.getName());
ft1.replace(R.id.listFragment, w1);
ft1.commit();

Server.Mappath in C# classlibrary

You can get the base path by using the following code and append your needed path with that.

string  path = System.AppDomain.CurrentDomain.BaseDirectory;

How to find longest string in the table column data

Instead of SELECT max(len(CR)) AS Max_Length_String FROM table1

Use

SELECT (CR) FROM table1

WHERE len(CR) = (SELECT max(len(CR)) FROM table1)

A Simple AJAX with JSP example

I have used jQuery AJAX to make AJAX requests.

Check the following code:

<html>
<head>
    <script type="text/javascript" src="http://ajax.googleapis.com/ajax/libs/jquery/1.4/jquery.min.js"></script>
    <script type="text/javascript">
        $(document).ready(function() {
            $('#call').click(function ()
            {
                $.ajax({
                    type: "post",
                    url: "testme", //this is my servlet
                    data: "input=" +$('#ip').val()+"&output="+$('#op').val(),
                    success: function(msg){      
                            $('#output').append(msg);
                    }
                });
            });

        });
    </script>
    <meta http-equiv="Content-Type" content="text/html; charset=UTF-8">
    <title>JSP Page</title>
</head>
<body>
    input:<input id="ip" type="text" name="" value="" /><br></br>
    output:<input id="op" type="text" name="" value="" /><br></br>
    <input type="button" value="Call Servlet" name="Call Servlet" id="call"/>
    <div id="output"></div>
</body>

Graphical user interface Tutorial in C

My favourite UI tutorials all come from zetcode.com:

These are tutorials I'd consider to be "starting tutorials". The example tutorial gets you up and going, but doesn't show you anything too advanced or give much explanation. Still, often, I find the big problem is "how do I start?" and these have always proved useful to me.

What does ON [PRIMARY] mean?

When you create a database in Microsoft SQL Server you can have multiple file groups, where storage is created in multiple places, directories or disks. Each file group can be named. The PRIMARY file group is the default one, which is always created, and so the SQL you've given creates your table ON the PRIMARY file group.

See MSDN for the full syntax.

Convert objective-c typedef to its string equivalent

This is really a C question, not specific to Objective-C (which is a superset of the C language). Enums in C are represented as integers. So you need to write a function that returns a string given an enum value. There are many ways to do this. An array of strings such that the enum value can be used as an index into the array or a map structure (e.g. an NSDictionary) that maps an enum value to a string work, but I find that these approaches are not as clear as a function that makes the conversion explicit (and the array approach, although the classic C way is dangerous if your enum values are not continguous from 0). Something like this would work:

- (NSString*)formatTypeToString:(FormatType)formatType {
    NSString *result = nil;

    switch(formatType) {
        case JSON:
            result = @"JSON";
            break;
        case XML:
            result = @"XML";
            break;
        case Atom:
            result = @"Atom";
            break;
        case RSS:
            result = @"RSS";
            break;
        default:
            [NSException raise:NSGenericException format:@"Unexpected FormatType."];
    }

    return result;
}

Your related question about the correct syntax for an enum value is that you use just the value (e.g. JSON), not the FormatType.JSON sytax. FormatType is a type and the enum values (e.g. JSON, XML, etc.) are values that you can assign to that type.

ObservableCollection not noticing when Item in it changes (even with INotifyPropertyChanged)

Here's an extension method for the above solution...

public static TrulyObservableCollection<T> ToTrulyObservableCollection<T>(this List<T> list)
     where T : INotifyPropertyChanged
{
    var newList = new TrulyObservableCollection<T>();

    if (list != null)
    {
        list.ForEach(o => newList.Add(o));
    }

    return newList;
}  

How to install multiple python packages at once using pip

give the same command as you used to give while installing a single module only pass it via space delimited format

how to set default main class in java?

you can right click on the project select "set configuration" then "Customize", from there you can choose your main class. ScreenShot

Windows command prompt log to a file

First method

For Windows 7 and above users, Windows PowerShell give you this option. Users with windows version less than 7 can download PowerShell online and install it.

Steps:

  1. type PowerShell in search area and click on "Windows PowerShell"

  2. If you have a .bat (batch) file go to step 3 OR copy your commands to a file and save it with .bat extension (e.g. file.bat)

  3. run the .bat file with following command

    PS (location)> <path to bat file>/file.bat | Tee-Object -file log.txt

This will generate a log.txt file with all command prompt output in it. Advantage is that you can also the output on command prompt.

Second method

You can use file redirection (>, >>) as suggest by Bali C above.

I will recommend first method if you have lots of commands to run or a script to run. I will recommend last method if there is only few commands to run.

GDB: Listing all mapped memory regions for a crashed process

You can also use info files to list all the sections of all the binaries loaded in process binary.

SpringApplication.run main method

You need to run Application.run() because this method starts whole Spring Framework. Code below integrates your main() with Spring Boot.

Application.java

@SpringBootApplication
public class Application {

    public static void main(String[] args) {
        SpringApplication.run(Application.class, args);
    }
}

ReconTool.java

@Component
public class ReconTool implements CommandLineRunner {

    @Override
    public void run(String... args) throws Exception {
        main(args);
    }

    public static void main(String[] args) {
        // Recon Logic
    }
}

Why not SpringApplication.run(ReconTool.class, args)

Because this way spring is not fully configured (no component scan etc.). Only bean defined in run() is created (ReconTool).

Example project: https://github.com/mariuszs/spring-run-magic

ASP.NET MVC 3 Razor: Include JavaScript file in the head tag

You can use Named Sections.

_Layout.cshtml

<head>
    <script type="text/javascript" src="@Url.Content("/Scripts/jquery-1.6.2.min.js")"></script>
    @RenderSection("JavaScript", required: false)
</head>

_SomeView.cshtml

@section JavaScript
{
   <script type="text/javascript" src="@Url.Content("/Scripts/SomeScript.js")"></script>
   <script type="text/javascript" src="@Url.Content("/Scripts/AnotherScript.js")"></script>
}

Is returning out of a switch statement considered a better practice than using break?

A break will allow you continue processing in the function. Just returning out of the switch is fine if that's all you want to do in the function.

Youtube - How to force 480p video quality in embed link / <iframe>

You can use the YouTube JavaScript player API, which has a feature on its own to set playback quality.

player.setPlaybackQuality(suggestedQuality:String):Void

This function sets the suggested video quality for the current video. The function causes the video to reload at its current position in the new quality. If the playback quality does change, it will only change for the video being played. Calling this function does not guarantee that the playback quality will actually change. However, if the playback quality does change, the onPlaybackQualityChange event will fire, and your code should respond to the event rather than the fact that it called the setPlaybackQuality function. [source]

What is the difference between i++ & ++i in a for loop?

Both i++ and ++i are short-hand for i = i + 1.

In addition to changing the value of i, they also return the value of i, either before adding one (i++) or after adding one (++i).

In a loop the third component is a piece of code that is executed after each iteration.

for (int i=0; i<10; i++)

The value of that part is not used, so the above is just the same as

for(int i=0; i<10; i = i+1)

or

for(int i=0; i<10; ++i)

Where it makes a difference (between i++ and ++i )is in these cases

while(i++ < 10)

for (int i=0; i++ < 10; )

if else condition in blade file (laravel 5.3)

I think you are putting one too many curly brackets. Try this

 @if($user->status=='waiting')
            <td><a href="#" class="viewPopLink btn btn-default1" role="button" data-id="{!! $user->travel_id !!}" data-toggle="modal" data-target="#myModal">Approve/Reject</a> </td>
            @else
            <td>{!! $user->status !!}</td>
        @endif

In plain English, what does "git reset" do?

In general, git reset's function is to take the current branch and reset it to point somewhere else, and possibly bring the index and work tree along. More concretely, if your master branch (currently checked out) is like this:

- A - B - C (HEAD, master)

and you realize you want master to point to B, not C, you will use git reset B to move it there:

- A - B (HEAD, master)      # - C is still here, but there's no branch pointing to it anymore

Digression: This is different from a checkout. If you'd run git checkout B, you'd get this:

- A - B (HEAD) - C (master)

You've ended up in a detached HEAD state. HEAD, work tree, index all match B, but the master branch was left behind at C. If you make a new commit D at this point, you'll get this, which is probably not what you want:

- A - B - C (master)
       \
        D (HEAD)

Remember, reset doesn't make commits, it just updates a branch (which is a pointer to a commit) to point to a different commit. The rest is just details of what happens to your index and work tree.

Use cases

I cover many of the main use cases for git reset within my descriptions of the various options in the next section. It can really be used for a wide variety of things; the common thread is that all of them involve resetting the branch, index, and/or work tree to point to/match a given commit.

Things to be careful of

  • --hard can cause you to really lose work. It modifies your work tree.

  • git reset [options] commit can cause you to (sort of) lose commits. In the toy example above, we lost commit C. It's still in the repo, and you can find it by looking at git reflog show HEAD or git reflog show master, but it's not actually accessible from any branch anymore.

  • Git permanently deletes such commits after 30 days, but until then you can recover C by pointing a branch at it again (git checkout C; git branch <new branch name>).

Arguments

Paraphrasing the man page, most common usage is of the form git reset [<commit>] [paths...], which will reset the given paths to their state from the given commit. If the paths aren't provided, the entire tree is reset, and if the commit isn't provided, it's taken to be HEAD (the current commit). This is a common pattern across git commands (e.g. checkout, diff, log, though the exact semantics vary), so it shouldn't be too surprising.

For example, git reset other-branch path/to/foo resets everything in path/to/foo to its state in other-branch, git reset -- . resets the current directory to its state in HEAD, and a simple git reset resets everything to its state in HEAD.

The main work tree and index options

There are four main options to control what happens to your work tree and index during the reset.

Remember, the index is git's "staging area" - it's where things go when you say git add in preparation to commit.

  • --hard makes everything match the commit you've reset to. This is the easiest to understand, probably. All of your local changes get clobbered. One primary use is blowing away your work but not switching commits: git reset --hard means git reset --hard HEAD, i.e. don't change the branch but get rid of all local changes. The other is simply moving a branch from one place to another, and keeping index/work tree in sync. This is the one that can really make you lose work, because it modifies your work tree. Be very very sure you want to throw away local work before you run any reset --hard.

  • --mixed is the default, i.e. git reset means git reset --mixed. It resets the index, but not the work tree. This means all your files are intact, but any differences between the original commit and the one you reset to will show up as local modifications (or untracked files) with git status. Use this when you realize you made some bad commits, but you want to keep all the work you've done so you can fix it up and recommit. In order to commit, you'll have to add files to the index again (git add ...).

  • --soft doesn't touch the index or work tree. All your files are intact as with --mixed, but all the changes show up as changes to be committed with git status (i.e. checked in in preparation for committing). Use this when you realize you've made some bad commits, but the work's all good - all you need to do is recommit it differently. The index is untouched, so you can commit immediately if you want - the resulting commit will have all the same content as where you were before you reset.

  • --merge was added recently, and is intended to help you abort a failed merge. This is necessary because git merge will actually let you attempt a merge with a dirty work tree (one with local modifications) as long as those modifications are in files unaffected by the merge. git reset --merge resets the index (like --mixed - all changes show up as local modifications), and resets the files affected by the merge, but leaves the others alone. This will hopefully restore everything to how it was before the bad merge. You'll usually use it as git reset --merge (meaning git reset --merge HEAD) because you only want to reset away the merge, not actually move the branch. (HEAD hasn't been updated yet, since the merge failed)

    To be more concrete, suppose you've modified files A and B, and you attempt to merge in a branch which modified files C and D. The merge fails for some reason, and you decide to abort it. You use git reset --merge. It brings C and D back to how they were in HEAD, but leaves your modifications to A and B alone, since they weren't part of the attempted merge.

Want to know more?

I do think man git reset is really quite good for this - perhaps you do need a bit of a sense of the way git works for them to really sink in though. In particular, if you take the time to carefully read them, those tables detailing states of files in index and work tree for all the various options and cases are very very helpful. (But yes, they're very dense - they're conveying an awful lot of the above information in a very concise form.)

Strange notation

The "strange notation" (HEAD^ and HEAD~1) you mention is simply a shorthand for specifying commits, without having to use a hash name like 3ebe3f6. It's fully documented in the "specifying revisions" section of the man page for git-rev-parse, with lots of examples and related syntax. The caret and the tilde actually mean different things:

  • HEAD~ is short for HEAD~1 and means the commit's first parent. HEAD~2 means the commit's first parent's first parent. Think of HEAD~n as "n commits before HEAD" or "the nth generation ancestor of HEAD".
  • HEAD^ (or HEAD^1) also means the commit's first parent. HEAD^2 means the commit's second parent. Remember, a normal merge commit has two parents - the first parent is the merged-into commit, and the second parent is the commit that was merged. In general, merges can actually have arbitrarily many parents (octopus merges).
  • The ^ and ~ operators can be strung together, as in HEAD~3^2, the second parent of the third-generation ancestor of HEAD, HEAD^^2, the second parent of the first parent of HEAD, or even HEAD^^^, which is equivalent to HEAD~3.

caret and tilde

Why is there extra padding at the top of my UITableView with style UITableViewStyleGrouped in iOS7

Try changing the contentInset property that UITableView inherits from UIScrollView.

self.tableView.contentInset = UIEdgeInsetsMake(-20, 0, 0, 0);

It's a workaround, but it works

JPQL SELECT between date statement

public List<Student> findStudentByReports(Date startDate, Date endDate) {
    System.out.println("call findStudentMethd******************with this pattern"
                    + startDate
                    + endDate
                    + "*********************************************");

    return em
            .createQuery(
                    "' select attendence from Attendence attendence where attendence.admissionDate BETWEEN : startDate '' AND endDate ''"
                            + "'")
            .setParameter("startDate", startDate, TemporalType.DATE)
            .setParameter("endDate", endDate, TemporalType.DATE)
            .getResultList();

}

PHP case-insensitive in_array function

$a = [1 => 'funny', 3 => 'meshgaat', 15 => 'obi', 2 => 'OMER'];  

$b = 'omer';

function checkArr($x,$array)
{
    $arr = array_values($array);
    $arrlength = count($arr);
    $z = strtolower($x);

    for ($i = 0; $i < $arrlength; $i++) {
        if ($z == strtolower($arr[$i])) {
            echo "yes";
        }  
    } 
};

checkArr($b, $a);

Getting an element from a Set

Quick helper method that might address this situation:

<T> T onlyItem(Collection<T> items) {
    if (items.size() != 1)
        throw new IllegalArgumentException("Collection must have single item; instead it has " + items.size());

    return items.iterator().next();
}

Hive load CSV with commas in quoted fields

If you can re-create or parse your input data, you can specify an escape character for the CREATE TABLE:

ROW FORMAT DELIMITED FIELDS TERMINATED BY "," ESCAPED BY '\\';

Will accept this line as 4 fields

1,some text\, with comma in it,123,more text

Splitting string into multiple rows in Oracle

I'd like to add another method. This one uses recursive querys, something I haven't seen in the other answers. It is supported by Oracle since 11gR2.

with cte0 as (
    select phone_number x
    from hr.employees
), cte1(xstr,xrest,xremoved) as (
        select x, x, null
        from cte0
    union all        
        select xstr,
            case when instr(xrest,'.') = 0 then null else substr(xrest,instr(xrest,'.')+1) end,
            case when instr(xrest,'.') = 0 then xrest else substr(xrest,1,instr(xrest,'.') - 1) end
        from cte1
        where xrest is not null
)
select xstr, xremoved from cte1  
where xremoved is not null
order by xstr

It is quite flexible with the splitting character. Simply change it in the INSTR calls.

How can I install packages using pip according to the requirements.txt file from a local directory?

This works for everyone:

pip install -r /path/to/requirements.txt

What does "Git push non-fast-forward updates were rejected" mean?

you might want to use force with push operation in this case

git push origin master --force

Putting GridView data in a DataTable

protected void btnExportExcel_Click(object sender, EventArgs e)
{
    DataTable _datatable = new DataTable();
    for (int i = 0; i < grdReport.Columns.Count; i++)
    {
        _datatable.Columns.Add(grdReport.Columns[i].ToString());
    }
    foreach (GridViewRow row in grdReport.Rows)
    {
        DataRow dr = _datatable.NewRow();
        for (int j = 0; j < grdReport.Columns.Count; j++)
        {
            if (!row.Cells[j].Text.Equals("&nbsp;"))
                dr[grdReport.Columns[j].ToString()] = row.Cells[j].Text;
        }

        _datatable.Rows.Add(dr);
    }
    ExportDataTableToExcel(_datatable);
}

Reference an Element in a List of Tuples

You also can use itemgetter operator:

from operator import itemgetter
my_tuples = [('c','r'), (2, 3), ('e'), (True, False),('text','sample')]
map(itemgetter(0), my_tuples)

MySQL Foreign Key Error 1005 errno 150 primary key as foreign key

It's not your specific case, but it's worth noting for anybody else that this error can occur if you try to reference some fields in a table that are not the whole primary key of that table. Obviously this is not allowed.

Finding the max/min value in an array of primitives using Java

You can simply use the new Java 8 Streams but you have to work with int.

The stream method of the utility class Arrays gives you an IntStream on which you can use the min method. You can also do max, sum, average,...

The getAsInt method is used to get the value from the OptionalInt

import java.util.Arrays;

public class Test {
    public static void main(String[] args){
        int[] tab = {12, 1, 21, 8};
        int min = Arrays.stream(tab).min().getAsInt();
        int max = Arrays.stream(tab).max().getAsInt();
        System.out.println("Min = " + min);
        System.out.println("Max = " + max)
    }

}

==UPDATE==

If execution time is important and you want to go through the data only once you can use the summaryStatistics() method like this

import java.util.Arrays;
import java.util.IntSummaryStatistics;

public class SOTest {
    public static void main(String[] args){
        int[] tab = {12, 1, 21, 8};
        IntSummaryStatistics stat = Arrays.stream(tab).summaryStatistics();
        int min = stat.getMin();
        int max = stat.getMax();
        System.out.println("Min = " + min);
        System.out.println("Max = " + max);
    }
}

This approach can give better performance than classical loop because the summaryStatistics method is a reduction operation and it allows parallelization.

Warning: Null value is eliminated by an aggregate or other SET operation in Aqua Data Studio

Use ISNULL(field, 0) It can also be used with aggregates:

ISNULL(count(field), 0)

However, you might consider changing count(field) to count(*)

Edit:

try:

closedcases = ISNULL(
   (select count(closed) from ticket       
    where assigned_to = c.user_id and closed is not null       
    group by assigned_to), 0), 

opencases = ISNULL(
    (select count(closed) from ticket 
     where assigned_to = c.user_id and closed is null 
     group by assigned_to), 0),

Removing highcharts.com credits link

You can customise the credits, changing the URL, text, Position etc. All the info is documented here: http://api.highcharts.com/highcharts/credits. To simply disable them altogether, use:

credits: {
    enabled: false
},

How to scroll to an element in jQuery?

Like @user293153 I only just discovered this question and it didn't seem to be answered correctly.

His answer was best. But you can also animate to the element as well.

$('html, body').animate({ scrollTop: $("#some_element").offset().top }, 500);

How can we stop a running java process through Windows cmd?

In case you want to kill not all java processes but specif jars running. It will work for multiple jars as well.

wmic Path win32_process Where "CommandLine Like '%YourJarName.jar%'" Call Terminate

Else taskkill /im java.exe will work to kill all java processes

Python lookup hostname from IP with 1 second timeout

>>> import socket
>>> socket.gethostbyaddr("69.59.196.211")
('stackoverflow.com', ['211.196.59.69.in-addr.arpa'], ['69.59.196.211'])

For implementing the timeout on the function, this stackoverflow thread has answers on that.

DELETE_FAILED_INTERNAL_ERROR Error while Installing APK

Try this: Go to File> invalidate Caches/Restart then click on Invalidate and Restart button from the pop up. Now, try running your project.

Subscript out of bounds - general definition and solution?

If this helps anybody, I encountered this while using purr::map() with a function I wrote which was something like this:

find_nearby_shops <- function(base_account) {
   states_table %>% 
        filter(state == base_account$state) %>% 
        left_join(target_locations, by = c('border_states' = 'state')) %>% 
        mutate(x_latitude = base_account$latitude,
               x_longitude = base_account$longitude) %>% 
        mutate(dist_miles = geosphere::distHaversine(p1 = cbind(longitude, latitude), 
                                                     p2 = cbind(x_longitude, x_latitude))/1609.344)
}

nearby_shop_numbers <- base_locations %>% 
    split(f = base_locations$id) %>% 
    purrr::map_df(find_nearby_shops) 

I would get this error sometimes with samples, but most times I wouldn't. The root of the problem is that some of the states in the base_locations table (PR) did not exist in the states_table, so essentially I had filtered out everything, and passed an empty table on to mutate. The moral of the story is that you may have a data issue and not (just) a code problem (so you may need to clean your data.)

Thanks for agstudy and zx8754's answers above for helping with the debug.

How can I correctly format currency using jquery?

Use jquery.inputmask 3.x. See demos here

Include files:

<script src="/assets/jquery.inputmask.js" type="text/javascript"></script>
<script src="/assets/jquery.inputmask.extensions.js" type="text/javascript"></script>
<script src="/assets/jquery.inputmask.numeric.extensions.js" type="text/javascript"></script>

And code as

$(selector).inputmask('decimal',
  { 'alias': 'numeric',
    'groupSeparator': '.',
    'autoGroup': true,
    'digits': 2,
    'radixPoint': ",",
    'digitsOptional': false,
    'allowMinus': false,
    'prefix': '$ ',
    'placeholder': '0'
  }
);

Highlights:

  • easy to use
  • optional parts anywere in the mask
  • possibility to define aliases which hide complexity
  • date / datetime masks
  • numeric masks
  • lots of callbacks
  • non-greedy masks
  • many features can be enabled/disabled/configured by options
  • supports readonly/disabled/dir="rtl" attributes
  • support data-inputmask attribute(s)
  • multi-mask support
  • regex-mask support
  • dynamic-mask support
  • preprocessing-mask support
  • value formatting / validating without input element

Transaction marked as rollback only: How do I find the cause

To quickly fetch the causing exception without the need to re-code or rebuild, set a breakpoint on

org.hibernate.ejb.TransactionImpl.setRollbackOnly() // Hibernate < 4.3, or
org.hibernate.jpa.internal.TransactionImpl() // as of Hibernate 4.3

and go up in the stack, usually to some Interceptor. There you can read the causing exception from some catch block.

Example of Named Pipes

You can actually write to a named pipe using its name, btw.

Open a command shell as Administrator to get around the default "Access is denied" error:

echo Hello > \\.\pipe\PipeName

How to detect duplicate values in PHP array?

Perhaps something like this (untested code but should give you an idea)?

$new = array();

foreach ($array as $value)
{
    if (isset($new[$value]))
        $new[$value]++;
    else
        $new[$value] = 1;
}

Then you'll get a new array with the values as keys and their value is the number of times they existed in the original array.

How to SUM parts of a column which have same text value in different column in the same row

If your data has the names grouped as shown then you can use this formula in D2 copied down to get a total against the last entry for each name

=IF((A2=A3)*(B2=B3),"",SUM(C$2:C2)-SUM(D$1:D1))

See screenshot

enter image description here

MySQL Event Scheduler on a specific time everyday

My use case is similar, except that I want a log cleanup event to run at 2am every night. As I said in the comment above, the DAY_HOUR doesn't work for me. In my case I don't really mind potentially missing the first day (and, given it is to run at 2am then 2am tomorrow is almost always the next 2am) so I use:

CREATE EVENT applog_clean_event
ON SCHEDULE 
    EVERY 1 DAY
    STARTS str_to_date( date_format(now(), '%Y%m%d 0200'), '%Y%m%d %H%i' ) + INTERVAL 1 DAY
COMMENT 'Test'
DO 

Batch program to to check if process exists

TASKLIST does not set errorlevel.

echo off
tasklist /fi "imagename eq notepad.exe" |find ":" > nul
if errorlevel 1 taskkill /f /im "notepad.exe"
exit

should do the job, since ":" should appear in TASKLIST output only if the task is NOT found, hence FIND will set the errorlevel to 0 for not found and 1 for found

Nevertheless,

taskkill /f /im "notepad.exe"

will kill a notepad task if it exists - it can do nothing if no notepad task exists, so you don't really need to test - unless there's something else you want to do...like perhaps

echo off
tasklist /fi "imagename eq notepad.exe" |find ":" > nul
if errorlevel 1 taskkill /f /im "notepad.exe"&exit

which would appear to do as you ask - kill the notepad process if it exists, then exit - otherwise continue with the batch

Get Cell Value from a DataTable in C#

The DataRow has also an indexer:

Object cellValue = dt.Rows[i][j];

But i would prefer the strongly typed Field extension method which also supports nullable types:

int number = dt.Rows[i].Field<int>(j);

or even more readable and less error-prone with the name of the column:

double otherNumber = dt.Rows[i].Field<double>("DoubleColumn");

XML Parsing - Read a Simple XML File and Retrieve Values

class Program
{

    static void Main(string[] args)
    {

        //Load XML from local
        string sourceFileName="";
        string element=string.Empty;
        var FolderPath=@"D:\Test\RenameFileWithXmlAttribute";

            string[] files = Directory.GetFiles(FolderPath, "*.xml");
            foreach (string xmlfile in files)
            {
                try
                {
                    sourceFileName = xmlfile;
                    XElement xele = XElement.Load(sourceFileName);
                    string convertToString = xele.ToString();
                    XElement parseXML = XElement.Parse(convertToString);
                    element = parseXML.Descendants("Meta").Where(x => (string)x.Attribute("name") == "XMLTAG").Last().Value;
                    DirectoryInfo CurrentDate = Directory.CreateDirectory(DateTime.Now.ToString("yyyy-MM-dd"));
                    string saveWithThisName= Path.Combine(CurrentDate.FullName, element);
                    File.Copy(sourceFileName, saveWithThisName,true);                      
                }
                catch(Exception ex)
                {

                }      
            }       
    }
}

node.js vs. meteor.js what's the difference?

Meteor is a framework built ontop of node.js. It uses node.js to deploy but has several differences.

The key being it uses its own packaging system instead of node's module based system. It makes it easy to make web applications using Node. Node can be used for a variety of things and on its own is terrible at serving up dynamic web content. Meteor's libraries make all of this easy.

Display/Print one column from a DataFrame of Series in Pandas

Not sure what you are really after but if you want to print exactly what you have you can do:

Option 1

print(df['Item'].to_csv(index=False))

Sweet
Candy
Chocolate

Option 2

for v in df['Item']:
    print(v)

Sweet
Candy
Chocolate

How to get an object's property's value by property name?

Sure

write-host ($obj | Select -ExpandProperty "SomeProp")

Or for that matter:

$obj."SomeProp"

Injecting Mockito mocks into a Spring bean

Today I found out that a spring context where I declared a before the Mockito beans, was failing to load. After moving the AFTER the mocks, the app context was loaded successfully. Take care :)

Invalid character in identifier

I got that error, when sometimes I type in Chinese language. When it comes to punctuation marks, you do not notice that you are actually typing the Chinese version, instead of the English version.

The interpreter will give you an error message, but for human eyes, it is hard to notice the difference.

For example, "," in Chinese; and "," in English. So be careful with your language setting.

Any difference between await Promise.all() and multiple await?

Note:

This answer just covers the timing differences between await in series and Promise.all. Be sure to read @mikep's comprehensive answer that also covers the more important differences in error handling.


For the purposes of this answer I will be using some example methods:

  • res(ms) is a function that takes an integer of milliseconds and returns a promise that resolves after that many milliseconds.
  • rej(ms) is a function that takes an integer of milliseconds and returns a promise that rejects after that many milliseconds.

Calling res starts the timer. Using Promise.all to wait for a handful of delays will resolve after all the delays have finished, but remember they execute at the same time:

Example #1
const data = await Promise.all([res(3000), res(2000), res(1000)])
//                              ^^^^^^^^^  ^^^^^^^^^  ^^^^^^^^^
//                               delay 1    delay 2    delay 3
//
// ms ------1---------2---------3
// =============================O delay 1
// ===================O           delay 2
// =========O                     delay 3
//
// =============================O Promise.all

_x000D_
_x000D_
async function example() {
  const start = Date.now()
  let i = 0
  function res(n) {
    const id = ++i
    return new Promise((resolve, reject) => {
      setTimeout(() => {
        resolve()
        console.log(`res #${id} called after ${n} milliseconds`, Date.now() - start)
      }, n)
    })
  }

  const data = await Promise.all([res(3000), res(2000), res(1000)])
  console.log(`Promise.all finished`, Date.now() - start)
}

example()
_x000D_
_x000D_
_x000D_

This means that Promise.all will resolve with the data from the inner promises after 3 seconds.

But, Promise.all has a "fail fast" behavior:

Example #2
const data = await Promise.all([res(3000), res(2000), rej(1000)])
//                              ^^^^^^^^^  ^^^^^^^^^  ^^^^^^^^^
//                               delay 1    delay 2    delay 3
//
// ms ------1---------2---------3
// =============================O delay 1
// ===================O           delay 2
// =========X                     delay 3
//
// =========X                     Promise.all

_x000D_
_x000D_
async function example() {
  const start = Date.now()
  let i = 0
  function res(n) {
    const id = ++i
    return new Promise((resolve, reject) => {
      setTimeout(() => {
        resolve()
        console.log(`res #${id} called after ${n} milliseconds`, Date.now() - start)
      }, n)
    })
  }
  
  function rej(n) {
    const id = ++i
    return new Promise((resolve, reject) => {
      setTimeout(() => {
        reject()
        console.log(`rej #${id} called after ${n} milliseconds`, Date.now() - start)
      }, n)
    })
  }
  
  try {
    const data = await Promise.all([res(3000), res(2000), rej(1000)])
  } catch (error) {
    console.log(`Promise.all finished`, Date.now() - start)
  }
}

example()
_x000D_
_x000D_
_x000D_

If you use async-await instead, you will have to wait for each promise to resolve sequentially, which may not be as efficient:

Example #3
const delay1 = res(3000)
const delay2 = res(2000)
const delay3 = rej(1000)

const data1 = await delay1
const data2 = await delay2
const data3 = await delay3

// ms ------1---------2---------3
// =============================O delay 1
// ===================O           delay 2
// =========X                     delay 3
//
// =============================X await

_x000D_
_x000D_
async function example() {
  const start = Date.now()
  let i = 0
  function res(n) {
    const id = ++i
    return new Promise((resolve, reject) => {
      setTimeout(() => {
        resolve()
        console.log(`res #${id} called after ${n} milliseconds`, Date.now() - start)
      }, n)
    })
  }
  
  function rej(n) {
    const id = ++i
    return new Promise((resolve, reject) => {
      setTimeout(() => {
        reject()
        console.log(`rej #${id} called after ${n} milliseconds`, Date.now() - start)
      }, n)
    })
  }
  
  try {
    const delay1 = res(3000)
    const delay2 = res(2000)
    const delay3 = rej(1000)

    const data1 = await delay1
    const data2 = await delay2
    const data3 = await delay3
  } catch (error) {
    console.log(`await finished`, Date.now() - start)
  }
}

example()
_x000D_
_x000D_
_x000D_

java.lang.ClassNotFoundException: org.eclipse.core.runtime.adaptor.EclipseStarter

This can a time consuming problem. For those who want to get on with their work because they do not want to waste much time, I would like to suggest that you download the zip file and unpack the zip file and use it. The zip file is available for all major OS.

https://www.eclipse.org/downloads/packages/

Here instead of hitting the Download button, which will download the Eclipse installer, scroll to the middle area where a list of the various Eclipse IDEs with their respective features are shown. Select the Eclipse IDE of your like and click on the link on the right-hand-side to download the zip or corresponding file for your OS version.

If you still want to use your other Eclipse installations because, for example, you have plugins installed, then download the Eclipse installer and on the right-hand top corner press the icon with 3 minus. After that press Update, then after restart, install the version of Eclipse IDE, which you want (the one that you want to reactivate) in a different folder. The installation will take some time. After the installation is finished you should be able to start your old Eclipse IDE.

Set up a scheduled job?

Put the following at the top of your cron.py file:

#!/usr/bin/python
import os, sys
sys.path.append('/path/to/') # the parent directory of the project
sys.path.append('/path/to/project') # these lines only needed if not on path
os.environ['DJANGO_SETTINGS_MODULE'] = 'myproj.settings'

# imports and code below

Convert number to varchar in SQL with formatting

Correción: 3-LEN

declare @t  TINYINT 
set @t =233
SELECT ISNULL(REPLICATE('0',3-LEN(@t)),'') + CAST(@t AS VARCHAR) 

How to sort rows of HTML table that are called from MySQL

It depends on nature of your data. The answer varies based on its size and data type. I saw a lot of SQL solutions based on ORDER BY. I would like to suggest javascript alternatives.

In all answers, I don't see anyone mentioning pagination problem for your future table. Let's make it easier for you. If your table doesn't have pagination, it's more likely that a javascript solution makes everything neat and clean for you on the client side. If you think this table will explode after you put data in it, you have to think about pagination as well. (you have to go to first page every time when you change the sorting column)

Another aspect is the data type. If you use SQL you have to be careful about the type of your data and what kind of sorting suites for it. For example, if in one of your VARCHAR columns you store integer numbers, the sorting will not take their integer value into account: instead of 1, 2, 11, 22 you will get 1, 11, 2, 22.

You can find jquery plugins or standalone javascript sortable tables on google. It worth mentioning that the <table> in HTML5 has sortable attribute, but apparently it's not implemented yet.

How do I create a circle or square with just CSS - with a hollow center?

i don't know of a simple css(2.1 standard)-only solution for circles, but for squares you can do easily:

.squared {
    border: 2x solid black;
}

then, use the following html code:

<img src="…" alt="an image " class="squared" />

What does the exclamation mark do before the function?

There is a good point for using ! for function invocation marked on airbnb JavaScript guide

Generally idea for using this technique on separate files (aka modules) which later get concatenated. The caveat here is that files supposed to be concatenated by tools which put the new file at the new line (which is anyway common behavior for most of concat tools). In that case, using ! will help to avoid error in if previously concatenated module missed trailing semicolon, and yet that will give the flexibility to put them in any order with no worry.

!function abc(){}();
!function bca(){}();

Will work the same as

!function abc(){}();
(function bca(){})();

but saves one character and arbitrary looks better.

And by the way any of +,-,~,void operators have the same effect, in terms of invoking the function, for sure if you have to use something to return from that function they would act differently.

abcval = !function abc(){return true;}() // abcval equals false
bcaval = +function bca(){return true;}() // bcaval equals 1
zyxval = -function zyx(){return true;}() // zyxval equals -1
xyzval = ~function xyz(){return true;}() // your guess?

but if you using IIFE patterns for one file one module code separation and using concat tool for optimization (which makes one line one file job), then construction

!function abc(/*no returns*/) {}()
+function bca() {/*no returns*/}()

Will do safe code execution, same as a very first code sample.

This one will throw error cause JavaScript ASI will not be able to do its work.

!function abc(/*no returns*/) {}()
(function bca() {/*no returns*/})()

One note regarding unary operators, they would do similar work, but only in case, they used not in the first module. So they are not so safe if you do not have total control over the concatenation order.

This works:

!function abc(/*no returns*/) {}()
^function bca() {/*no returns*/}()

This not:

^function abc(/*no returns*/) {}()
!function bca() {/*no returns*/}()

Android : Fill Spinner From Java Code Programmatically

Here is an example to fully programmatically:

  • init a Spinner.
  • fill it with data via a String List.
  • resize the Spinner and add it to my View.
  • format the Spinner font (font size, colour, padding).
  • clear the Spinner.
  • add new values to the Spinner.
  • redraw the Spinner.

I am using the following class vars:

Spinner varSpinner;
List<String> varSpinnerData;

float varScaleX;
float varScaleY;    

A - Init and render the Spinner (varRoot is a pointer to my main Activity):

public void renderSpinner() {


    List<String> myArraySpinner = new ArrayList<String>();

    myArraySpinner.add("red");
    myArraySpinner.add("green");
    myArraySpinner.add("blue");     

    varSpinnerData = myArraySpinner;

    Spinner mySpinner = new Spinner(varRoot);               

    varSpinner = mySpinner;

    ArrayAdapter<String> spinnerArrayAdapter = new ArrayAdapter<String>(varRoot, android.R.layout.simple_spinner_item, myArraySpinner);
    spinnerArrayAdapter.setDropDownViewResource(android.R.layout.simple_spinner_dropdown_item); // The drop down vieww
    mySpinner.setAdapter(spinnerArrayAdapter);

B - Resize and Add the Spinner to my View:

    FrameLayout.LayoutParams myParamsLayout = new FrameLayout.LayoutParams(
            FrameLayout.LayoutParams.MATCH_PARENT, 
            FrameLayout.LayoutParams.WRAP_CONTENT);
    myParamsLayout.gravity = Gravity.NO_GRAVITY;             

    myParamsLayout.leftMargin = (int) (100 * varScaleX);
    myParamsLayout.topMargin = (int) (350 * varScaleY);             
    myParamsLayout.width = (int) (300 * varScaleX);;
    myParamsLayout.height = (int) (60 * varScaleY);;


    varLayoutECommerce_Dialogue.addView(mySpinner, myParamsLayout);

C - Make the Click handler and use this to set the font.

    mySpinner.setOnItemSelectedListener(new OnItemSelectedListener() {

        @Override
        public void onItemSelected(AdapterView<?> parentView, View selectedItemView, int myPosition, long myID) {

            Log.i("renderSpinner -> ", "onItemSelected: " + myPosition + "/" + myID);

            ((TextView) parentView.getChildAt(0)).setTextColor(Color.GREEN);
            ((TextView) parentView.getChildAt(0)).setTextSize(TypedValue.COMPLEX_UNIT_PX, (int) (varScaleY * 22.0f) );
            ((TextView) parentView.getChildAt(0)).setPadding(1,1,1,1);


        }

        @Override
        public void onNothingSelected(AdapterView<?> parentView) {
            // your code here
        }

    });

}   

D - Update the Spinner with new data:

private void updateInitSpinners(){

     String mySelected = varSpinner.getSelectedItem().toString();
     Log.i("TPRenderECommerce_Dialogue -> ", "updateInitSpinners -> mySelected: " + mySelected);


     varSpinnerData.clear();

     varSpinnerData.add("Hello World");
     varSpinnerData.add("Hello World 2");

     ((BaseAdapter) varSpinner.getAdapter()).notifyDataSetChanged();
     varSpinner.invalidate();
     varSpinner.setSelection(1);

}

}

What I have not been able to solve in the updateInitSpinners, is to do varSpinner.setSelection(0); and have the custom font settings activated automatically.

UPDATE:

This "ugly" solution solves the varSpinner.setSelection(0); issue, but I am not very happy with it:

private void updateInitSpinners(){

    String mySelected = varSpinner.getSelectedItem().toString();
    Log.i("TPRenderECommerce_Dialogue -> ", "updateInitSpinners -> mySelected: " + mySelected);


    varSpinnerData.clear();

    ArrayAdapter<String> spinnerArrayAdapter = new ArrayAdapter<String>(varRoot, android.R.layout.simple_spinner_item, varSpinnerData);
    spinnerArrayAdapter.setDropDownViewResource(android.R.layout.simple_spinner_dropdown_item); 
    varSpinner.setAdapter(spinnerArrayAdapter);  


    varSpinnerData.add("Hello World");
    varSpinnerData.add("Hello World 2");

    ((BaseAdapter) varSpinner.getAdapter()).notifyDataSetChanged();
    varSpinner.invalidate();
    varSpinner.setSelection(0);

}

}

Hope this helps......

How to convert color code into media.brush?

Sorry to be so late to the party! I came across a similar issue, in WinRT. I'm not sure whether you're using WPF or WinRT, but they do differ in some ways (some better than others). Hopefully this will help people across the board, whichever situation they're in.

You could always use the code from the converter class I created to re-use and do in your C# code-behind, or wherever you're using it, to be honest:

I made it with the intention that a 6-digit (RGB), or an 8-digit (ARGB) Hex value could be used either way.

So I created a converter class:

public class StringToSolidColorBrushConverter : IValueConverter
{
    public object Convert(object value, Type targetType, object parameter, string language)
    {
        var hexString = (value as string).Replace("#", "");

        if (string.IsNullOrWhiteSpace(hexString)) throw new FormatException();
        if (hexString.Length != 6 || hexString.Length != 8) throw new FormatException();

        try
        {
            var a = hexString.Length == 8 ? hexString.Substring(0, 2) : "255";
            var r = hexString.Length == 8 ? hexString.Substring(2, 2) : hexString.Substring(0, 2);
            var g = hexString.Length == 8 ? hexString.Substring(4, 2) : hexString.Substring(2, 2);
            var b = hexString.Length == 8 ? hexString.Substring(6, 2) : hexString.Substring(4, 2);

            return new SolidColorBrush(ColorHelper.FromArgb(
                byte.Parse(a, System.Globalization.NumberStyles.HexNumber),
                byte.Parse(r, System.Globalization.NumberStyles.HexNumber),
                byte.Parse(g, System.Globalization.NumberStyles.HexNumber),
                byte.Parse(b, System.Globalization.NumberStyles.HexNumber)));
        }
        catch
        {
            throw new FormatException();
        }
    }

    public object ConvertBack(object value, Type targetType, object parameter, string language)
    {
        throw new NotImplementedException();
    }
}

Added it into my App.xaml:

<ResourceDictionary>
    ...
    <converters:StringToSolidColorBrushConverter x:Key="StringToSolidColorBrushConverter" />
    ...
</ResourceDictionary>

And used it in my View's Xaml:

<Grid>
    <Rectangle Fill="{Binding RectangleColour,
               Converter={StaticResource StringToSolidColorBrushConverter}}"
               Height="20" Width="20" />
</Grid>

Works a charm!

Side note... Unfortunately, WinRT hasn't got the System.Windows.Media.BrushConverter that H.B. suggested; so I needed another way, otherwise I would have made a VM property that returned a SolidColorBrush (or similar) from the RectangleColour string property.

Access POST values in Symfony2 request object

Symfony doc to get request data

Finally, the raw data sent with the request body can be accessed using getContent():

$content = $request->getContent();

How to force input to only allow Alpha Letters?

<input type="text" name="name" id="event" onkeydown="return alphaOnly(event);"   required />


function alphaOnly(event) {
  var key = event.keyCode;`enter code here`
  return ((key >= 65 && key <= 90) || key == 8);
};

Populate unique values into a VBA array from Excel

If you don't mind using the Variant data type, then you can use the in-built worksheet function Unique as shown.

sub unique_results_to_array()
    dim rng_data as Range
    set rng_data = activesheet.range("A1:A10") 'enter the range of data here

    dim my_arr() as Variant
    my_arr = WorksheetFunction.Unique(rng_data)
    
    first_val  = my_arr(1,1)
    second_val = my_arr(2,1)
    third_val = my_arr(3,1)   'etc...    

end sub

How can I export data to an Excel file

I used interop to open Excel and to modify the column widths once the data was done. If you use interop to spit the data into a new Excel workbook (if this is what you want), it will be terribly slow. Instead, I generated a .CSV, then opened the .CSV in Excel. This has its own problems, but I've found this the quickest method.

First, convert the .CSV:

// Convert array data into CSV format.
// Modified from http://csharphelper.com/blog/2018/04/write-a-csv-file-from-an-array-in-c/.
private string GetCSV(List<string> Headers, List<List<double>> Data)
{
    // Get the bounds.
    var rows = Data[0].Count;
    var cols = Data.Count;
    var row = 0;

    // Convert the array into a CSV string.
    StringBuilder sb = new StringBuilder();

    // Add the first field in this row.
    sb.Append(Headers[0]);

    // Add the other fields in this row separated by commas.
    for (int col = 1; col < cols; col++)
        sb.Append("," + Headers[col]);

    // Move to the next line.
    sb.AppendLine();

    for (row = 0; row < rows; row++)
    {
        // Add the first field in this row.
        sb.Append(Data[0][row]);

        // Add the other fields in this row separated by commas.
        for (int col = 1; col < cols; col++)
            sb.Append("," + Data[col][row]);

        // Move to the next line.
        sb.AppendLine();
    }

    // Return the CSV format string.
    return sb.ToString();
}

Then, export it to Excel:

public void ExportToExcel()
{
    // Initialize app and pop Excel on the screen.
    var excelApp = new Excel.Application { Visible = true };

    // I use unix time to give the files a unique name that's almost somewhat useful.
    DateTime dateTime = DateTime.UtcNow;
    long unixTime = ((DateTimeOffset)dateTime).ToUnixTimeSeconds();
    var path = @"C:\Users\my\path\here + unixTime + ".csv";

    var csv = GetCSV();
    File.WriteAllText(path, csv);

    // Create a new workbook and get its active sheet.
    excelApp.Workbooks.Open(path);

    var workSheet = (Excel.Worksheet)excelApp.ActiveSheet;

    // iterate over each value and throw it in the chart
    for (var column = 0; column < Data.Count; column++)
    {
        ((Excel.Range)workSheet.Columns[column + 1]).AutoFit();
    }

    currentSheet = workSheet;
}

You'll have to install some stuff, too...

  1. Right click on the solution from solution explorer and select "Manage NuGet Packages." - add Microsoft.Office.Interop.Excel

  2. It might actually work right now if you created the project the way interop wants you to. If it still doesn't work, I had to create a new project in a different category. Under New > Project, select Visual C# > Windows Desktop > Console App. Otherwise, the interop tools won't work.

In case I forgot anything, here's my 'using' statements:

using System;
using System.Collections.Generic;
using System.IO;
using System.Linq;
using System.Text;
using Excel = Microsoft.Office.Interop.Excel;

How to download videos from youtube on java?

Regarding the format (mp4 or flv) decide which URL you want to use. Then use this tutorial to download the video and save it into a local directory.

How to use stringstream to separate comma separated strings

#include <iostream>
#include <string>
#include <sstream>
using namespace std;
int main()
{
    std::string input = "abc,def,   ghi";
    std::istringstream ss(input);
    std::string token;
    size_t pos=-1;
    while(ss>>token) {
      while ((pos=token.rfind(',')) != std::string::npos) {
        token.erase(pos, 1);
      }
      std::cout << token << '\n';
    }
}

Use of alloc init instead of new

There are a bunch of reasons here: http://macresearch.org/difference-between-alloc-init-and-new

Some selected ones are:

  • new doesn't support custom initializers (like initWithString)
  • alloc-init is more explicit than new

General opinion seems to be that you should use whatever you're comfortable with.

Moment js get first and last day of current month

You can do this without moment.js

A way to do this in native Javascript code :

var date = new Date(), y = date.getFullYear(), m = date.getMonth();
var firstDay = new Date(y, m, 1);
var lastDay = new Date(y, m + 1, 0);

firstDay = moment(firstDay).format(yourFormat);
lastDay = moment(lastDay).format(yourFormat);

"Items collection must be empty before using ItemsSource."

I've had this error when I tried applying context menus to my TreeView. Those tries ended up in a bad XAML which compiled somehow:

<TreeView Height="Auto" MinHeight="100"  ItemsSource="{Binding Path=TreeNodes, Mode=TwoWay}" 
    ContextMenu="{Binding Converter={StaticResource ContextMenuConverter}}">
    ContextMenu="">
    <TreeView.ItemContainerStyle>
    ...  

Note the problematic line: ContextMenu=""> .
I don't know why it compiled, but I figured it's worth mentioning as a reason for this cryptic exception message. Like Armentage said, look around the XAML carefully, especially in places you've recently edited.

Basic calculator in Java

public class SwitchExample {

    public static void main(String[] args) throws Exception {
        System.out.println(":::::::::::::::::::::Start:::::::::::::::::::");
        System.out.println("\n\n");

        System.out.println("1. Addition");
        System.out.println("2. Multiplication");
        System.out.println("3. Substraction");
        System.out.println("4. Division");
        System.out.println("0. Exit");
        System.out.println("\n");

        System.out.println("Enter Your Choice :::::::  ");
        BufferedReader br = new BufferedReader(new InputStreamReader(System.in));
        String str = br.readLine();

        int usrChoice = Integer.parseInt(str);

        switch (usrChoice) {
            case 1:
                doAddition();
                break;
            case 2:
                doMultiplication();
                break;
            case 3:
                doSubstraction();
                break;
            case 4:
                doDivision();
                break;

            case 0:
                System.out.println("Thank you.....");
                break;

            default:
                System.out.println("Invalid Value");
        }

        System.out.println(":::::::::::::::::::::End:::::::::::::::::::");
}

public static void doAddition() throws Exception {
    System.out.println("******* Enter in Addition Process ********");

    String strNo1, strNo2;

    System.out.println("Enter Number 1 For Addition : ");
    BufferedReader br = new BufferedReader(new InputStreamReader(System.in));
    strNo1 = br.readLine();

    System.out.println("Enter Number 2 For Addition : ");
    BufferedReader br1 = new BufferedReader(new InputStreamReader(System.in));
    strNo2 = br1.readLine();

    int no1 = Integer.parseInt(strNo1);
    int no2 = Integer.parseInt(strNo2);

    int result = no1 + no2;

    System.out.println("Addition of " + no1 + " and " + no2 + " is ::::::: " + result);
}

public static void doSubstraction() throws Exception {
    System.out.println("******* Enter in Substraction Process ********");

    String strNo1, strNo2;

    System.out.println("Enter Number 1 For Substraction : ");
    BufferedReader br = new BufferedReader(new InputStreamReader(System.in));
    strNo1 = br.readLine();

    System.out.println("Enter Number 2 For Substraction : ");
    BufferedReader br1 = new BufferedReader(new InputStreamReader(System.in));
    strNo2 = br1.readLine();

    int no1 = Integer.parseInt(strNo1);
    int no2 = Integer.parseInt(strNo2);

    int result = no1 - no2;

    System.out.println("Substraction of " + no1 + " and " + no2 + " is ::::::: " + result);
}

public static void doMultiplication() throws Exception {
    System.out.println("******* Enter in Multiplication Process ********");

    String strNo1, strNo2;

    System.out.println("Enter Number 1 For Multiplication : ");
    BufferedReader br = new BufferedReader(new InputStreamReader(System.in));
    strNo1 = br.readLine();

    System.out.println("Enter Number 2 For Multiplication : ");
    BufferedReader br1 = new BufferedReader(new InputStreamReader(System.in));
    strNo2 = br1.readLine();

    int no1 = Integer.parseInt(strNo1);
    int no2 = Integer.parseInt(strNo2);

    int result = no1 * no2;

    System.out.println("Multiplication of " + no1 + " and " + no2 + " is ::::::: " + result);
}

public static void doDivision() throws Exception {
    System.out.println("******* Enter in Dividion Process ********");

    String strNo1, strNo2;

    System.out.println("Enter Number 1 For Dividion : ");
    BufferedReader br = new BufferedReader(new InputStreamReader(System.in));
    strNo1 = br.readLine();

    System.out.println("Enter Number 2 For Dividion : ");
    BufferedReader br1 = new BufferedReader(new InputStreamReader(System.in));
    strNo2 = br1.readLine();

    int no1 = Integer.parseInt(strNo1);
    int no2 = Integer.parseInt(strNo2);

    float result = no1 / no2;

    System.out.println("Division of " + no1 + " and " + no2 + " is ::::::: " + result);
}

}

how do I get a new line, after using float:left?

Try the clear property.

Remember that float removes an element from the document layout - so yes, in a way it is "interfering" with br and p tags, in the sense that it would basically be ignoring anything in the main flow layout.

3D Plotting from X, Y, Z Data, Excel or other Tools

I ended up using matplotlib :)

from mpl_toolkits.mplot3d import Axes3D
from matplotlib import cm
import matplotlib.pyplot as plt
import numpy as np
x = [1000,1000,1000,1000,1000,5000,5000,5000,5000,5000,10000,10000,10000,10000,10000]
y = [13,21,29,37,45,13,21,29,37,45,13,21,29,37,45]
z = [75.2,79.21,80.02,81.2,81.62,84.79,87.38,87.9,88.54,88.56,88.34,89.66,90.11,90.79,90.87]
fig = plt.figure()
ax = fig.gca(projection='3d')
ax.plot_trisurf(x, y, z, cmap=cm.jet, linewidth=0.2)
plt.show()

Download old version of package with NuGet

In NuGet 3.x (Visual Studio 2015) you can just select the version from the UI

NuGet 3 package manager UI

Setting the focus to a text field

I did it by setting an AncesterAdded event on the textField and the requesting focus in the window.

Select all occurrences of selected word in VSCode

on Mac:

select all matches: Command + Shift + L

but if you just want to select another match up coming next: Command + D

CSS Background image not loading

I had the same issue. The problem ended up being the path to the image file. Make sure the image path is relative to the location of the CSS file instead of the HTML.

\n or \n in php echo not print

Better use PHP_EOL ("End Of Line") instead. It's cross-platform.

E.g.:

$unit1 = 'paragrahp1';
$unit2 = 'paragrahp2';
echo '<p>' . $unit1 . '</p>' . PHP_EOL;
echo '<p>' . $unit2 . '</p>';

How do I include a pipe | in my linux find -exec command?

I found that running a string shell command (sh -c) works best, for example:

find -name 'file_*' -follow -type f -exec bash -c "zcat \"{}\" | agrep -dEOE 'grep'" \;

Get DOM content of cross-domain iframe

There's a workaround to achieve it.

  1. First, bind your iframe to a target page with relative url. The browsers will treat the site in iframe the same domain with your website.

  2. In your web server, using a rewrite module to redirect request from the relative url to absolute url. If you use IIS, I recommend you check on IIRF module.

How to use a variable of one method in another method?

You can't. Variables defined inside a method are local to that method.

If you want to share variables between methods, then you'll need to specify them as member variables of the class. Alternatively, you can pass them from one method to another as arguments (this isn't always applicable).


Looks like you're using instance methods instead of static ones.

If you don't want to create an object, you should declare all your methods static, so something like

private static void methodName(Argument args...)

If you want a variable to be accessible by all these methods, you should initialise it outside the methods and to limit its scope, declare it private.

private static int[][] array = new int[3][5];

Global variables are usually looked down upon (especially for situations like your one) because in a large-scale program they can wreak havoc, so making it private will prevent some problems at the least.

Also, I'll say the usual: You should try to keep your code a bit tidy. Use descriptive class, method and variable names and keep your code neat (with proper indentation, linebreaks etc.) and consistent.

Here's a final (shortened) example of what your code should be like:

public class Test3 {
    //Use this array in your methods
    private static int[][] scores = new int[3][5];

    /* Rather than just "Scores" name it so people know what
     * to expect
     */
    private static void createScores() {
        //Code...
    }
    //Other methods...

    /* Since you're now using static methods, you don't 
     * have to initialise an object and call its methods.
     */
    public static void main(String[] args){
        createScores();
        MD();   //Don't know what these do
        sumD(); //so I'll leave them.
    }
}

Ideally, since you're using an array, you would create the array in the main method and pass it as an argument across each method, but explaining how that works is probably a whole new question on its own so I'll leave it at that.

Get an image extension from an uploaded file in Laravel

Do something like this:

if($request->hasFile('video')){

    $video=$request->file('video');
    $filename=str_random(20).".".$video->extension(); 
    $path = Storage::putFileAs(
            '/', $video, $filename
     );
    $data['video']=$filename;
 }

Submitting HTML form using Jquery AJAX

If you add:

jquery.form.min.js

You can simply do this:

<script>
$('#myform').ajaxForm(function(response) {
  alert(response);
});

// this will register the AJAX for <form id="myform" action="some_url">
// and when you submit the form using <button type="submit"> or $('myform').submit(), then it will send your request and alert response
</script>

NOTE:

You could use simple $('FORM').serialize() as suggested in post above, but that will not work for FILE INPUTS... ajaxForm() will.

IIS7 Settings File Locations

Also check this answer from here: Cannot manually edit applicationhost.config

The answer is simple, if not that obvious: win2008 is 64bit, notepad++ is 32bit. When you navigate to Windows\System32\inetsrv\config using explorer you are using a 64bit program to find the file. When you open the file using using notepad++ you are trying to open it using a 32bit program. The confusion occurs because, rather than telling you that this is what you are doing, windows allows you to open the file but when you save it the file's path is transparently mapped to Windows\SysWOW64\inetsrv\Config.

So in practice what happens is you open applicationhost.config using notepad++, make a change, save the file; but rather than overwriting the original you are saving a 32bit copy of it in Windows\SysWOW64\inetsrv\Config, therefore you are not making changes to the version that is actually used by IIS. If you navigate to the Windows\SysWOW64\inetsrv\Config you will find the file you just saved.

How to get around this? Simple - use a 64bit text editor, such as the normal notepad that ships with windows.

jQuery: Test if checkbox is NOT checked

Here is the simplest way given

 <script type="text/javascript">

        $(document).ready(function () {
            $("#chk_selall").change("click", function () {

                if (this.checked)
                {
                    //do something
                }
                if (!this.checked)
                {
                    //do something

                }

            });

        });

    </script>

What's the purpose of SQL keyword "AS"?

There is no difference between both statements above. AS is just a more explicit way of mentioning the alias

How to remove new line characters from data rows in mysql?

Playing with above answers, this one works for me

REPLACE(REPLACE(column_name , '\n', ''), '\r', '')

how to add a day to a date using jquery datepicker

The datepicker('setDate') sets the date in the datepicket not in the input.

You should add the date and set it in the input.

var date2 = $('.pickupDate').datepicker('getDate');
var nextDayDate = new Date();
nextDayDate.setDate(date2.getDate() + 1);
$('input').val(nextDayDate);

How do I use Node.js Crypto to create a HMAC-SHA1 hash?

Documentation for crypto: http://nodejs.org/api/crypto.html

const crypto = require('crypto')

const text = 'I love cupcakes'
const key = 'abcdeg'

crypto.createHmac('sha1', key)
  .update(text)
  .digest('hex')

Remove carriage return in Unix

There's a utility called dos2unix that exists on many systems, and can be easily installed on most.

XDocument or XmlDocument

If you're using .NET version 3.0 or lower, you have to use XmlDocument aka the classic DOM API. Likewise you'll find there are some other APIs which will expect this.

If you get the choice, however, I would thoroughly recommend using XDocument aka LINQ to XML. It's much simpler to create documents and process them. For example, it's the difference between:

XmlDocument doc = new XmlDocument();
XmlElement root = doc.CreateElement("root");
root.SetAttribute("name", "value");
XmlElement child = doc.CreateElement("child");
child.InnerText = "text node";
root.AppendChild(child);
doc.AppendChild(root);

and

XDocument doc = new XDocument(
    new XElement("root",
                 new XAttribute("name", "value"),
                 new XElement("child", "text node")));

Namespaces are pretty easy to work with in LINQ to XML, unlike any other XML API I've ever seen:

XNamespace ns = "http://somewhere.com";
XElement element = new XElement(ns + "elementName");
// etc

LINQ to XML also works really well with LINQ - its construction model allows you to build elements with sequences of sub-elements really easily:

// Customers is a List<Customer>
XElement customersElement = new XElement("customers",
    customers.Select(c => new XElement("customer",
        new XAttribute("name", c.Name),
        new XAttribute("lastSeen", c.LastOrder)
        new XElement("address",
            new XAttribute("town", c.Town),
            new XAttribute("firstline", c.Address1),
            // etc
    ));

It's all a lot more declarative, which fits in with the general LINQ style.

Now as Brannon mentioned, these are in-memory APIs rather than streaming ones (although XStreamingElement supports lazy output). XmlReader and XmlWriter are the normal ways of streaming XML in .NET, but you can mix all the APIs to some extent. For example, you can stream a large document but use LINQ to XML by positioning an XmlReader at the start of an element, reading an XElement from it and processing it, then moving on to the next element etc. There are various blog posts about this technique, here's one I found with a quick search.

python variable NameError

Your if statements are checking for int values. raw_input returns a string. Change the following line:

tSizeAns = raw_input() 

to

tSizeAns = int(raw_input()) 

Can a CSV file have a comment?

If you need something like:

  ¦ A                              ¦ B
--+--------------------------------+---
1 ¦ #My comment, something else    ¦
2 ¦ 1                              ¦ 2

Your CSV may contain the following lines:

"#My comment, something else"
1,2

Pay close attention at the 'quotes' in the first line.

When converting your text to columns using the Excel wizard, remember checking the 'Treat consecutive delimiters as one', setting it to use 'quotes' as delimiter.

Thus, Excel will split the text at the commas, keeping the 'comment' line as a single column value (and it will remove the quotes).

force css grid container to fill full screen of device

Two important CSS properties to set for full height pages are these:

  1. Allow the body to grow as high as the content in it requires.

    html { height: 100%; }
    
  2. Force the body not to get any smaller than then window height.

    body { min-height: 100%; }
    

What you do with your gird is irrelevant as long as you use fractions or percentages you should be safe in all cases.

Have a look at this common dashboard layout.

Assign format of DateTime with data annotations?

Apply DataAnnotation like:

[DisplayFormat(DataFormatString = "{0:MMM dd, yyyy}")]

Youtube - downloading a playlist - youtube-dl

I have tried everything above, but none could solve my problem. I fixed it by updating the old version of youtube-dl to download playlist. To update it

sudo youtube-dl -U

or

youtube-dl -U

after you have successfully updated using the above command

youtube-dl -cit https://www.youtube.com/playlist?list=PLttJ4RON7sleuL8wDpxbKHbSJ7BH4vvCk

RESTful URL design for search

RESTful does not recommend using verbs in URL's /cars/search is not restful. The right way to filter/search/paginate your API's is through Query Parameters. However there might be cases when you have to break the norm. For example, if you are searching across multiple resources, then you have to use something like /search?q=query

You can go through http://saipraveenblog.wordpress.com/2014/09/29/rest-api-best-practices/ to understand the best practices for designing RESTful API's

How to correctly save instance state of Fragments in back stack?

To correctly save the instance state of Fragment you should do the following:

1. In the fragment, save instance state by overriding onSaveInstanceState() and restore in onActivityCreated():

class MyFragment extends Fragment {

    @Override
    public void onActivityCreated(Bundle savedInstanceState) {
        super.onActivityCreated(savedInstanceState);
        ...
        if (savedInstanceState != null) {
            //Restore the fragment's state here
        }
    }
    ...
    @Override
    public void onSaveInstanceState(Bundle outState) {
        super.onSaveInstanceState(outState);

        //Save the fragment's state here
    }

}

2. And important point, in the activity, you have to save the fragment's instance in onSaveInstanceState() and restore in onCreate().

class MyActivity extends Activity {

    private MyFragment 

    public void onCreate(Bundle savedInstanceState) {
        ...
        if (savedInstanceState != null) {
            //Restore the fragment's instance
            mMyFragment = getSupportFragmentManager().getFragment(savedInstanceState, "myFragmentName");
            ...
        }
        ...
    }

    @Override
    protected void onSaveInstanceState(Bundle outState) {
        super.onSaveInstanceState(outState);

        //Save the fragment's instance
        getSupportFragmentManager().putFragment(outState, "myFragmentName", mMyFragment);
    }

}

Hope this helps.

How to fix Ora-01427 single-row subquery returns more than one row in select?

Use the following query:

SELECT E.I_EmpID AS EMPID,
       E.I_EMPCODE AS EMPCODE,
       E.I_EmpName AS EMPNAME,
       REPLACE(TO_CHAR(A.I_REQDATE, 'DD-Mon-YYYY'), ' ', '') AS FROMDATE,
       REPLACE(TO_CHAR(A.I_ENDDATE, 'DD-Mon-YYYY'), ' ', '') AS TODATE,
       TO_CHAR(NOD) AS NOD,
       DECODE(A.I_DURATION,
              'FD',
              'FullDay',
              'FN',
              'ForeNoon',
              'AN',
              'AfterNoon') AS DURATION,
       L.I_LeaveType AS LEAVETYPE,
       REPLACE(TO_CHAR((SELECT max(C.I_WORKDATE)
                         FROM T_COMPENSATION C
                        WHERE C.I_COMPENSATEDDATE = A.I_REQDATE
                          AND C.I_EMPID = A.I_EMPID),
                       'DD-Mon-YYYY'),
               ' ',
               '') AS WORKDATE,
       A.I_REASON AS REASON,
       AP.I_REJECTREASON AS REJECTREASON
  FROM T_LEAVEAPPLY A
 INNER JOIN T_EMPLOYEE_MS E
    ON A.I_EMPID = E.I_EmpID
   AND UPPER(E.I_IsActive) = 'YES'
   AND A.I_STATUS = '1'
 INNER JOIN T_LeaveType_MS L
    ON A.I_LEAVETYPEID = L.I_LEAVETYPEID
  LEFT OUTER JOIN T_APPROVAL AP
    ON A.I_REQDATE = AP.I_REQDATE
   AND A.I_EMPID = AP.I_EMPID
   AND AP.I_APPROVALSTATUS = '1'
 WHERE E.I_EMPID <> '22'
 ORDER BY A.I_REQDATE DESC

The trick is to force the inner query return only one record by adding an aggregate function (I have used max() here). This will work perfectly as far as the query is concerned, but, honestly, OP should investigate why the inner query is returning multiple records by examining the data. Are these multiple records really relevant business wise?

convert string to number node.js

Using parseInt() is a bad idea mainly because it never fails. Also because some results can be unexpected, like in the case of INFINITY.
Below is the function for handling unexpected behaviour.

function cleanInt(x) {
    x = Number(x);
    return x >= 0 ? Math.floor(x) : Math.ceil(x);
}

See results of below test cases.

console.log("CleanInt: ", cleanInt('xyz'), " ParseInt: ", parseInt('xyz'));
console.log("CleanInt: ", cleanInt('123abc'), " ParseInt: ", parseInt('123abc'));
console.log("CleanInt: ", cleanInt('234'), " ParseInt: ", parseInt('234'));
console.log("CleanInt: ", cleanInt('-679'), " ParseInt: ", parseInt('-679'));
console.log("CleanInt: ", cleanInt('897.0998'), " ParseInt: ", parseInt('897.0998'));
console.log("CleanInt: ", cleanInt('Infinity'), " ParseInt: ", parseInt('Infinity'));

result:

CleanInt:  NaN  ParseInt:  NaN
CleanInt:  NaN  ParseInt:  123
CleanInt:  234  ParseInt:  234
CleanInt:  -679  ParseInt:  -679
CleanInt:  897  ParseInt:  897
CleanInt:  Infinity  ParseInt:  NaN

Xcode 6: Keyboard does not show up in simulator

in viewDidLoad add this line

yourUiTextField.becomeFirstResponder()

NSDate get year/month/day

I'm writing this answer because it's the only approach that doesn't give you optionals back from NSDateComponent variables and/or force unwrapping those variables (also for Swift 3).

Swift 3

let date = Date()
let cal = Calendar.current
let year = cal.component(.year, from: date)
let month = cal.component(.month, from: date)
let day = cal.component(.day, from: date)

Swift 2

let date = NSDate()
let cal = NSCalendar.currentCalendar()
let year = cal.component(.Year, fromDate: date)
let month = cal.component(.Month, fromDate: date)
let day = cal.component(.Day, fromDate: date)

Bonus Swift 3 fun version

let date = Date()
let component = { (unit) in return Calendar.current().component(unit, from: date) }
let year = component(.year)
let month = component(.month)
let day = component(.day)

How to determine the longest increasing subsequence using dynamic programming?

here is java O(nlogn) implementation

import java.util.Scanner;

public class LongestIncreasingSeq {


    private static int binarySearch(int table[],int a,int len){

        int end = len-1;
        int beg = 0;
        int mid = 0;
        int result = -1;
        while(beg <= end){
            mid = (end + beg) / 2;
            if(table[mid] < a){
                beg=mid+1;
                result = mid;
            }else if(table[mid] == a){
                return len-1;
            }else{
                end = mid-1;
            }
        }
        return result;
    }
    
    public static void main(String[] args) {        
        
//        int[] t = {1, 2, 5,9,16};
//        System.out.println(binarySearch(t , 9, 5));
        Scanner in = new Scanner(System.in);
        int size = in.nextInt();//4;
        
        int A[] = new int[size];
        int table[] = new int[A.length]; 
        int k = 0;
        while(k<size){
            A[k++] = in.nextInt();
            if(k<size-1)
                in.nextLine();
        }        
        table[0] = A[0];
        int len = 1; 
        for (int i = 1; i < A.length; i++) {
            if(table[0] > A[i]){
                table[0] = A[i];
            }else if(table[len-1]<A[i]){
                table[len++]=A[i];
            }else{
                table[binarySearch(table, A[i],len)+1] = A[i];
            }            
        }
        System.out.println(len);
    }    
}

//TreeSet can be used

Shell Script: How to write a string to file and to stdout on console?

You can use >> to print in another file.

echo "hello" >> logfile.txt

How to configure log4j to only keep log files for the last seven days?

There's also a DailyRollingFileAppender

Edit: after reading this worrying statement:

DailyRollingFileAppender has been observed to exhibit synchronization issues and data loss. The log4j extras companion includes alternatives which should be considered for new deployments and which are discussed in the documentation for org.apache.log4j.rolling.RollingFileAppender.

from the above URL (which I never realized before), then the log4j-extras looks to be a better option.

"The breakpoint will not currently be hit. The source code is different from the original version." What does this mean?

Because build's version changed (most probably you modified the source code), so recompile the solution and run the application again, after then it will hit debug break point.

UIButton title text color

Besides de color, my problem was that I was setting the text using textlabel

bt.titleLabel?.text = title

and I solved changing to:

bt.setTitle(title, for: .normal)

In DB2 Display a table's definition

I know this is an old question, but this will do the job.

SELECT colname, typename, length, scale, default, nulls
  FROM syscat.columns
 WHERE tabname = '<table name>'
   AND tabschema = '<schema name>'
 ORDER BY colno

isolating a sub-string in a string before a symbol in SQL Server 2008

This can achieve using two SQL functions- SUBSTRING and CHARINDEX

You can read strings to a variable as shown in the above answers, or can add it to a SELECT statement as below:

SELECT SUBSTRING('Net Operating Loss - 2007' ,0, CHARINDEX('-','Net Operating Loss - 2007'))

How to set MouseOver event/trigger for border in XAML?

Yes, this is confusing...

According to this blog post, it looks like this is an omission from WPF.

To make it work you need to use a style:

    <Border Name="ClearButtonBorder" Grid.Column="1" CornerRadius="0,3,3,0">
        <Border.Style>
            <Style>
                <Setter Property="Border.Background" Value="Blue"/>
                <Style.Triggers>
                    <Trigger Property="Border.IsMouseOver" Value="True">
                        <Setter Property="Border.Background" Value="Green" />
                    </Trigger>
                </Style.Triggers>
            </Style>
        </Border.Style>
        <TextBlock HorizontalAlignment="Center" VerticalAlignment="Center" Text="X" />
    </Border>

I guess this problem isn't that common as most people tend to factor out this sort of thing into a style, so it can be used on multiple controls.

Limiting the number of characters per line with CSS

If you use CSS to select a monospace font, the problem of varying character length is easily solved.

SyntaxError: missing ) after argument list

SyntaxError: missing ) after argument list.

the issue also may occur if you pass string directly without a single or double quote.

$('#contentData').append("<div class='media'><div class='media-body'><a class='btn' href='" + type + "'  onclick=\"(canLaunch(' + v.LibraryItemName +  '))\">View &raquo;</a></div></div>").

so always keep the habit to pass in a quote like

 onclick=\"(canLaunch(\'' + v.LibraryItemName  + '\'))"\

Sorting 1 million 8-decimal-digit numbers with 1 MB of RAM

Here's some working C++ code which solves the problem.

Proof that the memory constraints are satisfied:

Editor: There is no proof of the maximum memory requirements offered by the author either in this post or in his blogs. Since the number of bits necessary to encode a value depends on the values previously encoded, such a proof is likely non-trivial. The author notes that the largest encoded size he could stumble upon empirically was 1011732, and chose the buffer size 1013000 arbitrarily.

typedef unsigned int u32;

namespace WorkArea
{
    static const u32 circularSize = 253250;
    u32 circular[circularSize] = { 0 };         // consumes 1013000 bytes

    static const u32 stageSize = 8000;
    u32 stage[stageSize];                       // consumes 32000 bytes

    ...

Together, these two arrays take 1045000 bytes of storage. That leaves 1048576 - 1045000 - 2×1024 = 1528 bytes for remaining variables and stack space.

It runs in about 23 seconds on my Xeon W3520. You can verify that the program works using the following Python script, assuming a program name of sort1mb.exe.

from subprocess import *
import random

sequence = [random.randint(0, 99999999) for i in xrange(1000000)]

sorter = Popen('sort1mb.exe', stdin=PIPE, stdout=PIPE)
for value in sequence:
    sorter.stdin.write('%08d\n' % value)
sorter.stdin.close()

result = [int(line) for line in sorter.stdout]
print('OK!' if result == sorted(sequence) else 'Error!')

A detailed explanation of the algorithm can be found in the following series of posts:

Highlighting Text Color using Html.fromHtml() in Android?

Adding also Kotlin version with:

  • getting text from resources (strings.xml)
  • getting color from resources (colors.xml)
  • "fetching HEX" moved as extension
fun getMulticolorSpanned(): Spanned {
    // Get text from resources
    val text: String = getString(R.string.your_text_from_resources)

    // Get color from resources and parse it to HEX (RGB) value
    val warningHexColor = getHexFromColors(R.color.your_error_color)

    // Use above string & color in HTML
    val html = "<string>$text<span style=\"color:#$warningHexColor;\">*</span></string>"

    // Parse HTML (base on API version)
    return if (Build.VERSION.SDK_INT >= Build.VERSION_CODES.N) {
        Html.fromHtml(html, Html.FROM_HTML_MODE_LEGACY)
    } else {
        Html.fromHtml(html)
    }
}

And Kotlin extension (with removing alpha):

fun Context.getHexFromColors(
    colorRes: Int
): String {
    val labelColor: Int = ContextCompat.getColor(this, colorRes)
    return String.format("%X", labelColor).substring(2)
}

Demo

demo

Sqlite in chrome

I'm not quite sure if you mean 'can i use sqlite (websql) in chrome' or 'can i use sqlite (websql) in firefox', so I'll answer both:

Note that WebSQL is not a full-access pipe into an .sqlite database. It's WebSQL. You will not be able to run some specific queries like VACUUM

It's awesome for Create / Read / Update / Delete though. I made a little library that helps with all the annoying nitty gritty like creating tables and querying and a provides a little ORM/ActiveRecord pattern with relations and all and a huge stack of examples that should get you started in no-time, you can check that here

Also, be aware that if you want to build a FireFox extension: Their extension format is about to change. Make sure you want to invest the time twice.

While the WebSQL spec has been deprecated for years, even now in 2017 still does not look like it will be be removed from Chrome for the foreseeable time. They are tracking usage statistics and there are still a large number of chrome extensions and websites out there in the real world implementing the spec.

UILabel with text of two different colors

NSAttributedString is the way to go. The following question has a great answer that shows you how to do it How do you use NSAttributedString

How to add a custom Ribbon tab using VBA?

I struggled like mad, but this is actually the right answer. For what it is worth, what I missed was is this:

  1. As others say, one can't create the CustomUI ribbon with VBA, however, you don't need to!
  2. The idea is you create your xml Ribbon code using Excel's File > Options > Customize Ribbon, and then export the Ribbon to a .customUI file (it's just a txt file, with xml in it)
  3. Now comes the trick: you can include the .customUI code in your .xlsm file using the MS tool they refer to here, by copying the code from the .customUI file
  4. Once it is included in the .xlsm file, every time you open it, the ribbon you defined is added to the user's ribbon - but do use < ribbon startFromScratch="false" > or you lose the rest of the ribbon. On exit-ing the workbook, the ribbon is removed.
  5. From here on it is simple, create your ribbon, copy the xml code that is specific to your ribbon from the .customUI file, and place it in a wrapper as shown above (...< tabs> your xml < /tabs...)

By the way the page that explains it on Ron's site is now at http://www.rondebruin.nl/win/s2/win002.htm

And here is his example on how you enable /disable buttons on the Ribbon http://www.rondebruin.nl/win/s2/win013.htm

For other xml examples of ribbons please also see http://msdn.microsoft.com/en-us/library/office/aa338202%28v=office.12%29.aspx

How to plot an array in python?

if you give a 2D array to the plot function of matplotlib it will assume the columns to be lines:

If x and/or y is 2-dimensional, then the corresponding columns will be plotted.

In your case your shape is not accepted (100, 1, 1, 8000). As so you can using numpy squeeze to solve the problem quickly:

np.squeez doc: Remove single-dimensional entries from the shape of an array.

import numpy as np
import matplotlib.pyplot as plt

data = np.random.randint(3, 7, (10, 1, 1, 80))
newdata = np.squeeze(data) # Shape is now: (10, 80)
plt.plot(newdata) # plotting by columns
plt.show()

But notice that 100 sets of 80 000 points is a lot of data for matplotlib. I would recommend that you look for an alternative. The result of the code example (run in Jupyter) is:

Jupyter matplotlib plot

mongodb/mongoose findMany - find all documents with IDs listed in array

Ids is the array of object ids:

const ids =  [
    '4ed3ede8844f0f351100000c',
    '4ed3f117a844e0471100000d', 
    '4ed3f18132f50c491100000e',
];

Using Mongoose with callback:

Model.find().where('_id').in(ids).exec((err, records) => {});

Using Mongoose with async function:

const records = await Model.find().where('_id').in(ids).exec();

Or more concise:

const records = await Model.find({ '_id': { $in: ids } });

Don't forget to change Model with your actual model.

Apache Tomcat :java.net.ConnectException: Connection refused

Another possible root cause is that your tomcat has not completely started yet.

If you do a ps -ef| grep apache, you would see the server running and if you check the catalina.out, it will show that the server initialized in 123ms - but it might still be deploying the applications in your webapps directory.

How to call javascript function on page load in asp.net

Calling JavaScript function on code behind i.e. On Page_Load

ClientScript.RegisterStartupScript(GetType(), "Javascript", "javascript:FUNCTIONNAME(); ", true);

If you have UpdatePanel there then try like this

ScriptManager.RegisterStartupScript(GetType(), "Javascript", "javascript:FUNCTIONNAME(); ", true);

View Blog Article : How to Call javascript function from code behind in asp.net c#

How to add row of data to Jtable from values received from jtextfield and comboboxes

String[] tblHead={"Item Name","Price","Qty","Discount"};
DefaultTableModel dtm=new DefaultTableModel(tblHead,0);
JTable tbl=new JTable(dtm);
String[] item={"A","B","C","D"};
dtm.addRow(item);

Here;this is the solution.

pyplot axes labels for subplots

You can create a big subplot that covers the two subplots and then set the common labels.

import random
import matplotlib.pyplot as plt

x = range(1, 101)
y1 = [random.randint(1, 100) for _ in range(len(x))]
y2 = [random.randint(1, 100) for _ in range(len(x))]

fig = plt.figure()
ax = fig.add_subplot(111)    # The big subplot
ax1 = fig.add_subplot(211)
ax2 = fig.add_subplot(212)

# Turn off axis lines and ticks of the big subplot
ax.spines['top'].set_color('none')
ax.spines['bottom'].set_color('none')
ax.spines['left'].set_color('none')
ax.spines['right'].set_color('none')
ax.tick_params(labelcolor='w', top=False, bottom=False, left=False, right=False)

ax1.loglog(x, y1)
ax2.loglog(x, y2)

# Set common labels
ax.set_xlabel('common xlabel')
ax.set_ylabel('common ylabel')

ax1.set_title('ax1 title')
ax2.set_title('ax2 title')

plt.savefig('common_labels.png', dpi=300)

common_labels.png

Another way is using fig.text() to set the locations of the common labels directly.

import random
import matplotlib.pyplot as plt

x = range(1, 101)
y1 = [random.randint(1, 100) for _ in range(len(x))]
y2 = [random.randint(1, 100) for _ in range(len(x))]

fig = plt.figure()
ax1 = fig.add_subplot(211)
ax2 = fig.add_subplot(212)

ax1.loglog(x, y1)
ax2.loglog(x, y2)

# Set common labels
fig.text(0.5, 0.04, 'common xlabel', ha='center', va='center')
fig.text(0.06, 0.5, 'common ylabel', ha='center', va='center', rotation='vertical')

ax1.set_title('ax1 title')
ax2.set_title('ax2 title')

plt.savefig('common_labels_text.png', dpi=300)

common_labels_text.png

Why does git perform fast-forward merges by default?

Let me expand a bit on a VonC's very comprehensive answer:


First, if I remember it correctly, the fact that Git by default doesn't create merge commits in the fast-forward case has come from considering single-branch "equal repositories", where mutual pull is used to sync those two repositories (a workflow you can find as first example in most user's documentation, including "The Git User's Manual" and "Version Control by Example"). In this case you don't use pull to merge fully realized branch, you use it to keep up with other work. You don't want to have ephemeral and unimportant fact when you happen to do a sync saved and stored in repository, saved for the future.

Note that usefulness of feature branches and of having multiple branches in single repository came only later, with more widespread usage of VCS with good merging support, and with trying various merge-based workflows. That is why for example Mercurial originally supported only one branch per repository (plus anonymous tips for tracking remote branches), as seen in older revisions of "Mercurial: The Definitive Guide".


Second, when following best practices of using feature branches, namely that feature branches should all start from stable version (usually from last release), to be able to cherry-pick and select which features to include by selecting which feature branches to merge, you are usually not in fast-forward situation... which makes this issue moot. You need to worry about creating a true merge and not fast-forward when merging a very first branch (assuming that you don't put single-commit changes directly on 'master'); all other later merges are of course in non fast-forward situation.

HTH

How do you check current view controller class in Swift?

if let index = self.navigationController?.viewControllers.index(where: { $0 is MyViewController }) {
            let vc = self.navigationController?.viewControllers[vcIndex] as! MyViewController
            self.navigationController?.popToViewController(vc, animated: true)
        } else {
            self.navigationController?.popToRootViewController(animated: true)
        }

How to set default values in Rails?

For boolean fields in Rails 3.2.6 at least, this will work in your migration.

def change
  add_column :users, :eula_accepted, :boolean, default: false
end

Putting a 1 or 0 for a default will not work here, since it is a boolean field. It must be a true or false value.

What are the complexity guarantees of the standard containers?

I'm not aware of anything like a single table that lets you compare all of them in at one glance (I'm not sure such a table would even be feasible).

Of course the ISO standard document enumerates the complexity requirements in detail, sometimes in various rather readable tables, other times in less readable bullet points for each specific method.

Also the STL library reference at http://www.cplusplus.com/reference/stl/ provides the complexity requirements where appropriate.

Are the decimal places in a CSS width respected?

Even when the number is rounded when the page is painted, the full value is preserved in memory and used for subsequent child calculation. For example, if your box of 100.4999px paints to 100px, it's child with a width of 50% will be calculated as .5*100.4999 instead of .5*100. And so on to deeper levels.

I've created deeply nested grid layout systems where parents widths are ems, and children are percents, and including up to four decimal points upstream had a noticeable impact.

Edge case, sure, but something to keep in mind.

Convert DateTime to a specified Format

Easy peasy:

var date = DateTime.Parse("14/11/2011"); // may need some Culture help here
Console.Write(date.ToString("yyyy-MM-dd"));

Take a look at DateTime.ToString() method, Custom Date and Time Format Strings and Standard Date and Time Format Strings

string customFormattedDateTimeString = DateTime.Now.ToString("yyyy-MM-dd");

TypeError: Converting circular structure to JSON in nodejs

TypeError: Converting circular structure to JSON in nodejs: This error can be seen on Arangodb when using it with Node.js, because storage is missing in your database. If the archive is created under your database, check in the Aurangobi web interface.

Change MySQL root password in phpMyAdmin

You can change the mysql root password by logging in to the database directly (mysql -h your_host -u root) then run

SET PASSWORD FOR root@localhost = PASSWORD('yourpassword');

Insert picture/table in R Markdown

When it comes to inserting a picture, r2evans's suggestion of ![Caption for the picture.](/path/to/image.png) can be problematic if PDF output is required.

The knitr function include_graphics knitr::include_graphics('/path/to/image.png') is a more portable alternative that will generate, on your behalf, the markdown that is most appropriate to the output format that you are generating.

Initialize/reset struct to zero/null

The way to do such a thing when you have modern C (C99) is to use a compound literal.

a = (const struct x){ 0 };

This is somewhat similar to David's solution, only that you don't have to worry to declare an the empty structure or whether to declare it static. If you use the const as I did, the compiler is free to allocate the compound literal statically in read-only storage if appropriate.

How to parse a JSON object to a TypeScript Object

let employee = <Employee>JSON.parse(employeeString);

Remember: Strong typings is compile time only since javascript doesn't support it.

I have never set any passwords to my keystore and alias, so how are they created?

Keystore name: "debug.keystore"

Keystore password: "android"

Key alias: "androiddebugkey"

Key password: "android"

I use this information and successfully generate Signed APK.

Float right and position absolute doesn't work together

I was able to absolutely position a right-floated element with one layer of nesting and a tricky margin:

_x000D_
_x000D_
function test() {_x000D_
  document.getElementById("box").classList.toggle("hide");_x000D_
}
_x000D_
.right {_x000D_
  float:right;_x000D_
}_x000D_
#box {_x000D_
  position:absolute; background:#feb;_x000D_
  width:20em; margin-left:-20em; padding:1ex;_x000D_
}_x000D_
#box.hide {_x000D_
  display:none;_x000D_
}
_x000D_
<div>_x000D_
  <div class="right">_x000D_
    <button onclick="test()">box</button>_x000D_
    <div id="box">Lorem ipsum dolor sit amet, consectetur adipiscing elit,_x000D_
      sed do eiusmod tempor incididunt ut labore et dolore magna aliqua._x000D_
      Ut enim ad minim veniam, quis nostrud exercitation ullamco laboris_x000D_
      nisi ut aliquip ex ea commodo consequat._x000D_
    </div>_x000D_
  </div>_x000D_
  <p>_x000D_
    Lorem ipsum dolor sit amet, consectetur adipiscing elit,_x000D_
    sed do eiusmod tempor incididunt ut labore et dolore magna aliqua._x000D_
    Ut enim ad minim veniam, quis nostrud exercitation ullamco laboris_x000D_
    nisi ut aliquip ex ea commodo consequat._x000D_
  </p>_x000D_
</div>
_x000D_
_x000D_
_x000D_

I decided to make this toggleable so you can see how it does not affect the flow of the surrounding text (run it and press the button to show/hide the floated absolute box).

How to close jQuery Dialog within the dialog?

$(this).parents(".ui-dialog-content").dialog('close')

Simple, I like to make sure I don't:

  1. hardcode dialog #id values.
  2. Close all dialogs.

How to downgrade Node version

 curl -o- https://raw.githubusercontent.com/creationix/nvm/v0.33.11/install.sh | bash
 sudo npm install -g n
 sudo n 10.15
 npm install
 npm audit fix
 npm start

Proper use of mutexes in Python

I don't know why you're using the Window's Mutex instead of Python's. Using the Python methods, this is pretty simple:

from threading import Thread, Lock

mutex = Lock()

def processData(data):
    mutex.acquire()
    try:
        print('Do some stuff')
    finally:
        mutex.release()

while True:
    t = Thread(target = processData, args = (some_data,))
    t.start()

But note, because of the architecture of CPython (namely the Global Interpreter Lock) you'll effectively only have one thread running at a time anyway--this is fine if a number of them are I/O bound, although you'll want to release the lock as much as possible so the I/O bound thread doesn't block other threads from running.

An alternative, for Python 2.6 and later, is to use Python's multiprocessing package. It mirrors the threading package, but will create entirely new processes which can run simultaneously. It's trivial to update your example:

from multiprocessing import Process, Lock

mutex = Lock()

def processData(data):
    with mutex:
        print('Do some stuff')

if __name__ == '__main__':
    while True:
        p = Process(target = processData, args = (some_data,))
        p.start()

How to trigger an event after using event.preventDefault()

Another solution is to use window.setTimeout in the event listener and execute the code after the event's process has finished. Something like...

window.setTimeout(function() {
  // do your thing
}, 0);

I use 0 for the period since I do not care about waiting.

What is the best Java QR code generator library?

QRGen is a good library that creates a layer on top of ZXing and makes QR Code generation in Java a piece of cake.

ScalaTest in sbt: is there a way to run a single test without tags?

This is now supported (since ScalaTest 2.1.3) within interactive mode:

testOnly *MySuite -- -z foo

to run only the tests whose name includes the substring "foo".

For exact match rather than substring, use -t instead of -z.

How to format current time using a yyyyMMddHHmmss format?

Use

fmt.Println(t.Format("20060102150405"))

as Go uses following constants to format date,refer here

const (
    stdLongMonth      = "January"
    stdMonth          = "Jan"
    stdNumMonth       = "1"
    stdZeroMonth      = "01"
    stdLongWeekDay    = "Monday"
    stdWeekDay        = "Mon"
    stdDay            = "2"
    stdUnderDay       = "_2"
    stdZeroDay        = "02"
    stdHour           = "15"
    stdHour12         = "3"
    stdZeroHour12     = "03"
    stdMinute         = "4"
    stdZeroMinute     = "04"
    stdSecond         = "5"
    stdZeroSecond     = "05"
    stdLongYear       = "2006"
    stdYear           = "06"
    stdPM             = "PM"
    stdpm             = "pm"
    stdTZ             = "MST"
    stdISO8601TZ      = "Z0700"  // prints Z for UTC
    stdISO8601ColonTZ = "Z07:00" // prints Z for UTC
    stdNumTZ          = "-0700"  // always numeric
    stdNumShortTZ     = "-07"    // always numeric
    stdNumColonTZ     = "-07:00" // always numeric
)

How to customize Bootstrap 3 tab color

_x000D_
_x000D_
.panel.with-nav-tabs .panel-heading {_x000D_
  padding: 5px 5px 0 5px;_x000D_
}_x000D_
_x000D_
.panel.with-nav-tabs .nav-tabs {_x000D_
  border-bottom: none;_x000D_
}_x000D_
_x000D_
.panel.with-nav-tabs .nav-justified {_x000D_
  margin-bottom: -1px;_x000D_
}_x000D_
_x000D_
_x000D_
/********************************************************************/_x000D_
_x000D_
_x000D_
/*** PANEL DEFAULT ***/_x000D_
_x000D_
.with-nav-tabs.panel-default .nav-tabs>li>a,_x000D_
.with-nav-tabs.panel-default .nav-tabs>li>a:hover,_x000D_
.with-nav-tabs.panel-default .nav-tabs>li>a:focus {_x000D_
  color: #777;_x000D_
}_x000D_
_x000D_
.with-nav-tabs.panel-default .nav-tabs>.open>a,_x000D_
.with-nav-tabs.panel-default .nav-tabs>.open>a:hover,_x000D_
.with-nav-tabs.panel-default .nav-tabs>.open>a:focus,_x000D_
.with-nav-tabs.panel-default .nav-tabs>li>a:hover,_x000D_
.with-nav-tabs.panel-default .nav-tabs>li>a:focus {_x000D_
  color: #777;_x000D_
  background-color: #ddd;_x000D_
  border-color: transparent;_x000D_
}_x000D_
_x000D_
.with-nav-tabs.panel-default .nav-tabs>li.active>a,_x000D_
.with-nav-tabs.panel-default .nav-tabs>li.active>a:hover,_x000D_
.with-nav-tabs.panel-default .nav-tabs>li.active>a:focus {_x000D_
  color: #555;_x000D_
  background-color: #fff;_x000D_
  border-color: #ddd;_x000D_
  border-bottom-color: transparent;_x000D_
}_x000D_
_x000D_
.with-nav-tabs.panel-default .nav-tabs>li.dropdown .dropdown-menu {_x000D_
  background-color: #f5f5f5;_x000D_
  border-color: #ddd;_x000D_
}_x000D_
_x000D_
.with-nav-tabs.panel-default .nav-tabs>li.dropdown .dropdown-menu>li>a {_x000D_
  color: #777;_x000D_
}_x000D_
_x000D_
.with-nav-tabs.panel-default .nav-tabs>li.dropdown .dropdown-menu>li>a:hover,_x000D_
.with-nav-tabs.panel-default .nav-tabs>li.dropdown .dropdown-menu>li>a:focus {_x000D_
  background-color: #ddd;_x000D_
}_x000D_
_x000D_
.with-nav-tabs.panel-default .nav-tabs>li.dropdown .dropdown-menu>.active>a,_x000D_
.with-nav-tabs.panel-default .nav-tabs>li.dropdown .dropdown-menu>.active>a:hover,_x000D_
.with-nav-tabs.panel-default .nav-tabs>li.dropdown .dropdown-menu>.active>a:focus {_x000D_
  color: #fff;_x000D_
  background-color: #555;_x000D_
}_x000D_
_x000D_
_x000D_
/********************************************************************/_x000D_
_x000D_
_x000D_
/*** PANEL PRIMARY ***/_x000D_
_x000D_
.with-nav-tabs.panel-primary .nav-tabs>li>a,_x000D_
.with-nav-tabs.panel-primary .nav-tabs>li>a:hover,_x000D_
.with-nav-tabs.panel-primary .nav-tabs>li>a:focus {_x000D_
  color: #fff;_x000D_
}_x000D_
_x000D_
.with-nav-tabs.panel-primary .nav-tabs>.open>a,_x000D_
.with-nav-tabs.panel-primary .nav-tabs>.open>a:hover,_x000D_
.with-nav-tabs.panel-primary .nav-tabs>.open>a:focus,_x000D_
.with-nav-tabs.panel-primary .nav-tabs>li>a:hover,_x000D_
.with-nav-tabs.panel-primary .nav-tabs>li>a:focus {_x000D_
  color: #fff;_x000D_
  background-color: #3071a9;_x000D_
  border-color: transparent;_x000D_
}_x000D_
_x000D_
.with-nav-tabs.panel-primary .nav-tabs>li.active>a,_x000D_
.with-nav-tabs.panel-primary .nav-tabs>li.active>a:hover,_x000D_
.with-nav-tabs.panel-primary .nav-tabs>li.active>a:focus {_x000D_
  color: #428bca;_x000D_
  background-color: #fff;_x000D_
  border-color: #428bca;_x000D_
  border-bottom-color: transparent;_x000D_
}_x000D_
_x000D_
.with-nav-tabs.panel-primary .nav-tabs>li.dropdown .dropdown-menu {_x000D_
  background-color: #428bca;_x000D_
  border-color: #3071a9;_x000D_
}_x000D_
_x000D_
.with-nav-tabs.panel-primary .nav-tabs>li.dropdown .dropdown-menu>li>a {_x000D_
  color: #fff;_x000D_
}_x000D_
_x000D_
.with-nav-tabs.panel-primary .nav-tabs>li.dropdown .dropdown-menu>li>a:hover,_x000D_
.with-nav-tabs.panel-primary .nav-tabs>li.dropdown .dropdown-menu>li>a:focus {_x000D_
  background-color: #3071a9;_x000D_
}_x000D_
_x000D_
.with-nav-tabs.panel-primary .nav-tabs>li.dropdown .dropdown-menu>.active>a,_x000D_
.with-nav-tabs.panel-primary .nav-tabs>li.dropdown .dropdown-menu>.active>a:hover,_x000D_
.with-nav-tabs.panel-primary .nav-tabs>li.dropdown .dropdown-menu>.active>a:focus {_x000D_
  background-color: #4a9fe9;_x000D_
}_x000D_
_x000D_
_x000D_
/********************************************************************/_x000D_
_x000D_
_x000D_
/*** PANEL SUCCESS ***/_x000D_
_x000D_
.with-nav-tabs.panel-success .nav-tabs>li>a,_x000D_
.with-nav-tabs.panel-success .nav-tabs>li>a:hover,_x000D_
.with-nav-tabs.panel-success .nav-tabs>li>a:focus {_x000D_
  color: #3c763d;_x000D_
}_x000D_
_x000D_
.with-nav-tabs.panel-success .nav-tabs>.open>a,_x000D_
.with-nav-tabs.panel-success .nav-tabs>.open>a:hover,_x000D_
.with-nav-tabs.panel-success .nav-tabs>.open>a:focus,_x000D_
.with-nav-tabs.panel-success .nav-tabs>li>a:hover,_x000D_
.with-nav-tabs.panel-success .nav-tabs>li>a:focus {_x000D_
  color: #3c763d;_x000D_
  background-color: #d6e9c6;_x000D_
  border-color: transparent;_x000D_
}_x000D_
_x000D_
.with-nav-tabs.panel-success .nav-tabs>li.active>a,_x000D_
.with-nav-tabs.panel-success .nav-tabs>li.active>a:hover,_x000D_
.with-nav-tabs.panel-success .nav-tabs>li.active>a:focus {_x000D_
  color: #3c763d;_x000D_
  background-color: #fff;_x000D_
  border-color: #d6e9c6;_x000D_
  border-bottom-color: transparent;_x000D_
}_x000D_
_x000D_
.with-nav-tabs.panel-success .nav-tabs>li.dropdown .dropdown-menu {_x000D_
  background-color: #dff0d8;_x000D_
  border-color: #d6e9c6;_x000D_
}_x000D_
_x000D_
.with-nav-tabs.panel-success .nav-tabs>li.dropdown .dropdown-menu>li>a {_x000D_
  color: #3c763d;_x000D_
}_x000D_
_x000D_
.with-nav-tabs.panel-success .nav-tabs>li.dropdown .dropdown-menu>li>a:hover,_x000D_
.with-nav-tabs.panel-success .nav-tabs>li.dropdown .dropdown-menu>li>a:focus {_x000D_
  background-color: #d6e9c6;_x000D_
}_x000D_
_x000D_
.with-nav-tabs.panel-success .nav-tabs>li.dropdown .dropdown-menu>.active>a,_x000D_
.with-nav-tabs.panel-success .nav-tabs>li.dropdown .dropdown-menu>.active>a:hover,_x000D_
.with-nav-tabs.panel-success .nav-tabs>li.dropdown .dropdown-menu>.active>a:focus {_x000D_
  color: #fff;_x000D_
  background-color: #3c763d;_x000D_
}_x000D_
_x000D_
_x000D_
/********************************************************************/_x000D_
_x000D_
_x000D_
/*** PANEL INFO ***/_x000D_
_x000D_
.with-nav-tabs.panel-info .nav-tabs>li>a,_x000D_
.with-nav-tabs.panel-info .nav-tabs>li>a:hover,_x000D_
.with-nav-tabs.panel-info .nav-tabs>li>a:focus {_x000D_
  color: #31708f;_x000D_
}_x000D_
_x000D_
.with-nav-tabs.panel-info .nav-tabs>.open>a,_x000D_
.with-nav-tabs.panel-info .nav-tabs>.open>a:hover,_x000D_
.with-nav-tabs.panel-info .nav-tabs>.open>a:focus,_x000D_
.with-nav-tabs.panel-info .nav-tabs>li>a:hover,_x000D_
.with-nav-tabs.panel-info .nav-tabs>li>a:focus {_x000D_
  color: #31708f;_x000D_
  background-color: #bce8f1;_x000D_
  border-color: transparent;_x000D_
}_x000D_
_x000D_
.with-nav-tabs.panel-info .nav-tabs>li.active>a,_x000D_
.with-nav-tabs.panel-info .nav-tabs>li.active>a:hover,_x000D_
.with-nav-tabs.panel-info .nav-tabs>li.active>a:focus {_x000D_
  color: #31708f;_x000D_
  background-color: #fff;_x000D_
  border-color: #bce8f1;_x000D_
  border-bottom-color: transparent;_x000D_
}_x000D_
_x000D_
.with-nav-tabs.panel-info .nav-tabs>li.dropdown .dropdown-menu {_x000D_
  background-color: #d9edf7;_x000D_
  border-color: #bce8f1;_x000D_
}_x000D_
_x000D_
.with-nav-tabs.panel-info .nav-tabs>li.dropdown .dropdown-menu>li>a {_x000D_
  color: #31708f;_x000D_
}_x000D_
_x000D_
.with-nav-tabs.panel-info .nav-tabs>li.dropdown .dropdown-menu>li>a:hover,_x000D_
.with-nav-tabs.panel-info .nav-tabs>li.dropdown .dropdown-menu>li>a:focus {_x000D_
  background-color: #bce8f1;_x000D_
}_x000D_
_x000D_
.with-nav-tabs.panel-info .nav-tabs>li.dropdown .dropdown-menu>.active>a,_x000D_
.with-nav-tabs.panel-info .nav-tabs>li.dropdown .dropdown-menu>.active>a:hover,_x000D_
.with-nav-tabs.panel-info .nav-tabs>li.dropdown .dropdown-menu>.active>a:focus {_x000D_
  color: #fff;_x000D_
  background-color: #31708f;_x000D_
}_x000D_
_x000D_
_x000D_
/********************************************************************/_x000D_
_x000D_
_x000D_
/*** PANEL WARNING ***/_x000D_
_x000D_
.with-nav-tabs.panel-warning .nav-tabs>li>a,_x000D_
.with-nav-tabs.panel-warning .nav-tabs>li>a:hover,_x000D_
.with-nav-tabs.panel-warning .nav-tabs>li>a:focus {_x000D_
  color: #8a6d3b;_x000D_
}_x000D_
_x000D_
.with-nav-tabs.panel-warning .nav-tabs>.open>a,_x000D_
.with-nav-tabs.panel-warning .nav-tabs>.open>a:hover,_x000D_
.with-nav-tabs.panel-warning .nav-tabs>.open>a:focus,_x000D_
.with-nav-tabs.panel-warning .nav-tabs>li>a:hover,_x000D_
.with-nav-tabs.panel-warning .nav-tabs>li>a:focus {_x000D_
  color: #8a6d3b;_x000D_
  background-color: #faebcc;_x000D_
  border-color: transparent;_x000D_
}_x000D_
_x000D_
.with-nav-tabs.panel-warning .nav-tabs>li.active>a,_x000D_
.with-nav-tabs.panel-warning .nav-tabs>li.active>a:hover,_x000D_
.with-nav-tabs.panel-warning .nav-tabs>li.active>a:focus {_x000D_
  color: #8a6d3b;_x000D_
  background-color: #fff;_x000D_
  border-color: #faebcc;_x000D_
  border-bottom-color: transparent;_x000D_
}_x000D_
_x000D_
.with-nav-tabs.panel-warning .nav-tabs>li.dropdown .dropdown-menu {_x000D_
  background-color: #fcf8e3;_x000D_
  border-color: #faebcc;_x000D_
}_x000D_
_x000D_
.with-nav-tabs.panel-warning .nav-tabs>li.dropdown .dropdown-menu>li>a {_x000D_
  color: #8a6d3b;_x000D_
}_x000D_
_x000D_
.with-nav-tabs.panel-warning .nav-tabs>li.dropdown .dropdown-menu>li>a:hover,_x000D_
.with-nav-tabs.panel-warning .nav-tabs>li.dropdown .dropdown-menu>li>a:focus {_x000D_
  background-color: #faebcc;_x000D_
}_x000D_
_x000D_
.with-nav-tabs.panel-warning .nav-tabs>li.dropdown .dropdown-menu>.active>a,_x000D_
.with-nav-tabs.panel-warning .nav-tabs>li.dropdown .dropdown-menu>.active>a:hover,_x000D_
.with-nav-tabs.panel-warning .nav-tabs>li.dropdown .dropdown-menu>.active>a:focus {_x000D_
  color: #fff;_x000D_
  background-color: #8a6d3b;_x000D_
}_x000D_
_x000D_
_x000D_
/********************************************************************/_x000D_
_x000D_
_x000D_
/*** PANEL DANGER ***/_x000D_
_x000D_
.with-nav-tabs.panel-danger .nav-tabs>li>a,_x000D_
.with-nav-tabs.panel-danger .nav-tabs>li>a:hover,_x000D_
.with-nav-tabs.panel-danger .nav-tabs>li>a:focus {_x000D_
  color: #a94442;_x000D_
}_x000D_
_x000D_
.with-nav-tabs.panel-danger .nav-tabs>.open>a,_x000D_
.with-nav-tabs.panel-danger .nav-tabs>.open>a:hover,_x000D_
.with-nav-tabs.panel-danger .nav-tabs>.open>a:focus,_x000D_
.with-nav-tabs.panel-danger .nav-tabs>li>a:hover,_x000D_
.with-nav-tabs.panel-danger .nav-tabs>li>a:focus {_x000D_
  color: #a94442;_x000D_
  background-color: #ebccd1;_x000D_
  border-color: transparent;_x000D_
}_x000D_
_x000D_
.with-nav-tabs.panel-danger .nav-tabs>li.active>a,_x000D_
.with-nav-tabs.panel-danger .nav-tabs>li.active>a:hover,_x000D_
.with-nav-tabs.panel-danger .nav-tabs>li.active>a:focus {_x000D_
  color: #a94442;_x000D_
  background-color: #fff;_x000D_
  border-color: #ebccd1;_x000D_
  border-bottom-color: transparent;_x000D_
}_x000D_
_x000D_
.with-nav-tabs.panel-danger .nav-tabs>li.dropdown .dropdown-menu {_x000D_
  background-color: #f2dede;_x000D_
  /* bg color */_x000D_
  border-color: #ebccd1;_x000D_
  /* border color */_x000D_
}_x000D_
_x000D_
.with-nav-tabs.panel-danger .nav-tabs>li.dropdown .dropdown-menu>li>a {_x000D_
  color: #a94442;_x000D_
  /* normal text color */_x000D_
}_x000D_
_x000D_
.with-nav-tabs.panel-danger .nav-tabs>li.dropdown .dropdown-menu>li>a:hover,_x000D_
.with-nav-tabs.panel-danger .nav-tabs>li.dropdown .dropdown-menu>li>a:focus {_x000D_
  background-color: #ebccd1;_x000D_
  /* hover bg color */_x000D_
}_x000D_
_x000D_
.with-nav-tabs.panel-danger .nav-tabs>li.dropdown .dropdown-menu>.active>a,_x000D_
.with-nav-tabs.panel-danger .nav-tabs>li.dropdown .dropdown-menu>.active>a:hover,_x000D_
.with-nav-tabs.panel-danger .nav-tabs>li.dropdown .dropdown-menu>.active>a:focus {_x000D_
  color: #fff;_x000D_
  /* active text color */_x000D_
  background-color: #a94442;_x000D_
  /* active bg color */_x000D_
}
_x000D_
<script src="https://cdnjs.cloudflare.com/ajax/libs/jquery/3.3.1/jquery.min.js"></script>_x000D_
<link href="//netdna.bootstrapcdn.com/bootstrap/3.2.0/css/bootstrap.min.css" rel="stylesheet" id="bootstrap-css">_x000D_
<script src="//netdna.bootstrapcdn.com/bootstrap/3.2.0/js/bootstrap.min.js"></script>_x000D_
<!------ Include the above in your HEAD tag ---------->_x000D_
_x000D_
<div class="container">_x000D_
  <div class="page-header">_x000D_
    <h1>Panels with nav tabs.<span class="pull-right label label-default">:)</span></h1>_x000D_
  </div>_x000D_
  <div class="row">_x000D_
    <div class="col-md-6">_x000D_
      <div class="panel with-nav-tabs panel-default">_x000D_
        <div class="panel-heading">_x000D_
          <ul class="nav nav-tabs">_x000D_
            <li class="active"><a href="#tab1default" data-toggle="tab">Default 1</a></li>_x000D_
            <li><a href="#tab2default" data-toggle="tab">Default 2</a></li>_x000D_
            <li><a href="#tab3default" data-toggle="tab">Default 3</a></li>_x000D_
            <li class="dropdown">_x000D_
              <a href="#" data-toggle="dropdown">Dropdown <span class="caret"></span></a>_x000D_
              <ul class="dropdown-menu" role="menu">_x000D_
                <li><a href="#tab4default" data-toggle="tab">Default 4</a></li>_x000D_
                <li><a href="#tab5default" data-toggle="tab">Default 5</a></li>_x000D_
              </ul>_x000D_
            </li>_x000D_
          </ul>_x000D_
        </div>_x000D_
        <div class="panel-body">_x000D_
          <div class="tab-content">_x000D_
            <div class="tab-pane fade in active" id="tab1default">Default 1</div>_x000D_
            <div class="tab-pane fade" id="tab2default">Default 2</div>_x000D_
            <div class="tab-pane fade" id="tab3default">Default 3</div>_x000D_
            <div class="tab-pane fade" id="tab4default">Default 4</div>_x000D_
            <div class="tab-pane fade" id="tab5default">Default 5</div>_x000D_
          </div>_x000D_
        </div>_x000D_
      </div>_x000D_
    </div>_x000D_
    <div class="col-md-6">_x000D_
      <div class="panel with-nav-tabs panel-primary">_x000D_
        <div class="panel-heading">_x000D_
          <ul class="nav nav-tabs">_x000D_
            <li class="active"><a href="#tab1primary" data-toggle="tab">Primary 1</a></li>_x000D_
            <li><a href="#tab2primary" data-toggle="tab">Primary 2</a></li>_x000D_
            <li><a href="#tab3primary" data-toggle="tab">Primary 3</a></li>_x000D_
            <li class="dropdown">_x000D_
              <a href="#" data-toggle="dropdown">Dropdown <span class="caret"></span></a>_x000D_
              <ul class="dropdown-menu" role="menu">_x000D_
                <li><a href="#tab4primary" data-toggle="tab">Primary 4</a></li>_x000D_
                <li><a href="#tab5primary" data-toggle="tab">Primary 5</a></li>_x000D_
              </ul>_x000D_
            </li>_x000D_
          </ul>_x000D_
        </div>_x000D_
        <div class="panel-body">_x000D_
          <div class="tab-content">_x000D_
            <div class="tab-pane fade in active" id="tab1primary">Primary 1</div>_x000D_
            <div class="tab-pane fade" id="tab2primary">Primary 2</div>_x000D_
            <div class="tab-pane fade" id="tab3primary">Primary 3</div>_x000D_
            <div class="tab-pane fade" id="tab4primary">Primary 4</div>_x000D_
            <div class="tab-pane fade" id="tab5primary">Primary 5</div>_x000D_
          </div>_x000D_
        </div>_x000D_
      </div>_x000D_
    </div>_x000D_
  </div>_x000D_
</div>_x000D_
<div class="container">_x000D_
  <div class="row">_x000D_
    <div class="col-md-6">_x000D_
      <div class="panel with-nav-tabs panel-success">_x000D_
        <div class="panel-heading">_x000D_
          <ul class="nav nav-tabs">_x000D_
            <li class="active"><a href="#tab1success" data-toggle="tab">Success 1</a></li>_x000D_
            <li><a href="#tab2success" data-toggle="tab">Success 2</a></li>_x000D_
            <li><a href="#tab3success" data-toggle="tab">Success 3</a></li>_x000D_
            <li class="dropdown">_x000D_
              <a href="#" data-toggle="dropdown">Dropdown <span class="caret"></span></a>_x000D_
              <ul class="dropdown-menu" role="menu">_x000D_
                <li><a href="#tab4success" data-toggle="tab">Success 4</a></li>_x000D_
                <li><a href="#tab5success" data-toggle="tab">Success 5</a></li>_x000D_
              </ul>_x000D_
            </li>_x000D_
          </ul>_x000D_
        </div>_x000D_
        <div class="panel-body">_x000D_
          <div class="tab-content">_x000D_
            <div class="tab-pane fade in active" id="tab1success">Success 1</div>_x000D_
            <div class="tab-pane fade" id="tab2success">Success 2</div>_x000D_
            <div class="tab-pane fade" id="tab3success">Success 3</div>_x000D_
            <div class="tab-pane fade" id="tab4success">Success 4</div>_x000D_
            <div class="tab-pane fade" id="tab5success">Success 5</div>_x000D_
          </div>_x000D_
        </div>_x000D_
      </div>_x000D_
    </div>_x000D_
    <div class="col-md-6">_x000D_
      <div class="panel with-nav-tabs panel-info">_x000D_
        <div class="panel-heading">_x000D_
          <ul class="nav nav-tabs">_x000D_
            <li class="active"><a href="#tab1info" data-toggle="tab">Info 1</a></li>_x000D_
            <li><a href="#tab2info" data-toggle="tab">Info 2</a></li>_x000D_
            <li><a href="#tab3info" data-toggle="tab">Info 3</a></li>_x000D_
            <li class="dropdown">_x000D_
              <a href="#" data-toggle="dropdown">Dropdown <span class="caret"></span></a>_x000D_
              <ul class="dropdown-menu" role="menu">_x000D_
                <li><a href="#tab4info" data-toggle="tab">Info 4</a></li>_x000D_
                <li><a href="#tab5info" data-toggle="tab">Info 5</a></li>_x000D_
              </ul>_x000D_
            </li>_x000D_
          </ul>_x000D_
        </div>_x000D_
        <div class="panel-body">_x000D_
          <div class="tab-content">_x000D_
            <div class="tab-pane fade in active" id="tab1info">Info 1</div>_x000D_
            <div class="tab-pane fade" id="tab2info">Info 2</div>_x000D_
            <div class="tab-pane fade" id="tab3info">Info 3</div>_x000D_
            <div class="tab-pane fade" id="tab4info">Info 4</div>_x000D_
            <div class="tab-pane fade" id="tab5info">Info 5</div>_x000D_
          </div>_x000D_
        </div>_x000D_
      </div>_x000D_
    </div>_x000D_
  </div>_x000D_
</div>_x000D_
<div class="container">_x000D_
  <div class="row">_x000D_
    <div class="col-md-6">_x000D_
      <div class="panel with-nav-tabs panel-warning">_x000D_
        <div class="panel-heading">_x000D_
          <ul class="nav nav-tabs">_x000D_
            <li class="active"><a href="#tab1warning" data-toggle="tab">Warning 1</a></li>_x000D_
            <li><a href="#tab2warning" data-toggle="tab">Warning 2</a></li>_x000D_
            <li><a href="#tab3warning" data-toggle="tab">Warning 3</a></li>_x000D_
            <li class="dropdown">_x000D_
              <a href="#" data-toggle="dropdown">Dropdown <span class="caret"></span></a>_x000D_
              <ul class="dropdown-menu" role="menu">_x000D_
                <li><a href="#tab4warning" data-toggle="tab">Warning 4</a></li>_x000D_
                <li><a href="#tab5warning" data-toggle="tab">Warning 5</a></li>_x000D_
              </ul>_x000D_
            </li>_x000D_
          </ul>_x000D_
        </div>_x000D_
        <div class="panel-body">_x000D_
          <div class="tab-content">_x000D_
            <div class="tab-pane fade in active" id="tab1warning">Warning 1</div>_x000D_
            <div class="tab-pane fade" id="tab2warning">Warning 2</div>_x000D_
            <div class="tab-pane fade" id="tab3warning">Warning 3</div>_x000D_
            <div class="tab-pane fade" id="tab4warning">Warning 4</div>_x000D_
            <div class="tab-pane fade" id="tab5warning">Warning 5</div>_x000D_
          </div>_x000D_
        </div>_x000D_
      </div>_x000D_
    </div>_x000D_
    <div class="col-md-6">_x000D_
      <div class="panel with-nav-tabs panel-danger">_x000D_
        <div class="panel-heading">_x000D_
          <ul class="nav nav-tabs">_x000D_
            <li class="active"><a href="#tab1danger" data-toggle="tab">Danger 1</a></li>_x000D_
            <li><a href="#tab2danger" data-toggle="tab">Danger 2</a></li>_x000D_
            <li><a href="#tab3danger" data-toggle="tab">Danger 3</a></li>_x000D_
            <li class="dropdown">_x000D_
              <a href="#" data-toggle="dropdown">Dropdown <span class="caret"></span></a>_x000D_
              <ul class="dropdown-menu" role="menu">_x000D_
                <li><a href="#tab4danger" data-toggle="tab">Danger 4</a></li>_x000D_
                <li><a href="#tab5danger" data-toggle="tab">Danger 5</a></li>_x000D_
              </ul>_x000D_
            </li>_x000D_
          </ul>_x000D_
        </div>_x000D_
        <div class="panel-body">_x000D_
          <div class="tab-content">_x000D_
            <div class="tab-pane fade in active" id="tab1danger">Danger 1</div>_x000D_
            <div class="tab-pane fade" id="tab2danger">Danger 2</div>_x000D_
            <div class="tab-pane fade" id="tab3danger">Danger 3</div>_x000D_
            <div class="tab-pane fade" id="tab4danger">Danger 4</div>_x000D_
            <div class="tab-pane fade" id="tab5danger">Danger 5</div>_x000D_
          </div>_x000D_
        </div>_x000D_
      </div>_x000D_
    </div>_x000D_
  </div>_x000D_
</div>_x000D_
<br/>
_x000D_
_x000D_
_x000D_

Which TensorFlow and CUDA version combinations are compatible?

The compatibility table given in the tensorflow site does not contain specific minor versions for cuda and cuDNN. However, if the specific versions are not met, there will be an error when you try to use tensorflow.

For tensorflow-gpu==1.12.0 and cuda==9.0, the compatible cuDNN version is 7.1.4, which can be downloaded from here after registration.

You can check your cuda version using
nvcc --version

cuDNN version using
cat /usr/include/cudnn.h | grep CUDNN_MAJOR -A 2

tensorflow-gpu version using
pip freeze | grep tensorflow-gpu

UPDATE: Since tensorflow 2.0, has been released, I will share the compatible cuda and cuDNN versions for it as well (for Ubuntu 18.04).

  • tensorflow-gpu = 2.0.0
  • cuda = 10.0
  • cuDNN = 7.6.0

How to add double quotes to a string that is inside a variable?

An indirect, but simple to understand alternative to add quotes at start and end of string -

char quote = '"';

string modifiedString = quote + "Original String" + quote;

Add / remove input field dynamically with jQuery

Here is my JSFiddle with corrected line breaks, or see it below.

$(document).ready(function() {

var MaxInputs       = 2; //maximum extra input boxes allowed
var InputsWrapper   = $("#InputsWrapper"); //Input boxes wrapper ID
var AddButton       = $("#AddMoreFileBox"); //Add button ID

var x = InputsWrapper.length; //initlal text box count
var FieldCount=1; //to keep track of text box added

//on add input button click
$(AddButton).click(function (e) {
        //max input box allowed
        if(x <= MaxInputs) {
            FieldCount++; //text box added ncrement
            //add input box
            $(InputsWrapper).append('<div><input type="text" name="mytext[]" id="field_'+ FieldCount +'"/> <a href="#" class="removeclass">Remove</a></div>');
            x++; //text box increment

            $("#AddMoreFileId").show();

            $('AddMoreFileBox').html("Add field");

            // Delete the "add"-link if there is 3 fields.
            if(x == 3) {
                $("#AddMoreFileId").hide();
                $("#lineBreak").html("<br>");
            }
        }
        return false;
});

$("body").on("click",".removeclass", function(e){ //user click on remove text
        if( x > 1 ) {
                $(this).parent('div').remove(); //remove text box
                x--; //decrement textbox

                $("#AddMoreFileId").show();

                $("#lineBreak").html("");

                // Adds the "add" link again when a field is removed.
                $('AddMoreFileBox').html("Add field");
        }
    return false;
}) 

});

How do I display a text file content in CMD?

tail -3 d:\text_file.txt

tail -1 d:\text_file.txt

I assume this was added to Windows cmd.exe at some point.

Changing Fonts Size in Matlab Plots

To change the title font size, use the following example

title('mytitle','FontSize',12);

to the change the graph axes label font size, do the following

axes('FontSize',24);

How can I stop a running MySQL query?

You need to run following command to kill the process. Find out the id of the process which you wanted to kill by

> show processlist;

Take the value from id column and fire below command

kill query <processId>;

Query parameter specifies that we need to kill query command process.

The syntax for kill process as follows

KILL [CONNECTION | QUERY] processlist_id

Please refer this link for more information.

How to find a value in an excel column by vba code Cells.Find

Just for sake of completeness, you can also use the same technique above with excel tables.

In the example below, I'm looking of a text in any cell of a Excel Table named "tblConfig", place in the sheet named Config that normally is set to be hidden. I'm accepting the defaults of the Find method.

Dim list As ListObject
Dim config As Worksheet
Dim cell as Range


Set config = Sheets("Config")
Set list = config.ListObjects("tblConfig")

'search in any cell of the data range of excel table
Set cell = list.DataBodyRange.Find(searchTerm)

If cell Is Nothing Then
    'when information is not found
Else
    'when information is found
End If

Most efficient way to concatenate strings in JavaScript?

Seems based on benchmarks at JSPerf that using += is the fastest method, though not necessarily in every browser.

For building strings in the DOM, it seems to be better to concatenate the string first and then add to the DOM, rather then iteratively add it to the dom. You should benchmark your own case though.

(Thanks @zAlbee for correction)

Comparing two .jar files

I use to ZipDiff lib (have both Java and ant API).

How do I perform HTML decoding/encoding using Python/Django?

See at the bottom of this page at Python wiki, there are at least 2 options to "unescape" html.

How to install/start Postman native v4.10.3 on Ubuntu 16.04 LTS 64-bit?

sudo snap install postman

This single command worked for me.

Visual Studio 2017 does not have Business Intelligence Integration Services/Projects

VS2017 supports ssis or ssrs projects if you install SSDT for VS2017 here.

Click on the newly downloaded file and check SSIS or SSRS components that you required, as show in diagram :-

enter image description here

Once you have installed this, try opening ssis / ssrs project. I managed to open ssis developed on vs2010.

You should see these component installed. (reboot if you don't see them).

enter image description here

Try open your project again. If you get 'incompatible project' - right click on your project, select "reload project" (not reopen the solution)

Split string, convert ToList<int>() in one line

You can also do it this way without the need of Linq:

List<int> numbers = new List<int>( Array.ConvertAll(sNumbers.Split(','), int.Parse) );

// Uses Linq
var numbers = Array.ConvertAll(sNumbers.Split(','), int.Parse).ToList();

How do I delete virtual interface in Linux?

Have you tried:

ifconfig 10:35978f0 down

As the physical interface is 10 and the virtual aspect is after the colon :.

See also https://www.cyberciti.biz/faq/linux-command-to-remove-virtual-interfaces-or-network-aliases/

MySQL - How to select data by string length

I used this sentences to filter

SELECT table.field1, table.field2 FROM table WHERE length(field) > 10;

you can change 10 for other number that you want to filter.

MongoDb query condition on comparing 2 fields

In case performance is more important than readability and as long as your condition consists of simple arithmetic operations, you can use aggregation pipeline. First, use $project to calculate the left hand side of the condition (take all fields to left hand side). Then use $match to compare with a constant and filter. This way you avoid javascript execution. Below is my test in python:

import pymongo
from random import randrange

docs = [{'Grade1': randrange(10), 'Grade2': randrange(10)} for __ in range(100000)]

coll = pymongo.MongoClient().test_db.grades
coll.insert_many(docs)

Using aggregate:

%timeit -n1 -r1 list(coll.aggregate([
    {
        '$project': {
            'diff': {'$subtract': ['$Grade1', '$Grade2']},
            'Grade1': 1,
            'Grade2': 1
        }
    },
    {
        '$match': {'diff': {'$gt': 0}}
    }
]))

1 loop, best of 1: 192 ms per loop

Using find and $where:

%timeit -n1 -r1 list(coll.find({'$where': 'this.Grade1 > this.Grade2'}))

1 loop, best of 1: 4.54 s per loop

Does JSON syntax allow duplicate keys in an object?

From the standard (p. ii):

It is expected that other standards will refer to this one, strictly adhering to the JSON text format, while imposing restrictions on various encoding details. Such standards may require specific behaviours. JSON itself specifies no behaviour.

Further down in the standard (p. 2), the specification for a JSON object:

An object structure is represented as a pair of curly bracket tokens surrounding zero or more name/value pairs. A name is a string. A single colon token follows each name, separating the name from the value. A single comma token separates a value from a following name.

Diagram for JSON Object

It does not make any mention of duplicate keys being invalid or valid, so according to the specification I would safely assume that means they are allowed.

That most implementations of JSON libraries do not accept duplicate keys does not conflict with the standard, because of the first quote.

Here are two examples related to the C++ standard library. When deserializing some JSON object into a std::map it would make sense to refuse duplicate keys. But when deserializing some JSON object into a std::multimap it would make sense to accept duplicate keys as normal.

Operand type clash: int is incompatible with date + The INSERT statement conflicted with the FOREIGN KEY constraint

This expression 12-4-2005 is a calculated int and the value is -1997. You should do like this instead '2005-04-12' with the ' before and after.

Credentials for the SQL Server Agent service are invalid

I've had this error as a result of trying to use a cloned VM that had the same SID as the domain. The two options to fix it were: sysprep (or rebuild) the database server OR dcpromo the DC down and back up to change the domain SID.

MySQL Sum() multiple columns

//Mysql sum of multiple rows Hi Here is the simple way to do sum of columns

SELECT sum(IF(day_1 = 1,1,0)+IF(day_3 = 1,1,0)++IF(day_4 = 1,1,0)) from attendence WHERE class_period_id='1' and student_id='1'

Dynamic type languages versus static type languages

Perhaps the single biggest "benefit" of dynamic typing is the shallower learning curve. There is no type system to learn and no non-trivial syntax for corner cases such as type constraints. That makes dynamic typing accessible to a lot more people and feasible for many people for whom sophisticated static type systems are out of reach. Consequently, dynamic typing has caught on in the contexts of education (e.g. Scheme/Python at MIT) and domain-specific languages for non-programmers (e.g. Mathematica). Dynamic languages have also caught on in niches where they have little or no competition (e.g. Javascript).

The most concise dynamically-typed languages (e.g. Perl, APL, J, K, Mathematica) are domain specific and can be significantly more concise than the most concise general-purpose statically-typed languages (e.g. OCaml) in the niches they were designed for.

The main disadvantages of dynamic typing are:

  • Run-time type errors.

  • Can be very difficult or even practically impossible to achieve the same level of correctness and requires vastly more testing.

  • No compiler-verified documentation.

  • Poor performance (usually at run-time but sometimes at compile time instead, e.g. Stalin Scheme) and unpredictable performance due to dependence upon sophisticated optimizations.

Personally, I grew up on dynamic languages but wouldn't touch them with a 40' pole as a professional unless there were no other viable options.

Xampp MySQL not starting - "Attempting to start MySQL service..."

Everytime my MySQL starts, it will stop, then I noticed that some files are getting generated in C:\xampp\mysql\data. I tried deleting some files (like the Error logs, err file, etc.) on that directory. Back up first what you're going to delete to avoid losing data.

I don't know how it works. I'm just trying to enable HTTPS in my local machine but then suddenly MySQL can't be started. But now it's working.

I'm using Windows 10 and XAMPP 3.2.4.

How do you redirect HTTPS to HTTP?

For those that are using a .conf file.

<VirtualHost *:443>
    ServerName domain.com
    RewriteEngine On
    RewriteCond %{HTTPS} on
    RewriteRule (.*) http://%{HTTP_HOST}%{REQUEST_URI}

    SSLEngine on
    SSLCertificateFile /etc/apache2/ssl/domain.crt
    SSLCertificateKeyFile /etc/apache2/ssl/domain.key
    SSLCACertificateFile /etc/apache2/ssl/domain.crt

</VirtualHost>

VueJs get url query

You can also get them with pure javascript.

For example:

new URL(location.href).searchParams.get('page')

For this url: websitename.com/user/?page=1, it would return a value of 1

How to declare an array of strings in C++?

Problems - no way to get the number of strings automatically (that i know of).

There is a bog-standard way of doing this, which lots of people (including MS) define macros like arraysize for:

#define arraysize(ar)  (sizeof(ar) / sizeof(ar[0]))

How can I convert a Timestamp into either Date or DateTime object?

java.time

Modern answer: use java.time, the modern Java date and time API, for your date and time work. Back in 2011 it was right to use the Timestamp class, but since JDBC 4.2 it is no longer advised.

For your work we need a time zone and a couple of formatters. We may as well declare them static:

static ZoneId zone = ZoneId.of("America/Marigot");
static DateTimeFormatter dateFormatter = DateTimeFormatter.ofPattern("MM/dd/uuuu");
static DateTimeFormatter timeFormatter = DateTimeFormatter.ofPattern("HH:mm xx");

Now the code could be for example:

    while(resultSet.next()) {
        ZonedDateTime dtStart = resultSet.getObject("dtStart", OffsetDateTime.class)
                 .atZoneSameInstant(zone);

        // I would like to then have the date and time
        // converted into the formats mentioned...
        String dateFormatted = dtStart.format(dateFormatter);
        String timeFormatted = dtStart.format(timeFormatter);
        System.out.format("Date: %s; time: %s%n", dateFormatted, timeFormatted);
    }

Example output (using the time your question was asked):

Date: 09/20/2011; time: 18:13 -0400

In your database timestamp with time zone is recommended for timestamps. If this is what you’ve got, retrieve an OffsetDateTime as I am doing in the code. I am also converting the retrieved value to the user’s time zone before formatting date and time separately. As time zone I supplied America/Marigot as an example, please supply your own. You may also leave out the time zone conversion if you don’t want any, of course.

If the datatype in SQL is a mere timestamp without time zone, retrieve a LocalDateTime instead. For example:

        ZonedDateTime dtStart = resultSet.getObject("dtStart", LocalDateTime.class)
                 .atZone(zone);

No matter the details I trust you to do similarly for dtEnd.

I wasn’t sure what you meant by the xx in HH:MM xx. I just left it in the format pattern string, which yields the UTC offset in hours and minutes without colon.

Link: Oracle tutorial: Date Time explaining how to use java.time.